Você está na página 1de 195

Lecture notes to the 1-st year master course

Particle Physics 1
Nikhef - Autumn 2013

Marcel Merk (marcel.merk@nikhef.nl) Wouter Hulsbergen (wouter.hulsbergen@nikhef.nl)

Contents
Preliminaries 1 Particles and Forces 1.1 The Yukawa Potential . 1.2 Strange Particles . . . . 1.3 The Eightfold Way . . . 1.4 The Quark Model . . . . 1.5 The Standard Model . . 1.6 Units in particle physics i 1 1 4 6 6 12 14 19 22 23 25 26 28 29 30 30 35 35 37 38 41 43 47 47 49 54 55 56 57

. . . . . .

. . . . . .

. . . . . .

. . . . . .

. . . . . .

. . . . . .

. . . . . .

. . . . . .

. . . . . .

. . . . . .

. . . . . .

. . . . . .

. . . . . .

. . . . . .

. . . . . .

. . . . . .

. . . . . .

. . . . . .

. . . . . .

. . . . . .

. . . . . .

. . . . . .

. . . . . .

. . . . . .

. . . . . .

. . . . . .

. . . . . .

2 Wave Equations and Anti-Particles 2.1 Particle-wave duality . . . . . . . . . . . . 2.2 The Schr odinger equation . . . . . . . . . 2.3 Intermezzo: four-vector notation . . . . . . 2.4 The Klein-Gordon equation . . . . . . . . 2.5 Interpretation of negative energy solutions 2.5.1 Diracs interpretation . . . . . . . . 2.5.2 Pauli-Weisskopf interpretation . . . 2.5.3 Feynman-St uckelberg interpretation 3 The 3.1 3.2 3.3 3.4 3.5 Electromagnetic Field The Maxwell Equations . . Gauge transformations . . . The photon . . . . . . . . . Electrodynamics in quantum The Aharanov-Bohm Eect

. . . . . . . .

. . . . . . . .

. . . . . . . .

. . . . . . . .

. . . . . . . .

. . . . . . . .

. . . . . . . .

. . . . . . . .

. . . . . . . .

. . . . . . . .

. . . . . . . .

. . . . . . . .

. . . . . . . .

. . . . . . . .

. . . . . . . .

. . . . . . . .

. . . . . . . .

. . . . . . . . . . . . . . . . . . mechanics . . . . . .

. . . . .

. . . . .

. . . . .

. . . . .

. . . . .

. . . . .

. . . . .

. . . . .

. . . . .

. . . . .

. . . . .

. . . . .

. . . . .

. . . . .

. . . . .

. . . . .

. . . . .

. . . . .

. . . . .

4 Perturbation Theory and Fermis Golden Rule 4.1 Decay and scattering observables . . . . . . . . 4.2 Non-relativistic scattering . . . . . . . . . . . . 4.3 Relativistic scattering . . . . . . . . . . . . . . . 4.3.1 Normalisation of the Wave Function . . 4.3.2 The Flux Factor . . . . . . . . . . . . . 4.3.3 The Phase Space Factor . . . . . . . . . i

. . . . . .

. . . . . .

. . . . . .

. . . . . .

. . . . . .

. . . . . .

. . . . . .

. . . . . .

. . . . . .

. . . . . .

. . . . . .

. . . . . .

. . . . . .

. . . . . .

ii 4.3.4

CONTENTS Golden rules for cross-section and decay . . . . . . . . . . . . . . 59 63 63 65 67 70 72 73 77 77 80 81 83 83 85 85 89 89 90 92 93 94 95 96 101 101 104 109 111

5 Electromagnetic Scattering of Spinless Particles 5.1 Electromagnetic current . . . . . . . . . . . . . . 5.2 Coulomb scattering . . . . . . . . . . . . . . . . . 5.3 Spinless K Scattering . . . . . . . . . . . . . 5.4 Feynman calculus: propagators and vertex factors 5.5 Form factors . . . . . . . . . . . . . . . . . . . . . 5.6 Particles and Anti-Particles . . . . . . . . . . . . 6 The 6.1 6.2 6.3 6.4 6.5 6.6 6.7 Dirac Equation Spin, spinors and the gyromagnetic ratio Dirac equation . . . . . . . . . . . . . . Covariant form of the Dirac equation . . Dirac algebra . . . . . . . . . . . . . . . Adjoint spinors and current density . . . Bilinear covariants . . . . . . . . . . . . Charge current and anti-particles . . . .

. . . . . .

. . . . . .

. . . . . .

. . . . . .

. . . . . .

. . . . . .

. . . . . .

. . . . . .

. . . . . .

. . . . . .

. . . . . .

. . . . . .

. . . . . .

. . . . . . .

. . . . . . .

. . . . . . .

. . . . . . .

. . . . . . .

. . . . . . .

. . . . . . .

. . . . . . .

. . . . . . .

. . . . . . .

. . . . . . .

. . . . . . .

. . . . . . .

. . . . . . .

. . . . . . .

. . . . . . .

. . . . . . .

. . . . . . .

7 Solutions of the Dirac Equation 7.1 Plane waves solutions with p = 0 . 7.2 Plane wave solutions for p = 0 . . . 7.3 Helicity . . . . . . . . . . . . . . . 7.4 Antiparticle spinors . . . . . . . . . 7.5 Normalization of the wave function 7.6 The completeness relation . . . . . 7.7 The charge conjugation operation .

. . . . . . .

. . . . . . .

. . . . . . .

. . . . . . .

. . . . . . .

. . . . . . .

. . . . . . .

. . . . . . .

. . . . . . .

. . . . . . .

. . . . . . .

. . . . . . .

. . . . . . .

. . . . . . .

. . . . . . .

. . . . . . .

. . . . . . .

. . . . . . .

. . . . . . .

. . . . . . .

. . . . . . .

8 Spin-1/2 Electrodynamics 8.1 Feynman rules for fermion scattering 8.2 Electron-muon scattering . . . . . . . 8.3 Crossing: the process e e+ + . 8.4 Summary of QED Feynman rules . . 9 The 9.1 9.2 9.3 9.4 9.5 9.6 9.7

. . . .

. . . .

. . . .

. . . .

. . . .

. . . .

. . . .

. . . .

. . . .

. . . .

. . . .

. . . .

. . . .

. . . .

. . . .

. . . .

. . . .

. . . .

. . . .

. . . .

Weak Interaction The 4-point interaction . . . . . . . . . . . . . . . . . . . . . Parity . . . . . . . . . . . . . . . . . . . . . . . . . . . . . . Covariance of the wave equations under parity . . . . . . . . The V A interaction . . . . . . . . . . . . . . . . . . . . . The propagator of the weak interaction . . . . . . . . . . . . Muon decay . . . . . . . . . . . . . . . . . . . . . . . . . . . Quark mixing . . . . . . . . . . . . . . . . . . . . . . . . . . 9.7.1 Cabibbo - GIM mechanism . . . . . . . . . . . . . . . 9.7.2 The Cabibbo - Kobayashi - Maskawa (CKM) matrix

. . . . . . . . .

. . . . . . . . .

. . . . . . . . .

. . . . . . . . .

. . . . . . . . .

. . . . . . . . .

117 . 119 . 121 . 123 . 125 . 125 . 126 . 130 . 131 . 132

CONTENTS 10 Local Gauge Invariance 10.1 Introduction . . . . . . . . . . . . . . . . . . . . . . . . . . . 10.2 The Lagrangian . . . . . . . . . . . . . . . . . . . . . . . . . 10.3 Global phase invariance and Noethers theorem . . . . . . . 10.4 Local phase invariance . . . . . . . . . . . . . . . . . . . . . 10.5 Application to the Lagrangian for a Dirac eld . . . . . . . . 10.6 Yang-Mills theory . . . . . . . . . . . . . . . . . . . . . . . . 10.7 Historical interlude 1: isospin, QCD and weak isospin . . . . 10.8 Historical interlude 2: the origin of the name gauge theory 11 Electroweak Theory 11.1 SU(2) symmetry for left-handed douplets 11.2 The Charged Current . . . . . . . . . . . 11.3 The Neutral Current . . . . . . . . . . . 11.4 Couplings for Z f f . . . . . . . . . . 11.5 The mass of the W and Z bosons . . . .

iii 137 . 137 . 138 . 140 . 141 . 142 . 143 . 147 . 148 153 . 154 . 156 . 158 . 161 . 162 165 . 165 . 166 . 167 . 169 . 171 . 171 . 173 . 174 . 175 . 175 179 183

. . . . . . . .

. . . . . . . .

. . . . . . . .

. . . . . . . .

. . . . . . . .

. . . . . . . .

. . . . .

. . . . .

. . . . .

. . . . .

. . . . .

. . . . .

. . . . .

. . . . .

. . . . .

. . . . .

. . . . .

. . . . .

. . . . .

. . . . .

. . . . .

. . . . .

. . . . .

12 The Process e e+ + 12.1 Helicity conservation . . . . . . . . . . . . . . . 12.2 The cross section of e e+ + . . . . . . . 12.2.1 Photon contribution . . . . . . . . . . . 12.2.2 Z 0 contribution . . . . . . . . . . . . . . 12.2.3 Correcting for the nite width of the Z 0 12.2.4 Total unpolarized cross-section . . . . . 12.2.5 Near the resonance . . . . . . . . . . . . 12.3 The Z 0 decay widths . . . . . . . . . . . . . . . 12.4 Forward-backward asymmetry . . . . . . . . . . 12.5 The number of light neutrinos . . . . . . . . . . A Summary of electroweak theory B Some properties of Dirac matrices i and

. . . . . . . . . .

. . . . . . . . . .

. . . . . . . . . .

. . . . . . . . . .

. . . . . . . . . .

. . . . . . . . . .

. . . . . . . . . .

. . . . . . . . . .

. . . . . . . . . .

. . . . . . . . . .

. . . . . . . . . .

. . . . . . . . . .

. . . . . . . . . .

Preliminaries

These are the lecture notes for the Particle Physics 1 (PP1) master course of that is taught at Nikhef in the autumn semester of 2013. The notes mainly follows the material as discussed in the books of Halzen and Martin and Griths. They have been written by Marcel Merk in the period 2000-2011. Some parts have been edited by Wouter Hulsbergen for the master courses of 2012 and 2013. The notes can be used by the students but should not be distributed. The original material is found in the books used to prepare the lectures (see below). The contents of particle physics 1 is the following: Lecture 1: Concepts and History Lecture 2 - 5: Electrodynamics of spinless particles Lecture 6 - 8: Electrodynamics of spin 1/2 particles Lecture 9: The Weak interaction Lecture 10 - 12: Electroweak scattering: The Standard Model Each lecture of 2 45 minutes is followed by a 1 hour problem solving session. The particle physics 2 course contains the following topics: The Higgs Mechanism Quantum Chromodynamics This course will be given by Michiel Botje en Ivo van Vulpen. In addition the master oers in the next semester topical courses (not obligatory) on the particle physics subjects: CP Violation, Neutrino Physics and Physics Beyond the Standard Model i

ii

PRELIMINARIES

Examination
The examination consists of two parts: Homework (weight=1/3) and an Exam (weight=2/3).

Literature
The following literature is used in the preparation of this course (the comments reect my personal opinion): Halzen & Martin: Quarks & Leptons: an Introductory Course in Modern Particle Physics :
Although it is somewhat out of date (1984), I consider it to be the best book in the eld for a master course. It is somewhat of a theoretical nature. It builds on the earlier work of Aitchison (see below). Most of the course follows this book.

Griths: Introduction to Elementary Particle Physics, second, revised ed.


The text is somewhat easier to read than H & M and is more up-to-date (2008) (e.g. neutrino oscillations) but on the other hand has a somewhat less robust treatment in deriving the equations.

Perkins: Introduction to High Energy Physics, (1987) 3-rd ed., (2000) 4-th ed.
The rst three editions were a standard text for all experimental particle physics. It is dated, but gives an excellent description of, in particular, the experiments. The fourth edition is updated with more modern results, while some older material is omitted.

Aitchison: Relativistic Quantum Mechanics


(1972) A classical, very good, but old book, often referred to by H & M.

Aitchison & Hey: Gauge Theories in Particle Physics


(1982) 2nd edition: An updated version of the book of Aitchison; a bit more theoretical. (2003) 3rd edition (2 volumes): major rewrite in two volumes; very good but even more theoretical. It includes an introduction to quantum eld theory.

Burcham & Jobes: Nuclear & Particle Physics


(1995) An extensive text on nuclear physics and particle physics. It contains more (modern) material than H & M. Formulas are explained rather than derived and more text is spent to explain concepts.

Das & Ferbel: Introduction to Nuclear and Particle Physics


(2006) A book that is half on experimental techniques and half on theory. It is more suitable for a bachelor level course and does not contain a treatment of scattering theory for particles with spin.

Martin and Shaw: Particle Physics , 2-nd ed.


(1997) A textbook that is somewhere inbetween Perkins and Das & Ferbel. In my opinion it has the level inbetween bachelor and master.

iii Particle Data Group: Review of Particle Physics


This book appears every two years in two versions: the book and the booklet. Both of them list all aspects of the known particles and forces. The book also contains concise, but excellent short reviews of theories, experiments, accellerators, analysis techniques, statistics etc. There is also a version on the web: http://pdg.lbl.gov

The Internet: In particular Wikipedia contains a lot of information. However, one should note that Wikipedia does not contain original articles and they are certainly not reviewed! This means that they cannot be used for formal citations. In addition, have a look at google books, where (parts of) books are online available.

iv

PRELIMINARIES

About Nikhef
Nikhef is the Dutch institute for subatomic physics. (The name was originally an acronym for Nationaal Instituut voor Kern en Hoge Energie Fysica.) The name Nikhef is used to indicate simultaneously two overlapping organisations: Nikhef is a national research lab (institute) funded by the foundation FOM; the dutch foundation for fundamental research of matter. Nikhef is also a collaboration between the Nikhef institute and the particle physics departements of the UvA (Adam), the VU (Adam), the UU (Utrecht) and the RU (Nijmegen) contribute. In this collaboration all dutch activities in particle physics are coordinated. In addition there is a collaboration between Nikhef and the Rijksuniversiteit Groningen (the former FOM nuclear physics institute KVI) and there are contacts with the Universities of Twente, Leiden and Eindhoven. For more information see the Nikhef web page: http://www.nikhef.nl. The research at Nikhef includes both accelerator based particle physics and astro-particle physics. A strategic plan, describing the research programmes at Nikhef can be found on the web, from: www.nikhef.nl/leadmin/Doc/Docs & pdf/StrategicPlan.pdf . The accelerator physics research of Nikhef is currently focusing on the LHC experiments: Alice (Quark gluon plasma), Atlas (Higgs) and LHCb (CP violation). Each of these experiments search answers for open issues in particle physics (the state of matter at high temperature, the origin of mass, the mechanism behind missing antimatter) and hope to discover new phenomena (eg supersymmetry, extra dimensions). The LHC has started taking data in 2009 and the rst LHC physics run has ocially ended in winter 2013. In a joint meeting in summer 2012 the ATLAS and CMS experiments presented their biggest discovery so far, a new particle, called the Higgs particle, with a mass of approximately 125 GeV. The existence of this particle was a prediction of the Standard Model. Further research concentrates on signs of physics beyond the Standard Model, sometimes called New Physics. In preparation of these LHC experiments Nikhef has also been active at other labs: STAR (Brookhaven), D0 (Fermilab) and Babar (SLAC). Previous experiments that ended their activities are: L3 and Delphi at LEP, and Zeus, Hermes and HERA-B at Desy. A more recent development is the research eld of astroparticle physics. It includes Antares & KM3NeT (cosmic neutrino sources), Pierre Auger (high energy cosmic rays), Virgo & ET (gravitational waves) and Xenon (dark matter). Nikhef houses a theory departement with research on quantum eld theory and gravity, string theory, QCD (perturbative and lattice) and B-physics.

v Driven by the massive computing challenge of the LHC, Nikhef also has a scientic computing departement: the Physics Data Processing group. They are active in the development of a worldwide computing network to analyze the large datastreams from the (LHC-) experiments (The Grid). Nikhef program leaders/contact persons: Name Nikhef director Frank Linde Theory Eric Laenen Atlas Paul de Jong B-physics Marcel Merk Alice Thomas Peitzmann Antares Aart Heijboer Pierre Auger Charles Timmermans Virgo and ET Jo van den Brand Xenon Patrick Decowski Detector R&D Frank Linde Scientic Computing Je Templon oce H232 H323 H253 N243 N325 H342 N247 H349 H232 H158 phone 5001 5127 2087 5107 5050 5116 2015 2145 5001 2092 email z66@nikhef.nl t45@nikhef.nl p.d.jong@nikhef.nl marcel.merk@nikhef.nl t.peitzmann@uu.nl a.heijboer@nikhef.nl c.timmermans@hef.ru.nl jo@nikhef.nl p.decowski@nikhef.nl z66@nikhef.nl templon@nikhef.nl

vi

PRELIMINARIES

A very brief history of particle physics


The book of Griths starts with a nice historical overview of particle physics in the previous century. This is a summary of key events: Atomic Models 1897 Thomson: Discovery of Electron. The atom contains electrons as plums in a pudding. 1911 Rutherford: The atom mainly consists of empty space with a hard and heavy, positively charged nucleus. 1913 Bohr: First quantum model of the atom in which electrons circled in stable orbits, quatized as: L = n 1932 Chadwick: Discovery of the neutron. The atomic nucleus contains both protons and neutrons. The role of the neutrons is associated with the binding force between the positively charged protons. The Photon 1900 Planck: Description blackbody spectrum with quantized radiation. No interpretation. 1905 Einstein: Realization that electromagnetic radiation itself is fundamentally quantized, explaining the photoelectric eect. His theory received scepticism. 1916 Millikan: Measurement of the photo electric eect agrees with Einsteins theory. 1923 Compton: Scattering of photons on particles conrmed corpuscular character of light: the Compton wavelength. Mesons 1934 Yukawa: Nuclear binding potential described with the exchange of a quantized eld: the pi-meson or pion. 1937 Anderson & Neddermeyer: Search for the pion in cosmic rays but he nds a weakly interacting particle: the muon. (Rabi: Who ordered that?) 1947 Powell: Finds both the pion and the muon in an analysis of cosmic radiation with photo emulsions. Anti matter 1927 Dirac interprets negative energy solutions of Klein Gordon equation as energy levels of holes in an innite electron sea: positron. 1931 Anderson observes the positron.

vii 1940-1950 Feynman and St uckelberg interpret negative energy solutions as the positive energy of the anti-particle: QED. Neutrinos 1930 Pauli and Fermi propose neutrinos to be produced in -decay (m = 0). 1958 Cowan and Reines observe inverse beta decay. 1962 Lederman and Schwarz showed that e = . Conservation of lepton number. Strangeness 1947 Rochester and Butler observe V 0 events: K 0 meson. 1950 Anderson observes V 0 events: baryon. The Eightfold Way 1961 Gell-Mann makes particle multiplets and predicts the . 1964 particle found. The Quark Model 1964 Gell-Mann and Zweig postulate the existance of quarks 1968 Discovery of quarks in electron-proton collisions (SLAC). 1974 Discovery charm quark (J/ ) in SLAC & Brookhaven. 1977 Discovery bottom quarks ( ) in Fermilab. 1979 Discovery of the gluon in 3-jet events (Desy). 1995 Discovery of top quark (Fermilab). Broken Symmetry 1956 Lee and Yang postulate parity violation in weak interaction. 1957 Wu et. al. observe parity violation in beta decay. 1964 Christenson, Cronin, Fitch & Turlay observe CP violation in neutral K meson decays. The Standard Model 1978 Glashow, Weinberg, Salam formulate Standard Model for electroweak interactions 1983 W-boson has been found at CERN. 1984 Z-boson has been found at CERN. 1989-2000 LEP collider has veried Standard Model to high precision.

viii

PRELIMINARIES

Lecture 1 Particles and Forces


After Chadwick had discovered the neutron in 1932, the elementary constituents of matter were the proton and the neutron inside the atomic nucleus, and the electron circling around it. The force responsible for interactions between charged particles was the electromagnetic force. Moving charges emitted electromagnetic waves, which happened to be quantized in energy and were called photons. With these constituents the atomic elements could be described, as well as their chemistry. The answer to the question: What is the world made of? was indeed rather simple. However, there were already some signs that there were more fundamental particles than just protons, neutrons, electrons and photons: Dirac had postulated in 1927 the existence of anti-matter as a consequence of his relativistic version of the Schr odinger equation in quantum mechanics. (We will come back to the Dirac theory later on.) The anti-matter partner of the electron, the positron, was actually discovered in 1932 by Anderson (see Fig. 1.1). Pauli had postulated the existence of an invisible particle that was produced in nuclear beta decay: the neutrino. In a nuclear beta decay process: NA NB + e the energy of the emitted electron is determined by the mass dierence of the nuclei NA and NB . It was observed that the kinetic energy of the electrons, however, showed a broad mass spectrum (see Fig. 1.2), of which the maximum was equal to the expected kinetic energy. It was as if an additional invisible particle of low mass is produced in the same process: the (anti-) neutrino.

1.1

The Yukawa Potential

Though the constituents of atoms were fairly well established, there was something puzzling about atoms: What was keeping the nucleus together? It clearly had to be 1

LECTURE 1. PARTICLES AND FORCES

Figure 1.1: The discovery of the positron as reported by Anderson in 1932. Knowing the direction of the B eld Anderson deduced that the trace was originating from an anti electron. Question: how?

a new force, something beyond electromagnetism. Rutherfords scattering experiments had given an estimate of the size of the nucleus, of about 1 fm. With protons packed this close, the new force had to be very strong to overcome the repulsive coulomb interaction of the protons. (Being imaginative, physicists simply called it the strong nuclear force.) Yet, to explain scattering experiments, the range of the force had to be small, bound just to the nucleus itself. In an attempt to solve this problem Japanese physicist Yukawa published in 1935 a fundamentally new view of interactions. His idea was that forces, like the electromagnetic force and the nuclear force, could be described by the exchange of virtual particles, as illustrated in Fig. 1.3. These particles (or rather, their eld ) would follow a relativistic wave-equation, just like the electromagnetic eld. In this picture, the massless photon was the carrier of the electromagnetic eld. As we will see in exercise 1.6 the relativistic wave equation for a massless particle leads to an electrostatic potential of the form 1 (1.1) r where e is the fundamental unit of charge, also called the electromagnetic coupling constant. Because of its 1/r dependence, the force is said to be of innite range. V (r) = e2 By contrast, in Yukawas proposal the strong force were to be carried by a massive particle, later called the pion. A massive force carrier leads to a potential of the form U (r) = g 2 er/R r (1.2)

1.1. THE YUKAWA POTENTIAL

1.0

Relative Decay Probability

0.8

0.00012

0.6

0.00008 Mass = 0 0.00004

0.4 0 0.2

Mass = 30 eV

18.45

18.50

18.55

18.60

0 2 6

10 14 Energy (keV)

18

Figure 1. The Beta Decay Spectrum for Molecular Tritium


The plot on the left shows the probability that the emerging electron has a particular energy. If the electron were neutral, the spectrum would peak at higher energy and would be centered roughly on that peak. But because the electron is negatively charged, the positively charged nucleus exerts a drag on it, pulling the peak to a lower energy and generating a lopsided spectrum. A close-up of the endpoint (plot on the right) shows the subtle difference between the expected spectra for a massless neutrino and for a neutrino with a mass of 30 electron volts.

Figure 1.2: The beta spectrum as observed in tritium decay to helium. The endpoint of the spectrum can be used to set a limit of the neutrino mass. Question: how?

which is called the One Pion Exchange Potential. Since it falls of exponentially, it has a nite range. The range R is inversely proportional to the mass of the force carrier and for a massless carrier the expression reduces to that for the electrostatic potential. In the exercise you will derive the relation between mass and range properly, but it can also be obtained with a heuristic argument, following the Heisenberg uncertainty principle. (As you can read in Griths, whenever a physicists refers to the uncertainty principle to explain something, take all results with a grain of salt.) In some interpretation, the principle states that we can borrow the energy E = mc2 to create a virtual particle from the vacuum, as long as we give it back within a time t /E . With the particle traveling at the speed of light, this leads to a range R = ct = c /mc2 . From the size of the nucleus, Yukawa estimated the mass of the force carrier to be approximately 100 MeV/c2 . He called the particle a meson, since its mass was somewhere in between the mass of the electron and the nucleon. In 1937 Anderson and Neddermeyer, as well as Street and Stevenson, found that cosmic rays indeed consist of such a middle weight particle. However, in the years after, it became clear that this particle could not really be Yukawas meson, since it did not interact strongly, which was very strange for a carrier of the strong force. In fact this particle turned out to be the muon, the heavier brother of the electron. Only in 1947 Powell (as well as Perkins) found Yukawas pion in cosmic rays. They took their photographic emulsions to mountain tops to study the contents of cosmic

LECTURE 1. PARTICLES AND FORCES

Figure 1.3: Illustration of the interaction between protons and neutrons by charged pion exchange. (From Aichison and Hey.)

rays (see Fig. 1.4). (In a cosmic ray event a cosmic proton scatters with high energy on an atmospheric nucleon and produces many secondary particles.) Pions produced in the atmosphere decay long before they reach sea level, which is why they had not been observed before. As a carrier of the strong force Yukawas meson did not stand the test of time. We now know that the pion is a composite particle and that the true carrier for the strong force is the massless gluon. The range of the strong force is small, not because the force carrier is massive, but because gluons carry a strong interaction charge themselves. However, even if Yukawas original meson model did not survive, his interpretation of forces as the exchange of virtual particles is still central to the description of particle interactions in quantum eld theory.

1.2

Strange Particles

After the pion had been identied as Yukawas strong force carrier and the anti-electron was observed to conrm Diracs theory, things seemed reasonably well under control. The muon was a bit of a mystery. It lead to a famous quote of Isidore Rabi at the conference: Who ordered that ? But in December 1947 things became a lot more complicated when Rochester and Butler observed so-called V 0 events in cloud chamber photographs. It turned out that cosmic particles hitting a lead larget plate could produce many dierent types of new particles. Those new particles were classied as: baryons: particles whose decay products ultimately include a proton; mesons: particles whose decay products ultimately include only leptons or photons.

1.2. STRANGE PARTICLES

Figure 1.4: A pion entering from the left decays into a muon and an invisible neutrino.

LECTURE 1. PARTICLES AND FORCES

These events were strange because of an apparent mismatch between their production cross-section and their decay time. The observed yield indicated that the cross-section for these events was large, 1027 cm2 , typical for a strong nuclear interaction process. On the other hand, the long lifetime of these particles was typical for a weak interaction decay. So, if these were strongly interacting particles, why did not they decay faster? The explanation to this puzzle was given by Abraham Pais in 1952 and is called associated production. Pais suggested that these particles carried a new additive quantum number, called strangeness, that is conserved in strong interactions but not in weak interactions. The production of particles with non-zero strangeness via the strong interaction can only occur in pairs: a particle with strangeness S = +1 is always produced together with a particle with strangeness is S = 1 such that total strangeness is conserved. Such particles with non-zero strangeness would be stable if not for the weak interaction: each of the created particles decays through the weak interaction, leading to a large lifetime. An example of an associated production event is seen in Fig. 1.5. In the years 1950 - 1960 many elementary particles were discovered and one started to speak of the particle zoo. A quote: The nder of a new particle used to be awarded the Nobel prize, but such a discovery now ought to be punished by a $10.000 ne.

1.3

The Eightfold Way

In the early 60s Murray Gell-Mann (at the same time also Yuval Neeman) observed patterns of symmetry in the discovered mesons and baryons. He plotted the spin 1/2 baryons in a so-called octet (the eightfold way after the eightfold way to Nirvana in Buddhism). The octet of the lightest baryons and mesons is displayed in Fig. 1.6 and Fig. 1.7. In these graphs the strangeness quantum number is plotted vertically. Also heavier hadrons could be given a place in multiplets. The baryons with spin=3/2 were seen to form a decuplet, see Fig. 1.8. The particle at the bottom (at S=-3) had not been observed. Not only was it found later on, but also its predicted mass was found to be correct! The discovery of the particle is shown in Fig. 1.9.

1.4

The Quark Model

The observed structure of hadrons in multiplets hinted at an underlying structure. GellMann and Zweig postulated indeed that hadrons consist of more fundamental partons: the quarks. Initially three quarks and their anti-particle were assumed to exist (see Fig. 1.10). A baryon consists of 3 quarks: (q, q, q ), while a meson consists of a quark and an antiquark: (q, q ). Mesons can be their own anti-particle, baryons cannot. How does this explain that baryons and mesons appear in the form of octets, decuplets, nonets etc.? For example a baryon, consisting of 3 quarks with 3 avours (u, d, s),

1.4. THE QUARK MODEL

Figure 1.5: A bubble chamber picture of associated production.

S=0 S=1 S=2

n 0 Q=1

p+
0

Q=0

Q=+1

Figure 1.6: Octet of lightest baryons with spin=1/2.

LECTURE 1. PARTICLES AND FORCES

S=1 S=0 S=1

0 Q=0 Q=1

Q=1

Figure 1.7: Octet with lightest mesons of spin=0

S=0 S=1 S=2 S=3

+ +

++

mass ~1230 MeV

Q=+2 Q=+1

~1380 MeV ~1530 MeV ~1680 MeV

Q=0 Q=1

Figure 1.8: Decuplet of baryons with spin=3/2. The was not yet observed when this model was introduced. Its mass was predicted.

1.4. THE QUARK MODEL

Figure 1.9: Discovery of the omega particle.

S=0 S=1

d s

u Q=+2/3 Q=1/3

S=+1 S=0 u

s d Q=2/3 Q=+1/3

Figure 1.10: The fundamental quarks: u,d,s.

10

LECTURE 1. PARTICLES AND FORCES

could in principle lead to 3x3x3=27 combinations. The answer lies in the fact that the wave function of fermions is subject to a symmetry under exchange of fermions. Since baryons are fermions (half-integer spin) the total wave function must be anti-symmetric with respect to the interchange of two quarks. The multiplets are grouped by the symmetry of the quark wave functions, as illustrated in Fig. 1.11. The assumption that u, d and s quarks are interchangeable leads to a symmetry called SU(3) and the multiplets represent dierent representations of that group, identied by their total spin. (In the exercises you will work out an example yourself.) In group theory language the decomposition is written as 3 3 3 = 10 8 8 1 for the baryons and as 33=81 for the mesons. For more information on the static quark model read 2.10 and 2.11 in H&M, 5.5 and 5.6 in Griths, or chapter 5 in the book of Perkins.

Figure 1.11: Quark wave functions of the baryon multiplets. (From Griths.)

The baryon wave function can be schematically written as (baryon) = (space) (spin) (f lavour)

1.4. THE QUARK MODEL

11

On closer inspection (not detailed here) this leads to a problem for the states with three equal quarks in the decouplet, like the +++ : their wave function would not be antisymmetric. A solution is to introduce yet another SU(3) symmetry for quarks: each quark actually comes in three dierent congurations, labeled by the quantum number colour. The colour quantum number takes values red (r), green (g ) or blue (b) for quarks and anti-red ( r), anti-green ( g ) or anti-blue ( b) for anti-quarks. Of course, in the new SU(3) symmetry we would also expect a division in a multiplets. However, for a reason that is still not entirely understood all naturally occurring hadrons are in the colour singlet state only. For example, the colour wave function for baryons is given by (colour) = (rgb rbg + gbr grb + brg bgr)/ 6 (1.3)

This is anti-symmetric in the exchange of two quarks, which solves the problem for the wave-functions of qqq baryons. Likewise, mesons occur only in a colour singlet superposition of rr , b b and g g . Of course, when the idea of colour was introduced, people saw it just as a somewhat articial way to solve a problem in a model that was anyway not based on very solid footing. However, besides the theoretical arguments, there is also direct experimental for this hidden degree of freedom of quarks. Consider the ratio of the cross-section of e+ e to hadrons and muons: R (e+ e hadrons) = NC (e+ e + ) Q2 i
i

(1.4)

Since the initial state is a lepton pair, for energies well below the Z boson this crosssection is fully dominated by the electromagnetic interaction. Knowing the electric charge of quarks and leptons, we know the coupling strength and we can predict this ratio given the number of quarks. Only quarks with a mass less than half the centre-of momentum energy ( s) can be created. Therefore, we expect R to depend on s and jump with exactly the charge Qi of the quark when the energy passes the threshold to produce that quark. On the other hand, if there are actually identical quarks with the same mass, then we expect the jump to be higher. From the result, shown in Fig. 1.12, one can extract that NC = 3. We now identify the SU(3)-colour symmetry with the gauge symmetry of the strong interaction. Colour is the charge of the strong interaction, just like electric charge is the charge of the electromagnetic interaction. The carrier of the strong interaction is the massless gluon. Unlike the carrier of the electromagnetic interaction, which is the electrically neutral photon, the gluon carries itself colour charge. It is believed that as a result of the gluon self-coupling quarks are conned in colourless objects. This property of the strong interaction is called connement. It implies that one can never observe a free quark or gluon!

12

LECTURE 1. PARTICLES AND FORCES

10
3

J/

(2S )

10

R
10

1
-1

10 1

10

10

s [GeV]

Figure 1.12: The ratio R as a function of

s.

1.5

The Standard Model

1 fermions In the Standard Model (SM) of particle physics all matter particles are spin- 2 and all force carriers are spin-1 bosons. The fermions are the quarks and leptons, organized in three families:

charge Quarks u (up) c (charm) t (top) 2 3 1.54 MeV 1.151.35 GeV (174.3 5.1) GeV d (down) s (strange) b (bottom) 1 3 48 MeV 80130 MeV 4.14.4 GeV charge Leptons e (e neutrino) ( neutrino) ( neutrino) 0 < 3 eV < 0.19 MeV < 18.2 MeV e (electron) (muon) (tau) 1 0.511 MeV 106 MeV 1.78 GeV The force carriers are the photon, the Z and W and the gluons: Force Boson Strong g (8 gluons) Electromagnetic (photon) Weak Z 0 ,W (weak bosons) Relative strength s O(1) O(102 ) W O(106 )

In the SM forces originate from a mechanism called local gauge invariance, which will be discussed later on in the course. The strong force (or colour force) is mediated by

1.5. THE STANDARD MODEL

13

gluons, the weak force by intermediate vector bosons, and the electromagnetic force by photons. Only the charged weak interaction can change the avour of quarks and leptons: it allows for transitions between an up-type quark and a down-type quark, and between charged leptons and neutrinos. Some of the fundamental diagrams are represented below: e a: e
+

e b: W e

q g q

c:

Figure 1.13: Feynman diagrams of fundamental lowest order perturbation theory processes in a: electromagnetic, b: weak and c: strong interaction.

As also indicated above, there is an important dierence between the electromagnetic force on one hand, and the weak and strong force on the other hand. The photon does not carry charge and, therefore, does not interact with itself. The gluons, however, carry colour and do interact amongst each other. Also, the weak vector bosons carry weak isospin and undergo a self-coupling. The strength of an interaction is determined by the coupling constant as well as the mass of the vector boson. Contrary to its name the couplings are not constant, but vary as a function of energy. At a momentum transfer of 1015 GeV the couplings of electromagnetic, weak and strong interaction all obtain approximately the same value. In the quest of unication it is often assumed that the three forces unify to a single force (i.e. a single gauge symmetry group) at this energy. Due to the self-coupling of the force carriers the running of the coupling constants of the weak and strong interaction are opposite to that of electromagnetism. Electromagnetism becomes weaker at low momentum (i.e. at large distance), the weak and the strong force become stronger at low momentum or large distance. The strong interaction coupling even diverges at momenta less than a few 100 MeV (the perturbative QCD description breaks down), leading to connement. Finally, the Standard Model includes a scalar boson eld, the Higgs eld, which provides mass to the vector bosons and fermions in the Brout-Englert-Higgs mechanism. A new particle consistent with the Higgs particle was discovered in summer 2012 by the ATLAS and CMS collaborations. Despite the success of the standard model in describing all physics at low energy scale, there are still many open questions, such as Why are the masses of the particles what they are? Why are there 3 generations of fermions? Are quarks and leptons truly fundamental?

14

LECTURE 1. PARTICLES AND FORCES

Figure 1.14: Running of the coupling constants and possible unication point. On the left: Standard Model. On the right: Supersymmetric Standard Model.

Is there really only one Higgs particle? Why is the charge of the electron exactly opposite to that of the proton? Or phrased dierently: why is the total charge of leptons and quarks in one generation exactly zero? Is a neutrino its own anti-particle? Can all forces be described by a single gauge symmetry (unication)? Why is there no anti matter in the universe? What is the source of dark matter? What is the source of dark energy? Particle physicists try to address these questions both with scattering experiments in the laboratory and by studying high energy phenomena in the cosmos.

1.6

Units in particle physics

In particle physics we often make use of natural units to simplify expressions. In this system of units the action is expressed in units of Plancks constant 1.055 1034 Js and velocity is expressed in units of the speed of light in vacuum c = 2.998 108 m/s. (1.6) (1.5)

such that all factors and c can be omitted. As a consequence (see textbooks), there is only one basic unit for length (L), time (T), mass (M), energy and momentum. In high energy physics this basic unit is often chosen to be the energy in MeV or GeV, where

1.6. UNITS IN PARTICLE PHYSICS

15

1 eV is the kinetic energy an electron obtains when it is accelerated over a voltage of 1V. Momentum and mass then get units of energy, while length and time get units of inverse energy. To confront the result of calculation with experiments the factors and c usually need to be reintroduced. There are two ways to do this. First one can take the nal expressions in natural units and then use the table 1.1 to convert the quantities for space, time, mass, energy and momentum back to their original counterparts. (For the positron charge, see below.) quantity space time mass momentum energy positron charge symbol in natural units x t m p E e equivalent symbol in ordinary units x/ c t/ mc2 pc E e c/ 0

Table 1.1: Conversion of basic quantities between natural and ordinary units.

Alternatively, one can express all results in GeV, then use the following table with conversion factors to translate it into SI units: quantity mass length time conversion factor 1 kg = 5.61 1026 GeV 1 m = 5.07 1015 GeV1 1 s = 1.52 1024 GeV1 natural unit GeV GeV1 GeV1 normal unit GeV/c2 c/ GeV / GeV

Table 1.2: Conversion factors from natural units to ordinary units.

Where it concerns electromagnetic interactions, there is also freedom in choosing the unit of electric charge. The electrostatic force between two electrons in vacuum is given by Coulombs law e2 1 F = (1.7) 4 0 r2 where 0 is the vacuum permittivity. The dimension of the factor e2 / 0 is xed it is [L3 M/T2 ] but this still leaves a choice of what to put in the charges and what in the vacuum permittivity. In the SI system the unit of charge is the Coulomb. (It is currently dened via the Amp ere, which in turn is dened as the current leading to a particular force between two current-carrying wires. In the near future, this denition will probably be replaced by the charge corresponding to a xed number of particles with the positron charge.) The positron charge expressed in Coulombs is about e 1.6023 1019 C (1.8)

16 while the vacuum permittivity is


0

LECTURE 1. PARTICLES AND FORCES

8.854 1012 C2 s2 kg1 m3 .

(1.9)

As we shall see in Lecture 3 the Maxwell equations look much more neat if, in addition to c = 1, we choose 0 = 1. This is called the Heaviside-Lorentz system. Obviously, this choice aects the numerical value of e. However, note that the factor = e2 4 0 c (1.10)

is dimensionless. This parameter is called the ne structure constant. Its value is approximately 1/137 and independent of the system of units. It is because of the fact that 1 that perturbation theory works so well in quantum electrodynamics. Finally, it is customary to express cross sections in barn, which is equal to 1024 cm2 .

Glossary
hadron (greek: strong) lepton (greek: light, weak) baryon (greek: heavy) meson (greek: middle) pentaquark fermion boson gauge-boson particle that feels the strong interaction particle that feels only EM and weak interaction particle consisting of three quarks particle consisting of a quark and an anti-quark a hypothetical particle consisting of 4 quarks and an anti-quark half-integer spin particle integer spin particle force carrier as predicted from local gauge invariance

1.6. UNITS IN PARTICLE PHYSICS

17

Exercises
Exercise 1.1 (Conversion factors) Derive the conversion factors for mass, length and time in table 1.2. Exercise 1.2 (Kinematics of Z production) The Z-boson has a mass of 91.1 GeV. It can be produced by annihilation of an electron and a positron. The mass of an electron, as well as that of a positron, is 0.511 MeV. (a) Draw the (dominant) Feynman diagram for this process. (b) Assume that an electron and a positron are accelerated in opposite directions and collide head-on to produce a Z-boson in the lab frame. Calculate the beam energy required for the electron and the positron to produce a Z-boson. (c) Assume that a beam of positron particles is shot on a target containing electrons. Calculate the beam energy required for the positron beam to produce Z-bosons. (d) This experiment was carried out in the 1990s. Which method (b or c) do you think was used? Why? Exercise 1.3 (The Yukawa potential) (a) The wave equation for an electromagnetic wave in vacuum is given by: 2V =0 ; 2 2 2 t2

which in the static case can be written in the form of Laplace equation: 2 V = 0 Now consider a point charge in vacuum. Exploiting spherical symmetry, show that this equation leads to a potential V (r) 1/r. Hint: look up the expression for the Laplace operator in spherical coordinates. (b) The wave equation for a massive eld is the Klein Gordon equation: 2 U + m2 U = 0 which, again in the static case can be written in the form: 2 U m2 U = 0 Show, again assuming spherical symmetry, that Yukawas potential is a solution of the equation for a massive force carrier. What is the relation between the mass m of the force carrier and the range R of the force? (c) Estimate the mass of the -meson assuming that the range of the nucleon force is 1.5 1015 m = 1.5 fm.

18

LECTURE 1. PARTICLES AND FORCES

Exercise 1.4 (Quark content of hadrons) (a.) Assign the quark contents to the particles in the baryon decuplet (Fig. 1.8) and the meson octet (Fig. 1.7). (b.) Using the concept of strangeness conservation, explain why the threshold energy (for incident on stationary protons) for + p K 0 + anything is less than for + p K + anything assuming that both processes proceed through the strong interaction. (From Aichison and Hey, chapter 1.) Exercise 1.5 (The Quark Model) (a) Quarks are fermions with spin 1/2. Show that the spin of a meson (2 quarks) can be either a triplet of spin 1 or a singlet of spin 0. Hint: Remember the Clebsch Gordon coecients in adding quantum numbers. In group theory this is often represented as the product of two doublets leads to the sum of a triplet and a singlet: 2 2 = 3 1 or, in terms of quantum numbers: 1/2 1/2 = 1 0. (b) Show that for baryon spin states we can write: 1/2 1/2 1/2 = 3/2 1/2 1/2 or equivalently 2 2 2 = 4 2 2 (c) Let us restrict ourselves to two quark avours: u and d. We introduce a new quantum number, called isospin in complete analogy with spin, and we refer to the u quark as the isospin +1/2 component and the d quark to the isospin -1/2 component (or u= isospin up and d=isospin down). What are the possible isospin values for the resulting baryon? (d) The ++ particle is in the lowest angular momentum state (L = 0) and has spin J3 = 3/2 and isospin I3 = 3/2. The overall wavefunction (Lspace-part, Sspin-part, Iisospin-part) must be anti-symmetric under exchange of any of the quarks. The symmetry of the space, spin and isospin part has a consequence for the required symmetry of the Colour part of the wave function. Write down the colour part of the wave-function taking into account that the particle is colour neutral. (e) In the case that we include the s quark the avour part of the wave function becomes: 3 3 3 = 10 8 8 1. In the case that we include all 6 quarks it becomes: 6 6 6. However, this is not a good symmetry. Why not?
0

Lecture 2 Wave Equations and Anti-Particles


As briey discussed in Lecture 1, most of our knowledge of the physics of elementary particles comes from scattering experiments, from decays and from the spectroscopy of bound states. In this course we will only consider the electroweak theory, leaving quantum chromodynamics, the theory of the strong interaction, to the Particle Physics 2 course. The theory of bound states in electrodynamics is essentially the hydrogen atom, which (apart from a few subtle phenomena) is well described by non-relativistic quantum mechanics (QM). We do not discuss the hydrogen atom: instead we concentrate on processes at high energies, which requires an extension to relativistic velocities. The theoretical framework that allows this is called quantum eld theory (QFT). The quantum eld theory for electrodynamics is called Quantum Electrodynamics (QED). To understand when classical mechanics breaks down it is useful to look at a few typical distance scales of electromagnetic interactions in the quantum world. The rst distance scale is the Bohr radius, the distance at which an electron circles around an innitely heavy object (a proton) of opposite charge. Using just classical mechanics and imposing quantization of angular momentum by requiring that rp = , you will nd (try!) that this distance is given by . (2.1) me c (A proper treatment in QM tells you that the expectation value for the radius is not exactly the Bohr radius, but it comes close.) Hence, the velocity of the electron is rBohr = vBohr = p/m = c which indeed makes the electron in the hydrogen atom notably non-relativistic. The second distance scale is the Compton wavelength of the electron. Suppose that you study electrons by shooting photons at zero velocity electrons. The smaller the wavelength of the photon, the more precise you look. However, at some point the energy of the photons becomes large enough that you can create a new electron. (In 19 (2.2)

20

LECTURE 2. WAVE EQUATIONS AND ANTI-PARTICLES

our real theory, you can only create pairs, but that factor 2 is not important now.) The energy at which this happens is when = me c2 , or at a wavelength e = 2 me c (2.3)

Usually, we divide both sides by 2 and call this the reduced Compton wavelength, just like is usually called the reduced Plancks constant. In electromagnetic collisions at this energy, quantum mechanics no longer suces: as soon as collisions involve the creation of new particles, one needs QFT. Finally, consider the collisions of two electrons at even higher energy. If the electrons get close enough, the Coulomb energy is sucient to create a new electron. (Again, ignore the factor two required for pair production.) Expressing the Coulomb potential as V (r) = c/r, and setting this equal to me c2 , one obtains for the distance re = mc (2.4)

Note that, taking into account the denition of , this expression does not explicitly depend on : you do not need quantization to compute this distance, which is why it is usually called the classical radius of the electron. At energies this high lowest order perturbation theory may not be sucient to compute a cross-section. The eect of screening (see Lecture 1) becomes important, amplitudes described by Feynman diagrams with loops contribute and QED needs renormalization to provide meaningful answers. We will not discuss renormalization in this course. In fact, we will hardly discuss quantum eld theory at all! Do not be disappointed, there are two pragmatic reasons this. First, a proper treatment requires a proper course with some non-trivial math, which would leave insucient time for other things that we do need to address. Second, if you accept a little handwaving here and there, then we do not actually need QFT: starting from quantum mechanics and special relativity we can derive the Born level that is, leading order cross-sections, following a route that allows us to introduce new concepts in a somewhat historical, and hopefully enlightening, order. However, before continuing and setting aside the eld theory completely until chapter 10, it is worthwhile to briey discuss some gross features of QFT, in particular those that distinguish it from ordinary quantum mechanics. In QM particles are represented by waves, or wave packets. Quantization happens through the fundamental postulate of quantum mechanics that says that the operators for space coordinates and momentum coordinates do not commute, [ x, p ] = i (2.5) The dynamics of the waves is described by the Schr odinger equation. Scattering crosssections are derived by solving, in perturbation theory, a Schr odinger equation with a Hamiltonian operator that includes terms for kinetic and potential energy. Usually we expand the solution around the solution for a free particle and write the solution as a

21 sum of plane waves. This is exactly what you have learned in your QM course and we will come back to this in Lecture 3. In QFT particles are represented as excitations (or quanta) of a eld q (x), a function of spacetime coordinates x. There are only a nite number of elds, one for each type of particle, and one for each force carier. This solves one imminent problem, namely why all electrons are exactly identical. In its simplest form QED has only two elds: one for a spin- 1 electron and one for the photon. The dynamics of these eld are encoded in 2 a Lagrangian density L. Equations of motions are obtained with the principle of least action. Those for the free elds (in a Lagrangian without interaction terms) leads to wave equations, reminiscent of the Schrodinger equation, but now Lorentz covariant. Again, solutions are written as superpositions of plane waves. The elds are quantized by interpreting the elds as operators and imposing a quantization rule similar to that in ordinary quantum mechanics, namely

[q, p] = i

(2.6)

where the momentum p = L/ is the so-called adjoint coordinate to q . (You may remember that you used similar notation to arrive at Hamiltons principle in your classical or quantum mechanics course.) The Fourier components of the quantized elds can be identied as operators that create or destruct eld excitations, exactly what we need for a theory in which the number of particles is not conserved. The relation to classical QM can be made by identifying the result of a creation operator acting on the vacuum as the QM wave in the Schr odinger equation. That was a mouth full and you can to forget most of it. One last thing, though: one very important aspect of quantum eld theory is the role of symmetries in the Lagrangian. In fact, as we shall see in Lecture 10 through 12, the concept of phase invariance allows to dene the standard model Lagrangian by specifying only the matter eld and the symmetries: once the symmetries are dened, the dynamics (the force carriers) come for free. That said, we leave the formal theory of quantum elds alone for now. In this lecture, Lecture 2, we formulate a relativistic wave equation for a spin-0 particle. In Lecture 3, we show how the Maxwell equations take a very simple form when expressed in terms of a new spin-1 eld, which we identify as the photon. In Lecture 4 we discuss classical QM perturbation theory and Fermis Golden rule, which allows us to formalize the computation of a cross-section. In lecture 5 we apply the developed tools to compute 1 eld, which the scattering of spin-0 particles. In Lectures 6, 7 and 8 we turn to spin- 2 are considerably more realistic given that all SM matter elds are indeed fermions. Finally, in Lectures 9 through 12, we introduce the weak interaction, gauge theory and electroweak unication.

22

LECTURE 2. WAVE EQUATIONS AND ANTI-PARTICLES

2.1

Particle-wave duality

Ever since Maxwell we know that electromagnetic elds propagating in a vacuum are described by a wave equation 1 2 2 (x, t) = 0 c2 t2 The solution to this equation is given by plane waves of the form (x, t) = eikxit (2.8) (2.7)

where the wave-vector k and the angular frequency are related by the dispersion relation = c|k| (2.9) (Of course, since the equation above is real, we can restrict ourselves to real solutions. In fact, the photon eld is real. However, it is often more convenient to work with complex waves.) Maxwell identied propagating electromagnetic elds with light, and thereby rmly established what everybody already knew: light behaves as a wave. However, to explain the photo-electric eect Einstein hypothesized in 1904 that light is also a particle with zero mass. For a given frequency, lights comes in packets (quanta) with a xed energy. The energy of a quantum is related to the frequency by E = h = (2.10)

while its momentum is related to the wave-number p = k (2.11)

In terms of energy and momentum the dispersion relation takes the familiar form E = pc. The idea of light as a particle was received with much skepticism and only generally accepted after Compton showed in 1923 that photons scattering of electrons behave as one would expect from colliding particles. So, by 1923 light was a wave and a particle: it satised a wave equation, yet it only came about in packets of discrete energy. That lead De Broglie in 1924 to make another bold preposition: if light is both a wave and a particle, then why wouldnt matter particles be waves as well? It took another few years before physicists established the wave-like character of electrons in diraction experiments, but well before that people took De Broglie hypothesis seriously and started looking for a suitable wave-equation for massive particles. The crucial element is to establish the dispersion relation for the wave. Schr odinger started with the relativistic equation for the total energy E 2 = m2 c4 + p2 c2 (2.12)

2.2. THE SCHRODINGER EQUATION

23

but abandoned the idea, for reasons we will see later. He then continued with the equation for the kinetic energy in the non-relativistic limit E = p2 2m (2.13)

which, as we shall see now, led to his famous equation.

2.2

The Schr odinger equation

One heuristic way to quantize a classical theory is to take the classical equations of motion and substitute energy and momentum by their operators in the coordinate representation, =i = i . and p p (2.14) E E t Inserting these operators in Eq. (2.13), leads to the Schr odinger equation for a free particle, 2 i = 2 . (2.15) t 2m In quantum mechanics we interprete the square of the wave function as a probability density. The probability to nd a particle at time t in a box of nite size V is given by the volume integral P (particle in volume V , t) =
V

(x, t)d3 x

(2.16)

where the density is (x, t) = | (x, t)|2 (2.17) Since total probability is conserved, the density must satisfy a so-called continuity equation +j =0 (2.18) t where j is the density current or ux. When considering charged particles you can think of as the charge per volume and j as the charge times velocity per volume. The continuity equation can then be stated in words as The change of charge in a given volume equals the current through the surrounding surface. What is the current corresponding to a quantum mechanical wave ? It is straightforward to obtain this current from the continuity equation by writing /t = /t + /t and inserting the Schr odinger equation. However, because this is useful later on, we follow a slightly dierent approach. First, rewrite the Schr odinger equation as i = 2 . t 2m

24

LECTURE 2. WAVE EQUATIONS AND ANTI-PARTICLES

Now multiply both sides on the left by and add the expression to its complex conjugate i = t 2m i = t 2m 2 2 + i ( ) = ( ) t 2m
j

where in the last step we have used that ( ) = 2 2 . In the result we can recognize the continuity equation if we interpret the density and current as indicated. Plane waves of the form = N ei(pxEt)/ (2.19) with E = p2 /2m are solutions to the free Schr odinger equation. (In fact, starting from the idea of particle-wave duality, Schr odinger took the plane wave form above and derived his equation as the equation that described its time evolution.) We will leave the denition of the normalization constant N for the next lecture: as the plane wave is not localized in space (it has precise momentum, and innitely imprecise position!), it can only be normalized on a nite volume. To get rid of the inconvenient factor in the exponent, we usually express energy and momentum in terms of the wave vector and angular frequency dened above. Of course, in natural units there is no dierence. However, since it is sometimes useful to verify expressions with a dimensional analysis, we shall keep the factors for now. For the density of the plane wave we obtain = |N |2 |N |2 i ( ) = p j 2m m (2.20) (2.21)

Note that, as expected, the density current is equal to the density times the nonrelativistic velocity v = p/m. Any solution to the free Schr odinger can be written as a superposition of plane waves. Ignoring boundary conditions (which usually limit the energy to quantized values), the decomposition is written as the convolution integral (x, t) =
3

(p)ei(pxEt)/ d3 p

(2.22)

2.3. INTERMEZZO: FOUR-VECTOR NOTATION

25

with E = p2 /2m. Note that for t = 0 this is just the usual Fourier transform. For the exercises, remember that in one dimension the Fourier transform and its inverse are given by (Plancherels theorem), 1 f (x) = 2
+

F (k )e

ikx

dk

1 F (k ) = 2

f (x)eikx dx

(2.23)

If we replace p with p in the plane wave denition out = N ei(pxEt)/ (2.24)

we still have a solution to the Schr odinger equation, since the latter is quadratic in coordinate derivatives. Note that these solutions are already included in the decomposition in Eq. (2.22). By convention when describing scattering in terms of plane waves we identify those with +p x in the exponent as incoming waves and those with p x as outgoing waves. In one dimension, incoming waves travel in the positive x direction and outgoing waves in the negative x direction. Note that waves with E E are not solutions of the Schr odinger equation, but only to its complex conjugate. That is dierent for solutions to the Klein-Gordon equation, which we will describe next.

2.3

Intermezzo: four-vector notation

We dene the coordinate four-vector x as x = (x0 , x1 , x2 , x3 ) (2.25)

where the rst component x0 = ct is the time coordinate and the latter three components are the spatial coordinates (x1 , x2 , x3 ) = x. Under a Lorentz transformation along the x1 axis with velocity = v/c, x transforms as x0 = (x0 x1 ) x1 = (x1 x0 ) x2 = x2 x3 = x3 where = 1/ 1 2 . A general contravariant four-vector is dened to be any set of four quantities A = (A0 , A1 , A2 , A3 ) = (A0 , A) which transforms under Lorentz transformations exactly as (2.26)

26

LECTURE 2. WAVE EQUATIONS AND ANTI-PARTICLES

the corresponding components of the coordinate four-vector x . Note that it is the transformation property that denes what a contravariant vector is. Lorentz transformations leave the quantity |A|2 = A0 |A|2
2

(2.27)

invariant. This expression may be regarded as the scalar product of A with a related covariant vector A = (A0 , A), such that A A |A|2 =

A A .

(2.28)

From now on we omit the summation sign and implicitly sum over any index that appears twice. Dening the metric tensor 1 0 0 0 0 1 0 0 g = g = (2.29) 0 0 1 0 0 0 0 1 we have A = g A and A = g A . A scalar product of two four-vectors A and B can then be written as A B = A B = g A B . (2.30) One can show that such a scalar product is indeed also a Lorentz invariant. You will show in exercise 2.1 that if the contravariant and covariant four-vectors for the coordinates are dened as above, then the four-vectors of their derivatives are given by = 1 , c t and = 1 , . c t (2.31)

Note that the position of the minus sign is opposite to that of the coordinate four-vector itself.

2.4

The Klein-Gordon equation

To nd the wave equation for massive particles Schr odinger and others had originally started from the relativistic relation between energy and momentum, Eq. (2.12). Using again the operator substitution in Eq. (2.14) one obtains a wave equation 1 2 m 2 c4 2 = + 2 c2 t2 (2.32)

This equation is called the Klein-Gordon equation. Having seen it with the factors and c included once, we will from now on omit them.

2.4. THE KLEIN-GORDON EQUATION

27

The Klein-Gordon equation can then be eciently written in four-vector notation as 2 + m2 (x) = 0 where 2 is the so-called dAlembert operator. Planes waves of the form (x) = N ei(pxEt) = eip x

(2.33)

1 2 2 c2 t2

(2.34)

(2.35)

with p = (E, p) are solutions of the KG equation provided that they satisfy the dispersion relation E 2 = p2 + m2 . Note that nothing restricts solution to have positive energy: We will discuss the interpretation of negative energy solutions later in this lecture. Any solution to the KG equation can be written as a superposition of plane waves, like for the Schr odinger equation. However, in contrast to the classical case, the complex conjugate of the plane wave above (x) = N ei(px+Et) = eip x

(2.36)

is also a solution to the KG equation and need to be accounted for in the decomposition. Note that it is not independent though, since (p, E ) = (p, E ). Consequently, we can write the generic decomposition restricting ourselves to positive energy solutions, if we write (x) = d3 p A(p) eip x + B (p) eip x

(2.37)

with E = + p2 + m2 . By popular convention, motovated later, we identify the rst exponent as an incoming particle wave, or an outgoing anti-particle wave, and vice-versa for the second exponent. In analogy to the procedure applied above for the non-relativistic free particle, we now derive a continuity equation. We multiply the Klein Gorden equation for from the left by i , then add to the complex conjugate equation: i i 2 t2 2 2 t

= i 2 + m2 = i 2 + m2 +

i t t t

= [i ( )]
j

28

LECTURE 2. WAVE EQUATIONS AND ANTI-PARTICLES

where we can recognize again the continuity equation. In four-vector notation the conserved current becomes j = (, j ) = i [ ( ) ( ) ] while the continuity equation is simply j = 0 (2.39) (2.38)

You may wonder why we introduced the factor i in the current: this is in order to make the density real. Substituting the plane wave solution gives = 2 |N |2 E j = 2 |N |2 p or in four-vector notation j = 2 |N |2 p . (2.41) (2.40)

Note that, like for the the classical Schr odinger equation, the ratio of the current to the density is still a velocity since v = p/E . However, in contrast to the non-relativistic case, the density of the Klein-Gordon wave is proportional to the energy. This is a direct consequence of the Klein-Gordon equation being second order in the time derivative. We write the conserved current as a four-vector assuming that it transforms under Lorentz transformation the way four-vector are supposed to do. It is not so hard to show this by looking at how a volume and velocity change under Lorentz transformations (see e.g. the discussion in Feynmans Lectures, Vol. 2, sec. 13.7.) The short argument is that since is a Lorentz-scalar, and a Lorentz vector, their product must be a Lorentz vector. You may remember that conservation rules in physics are related to symmetries. That makes you wonder which symmetry leads to the conserved currents for the Schr odinger and Klein-Gordon equations. In Lecture 11 we discuss Noethers theorem and show that it is the phase invariance of the Lagrangian. The phase of the wave functions is not a physical observable. For QM wave functions the conserved current means that probability is conserved. For the QED Lagrangian it implies that charge is conserved.

2.5

Interpretation of negative energy solutions

The constraint E 2 = p2 + m2 leaves the sign of the energy ambiguous. This leads to an interpretation problem: what is the meaning of the states with E = p2 + m2 which have a negative density? We cannot just leave those states away since we need to work with a complete set of states.

2.5. INTERPRETATION OF NEGATIVE ENERGY SOLUTIONS

29

+m

+m

Figure 2.1: Diracs interpretation of negative energy solutions: holes

2.5.1

Diracs interpretation

In 1927 Dirac oered an interpretation of the negative energy states. To circumvent the problem of a negative density he developed a wave equation that was linear in time and space. The Dirac equation turned out to describe particles with spin 1/2. (At this point in the course we consider spinless particles. The wave function is a scalar quantity as there is no individual spin up or spin down component. We shall discuss the Dirac equation later in this course.) Unfortunately, this did not solve the problem of negative energy states. In a feat that is illustrative for his ingenuity Dirac turned to Paulis exclusion principle. The exclusion principle states that identical fermions cannot occupy the same quantum state. Diracs picture of the vacuum and of a particle are schematically represented in Fig. 2.1. The plot shows all the available energy levels of an electron. Its lowest absolute energy level is given by |E | = m. Dirac imagined the vacuum to contain an innite number of states with negative energy which are all occupied. Since an electron is a spin-1/2 particle each state can only contain one spin up electron and one spin-down electron. All the negative energy levels are lled. Such a vacuum (sea) is not detectable since the electrons in it cannot interact, i.e. go to another state. If energy is added to the system, an electron can be kicked out of the sea. It now gets a positive energy with E > m. This means this electron becomes visible as it can now interact. At the same time a hole in the sea has appeared. This hole can be interpreted as a positive charge at that position: an anti-electron! Diracs original hope was that he could describe the proton in such a way, but it is essential that the anti-particle mass is identical to that of the electron. Thus, Dirac predicted the positron, a particle that can be created by pair production. The positron was discovered in 1931 by Anderson.

30

LECTURE 2. WAVE EQUATIONS AND ANTI-PARTICLES

There is one problem with the Dirac interpretation: it only works for fermions!

2.5.2

Pauli-Weisskopf interpretation

Pauli and Weiskopf proposed in 1934 that the density should be regarded as a charge density. For an electron the charge density is written as j = ie( ). (2.42)

To describe electromagnetic interactions of charged particles we do not need to consider anything but the movement of charge. This motivates the interpretation as a charge current. Clearly, in this interpretation solutions with a negative density pose no longer a concern. However, it does not yet solve the issue of negative energies.

2.5.3

Feynman-St uckelberg interpretation

St uckelberg and later Feynman took this approach one step further. Consider the current for a plane wave describing an electron with momentum p and energy E . Since the electron has charge e, this current is j (e) = 2e |N |2 p = 2e |N |2 (E, p) . Now consider the current for a positron with momentum p. Its current is j (+e) = +2e |N |2 p = 2e |N |2 (E, p) . (2.44) (2.43)

Consequently, the current for the positron is identical to the current for the electron but with negative energy and traveling in the opposite direction. Or, in terms of the plane waves, to go from the positron current to the electron current, we just need to change the sign in the exponent of eix p . By our earlier convention, this is equivalent to saying that the ingoing plane wave of a positron is identical to the outgoing wave of an electron. Now consider what happens to the electron wave function if we change the direction of time: We will have ct ct and p p. You immediately notice that this has exactly the same eect on the plane wave exponent as the transformation (E, p) (E, p). In other words, we can interprete the negative energy current of the electron as an electron moving backward in time. This current is identical to that of a positron moving forward in time. This interpretation, illustrated in Fig. 2.2, is very convenient when computing scattering amplitudes: in our calculations with Feynman diagrams we can now express everything in terms of particle waves, replacing every anti-particle with momentum p by a particle with momentum p , as if it were traveling backward in time. For example, the process of an absorption of a positron with energy E is the same as the emission of an electron

2.5. INTERPRETATION OF NEGATIVE ENERGY SOLUTIONS


e+

31

E>0

E<0

Figure 2.2: A positron travelling forward in time is an electron travelling backwards in time.

with energy E (see Fig.2.3). Likewise, the process of an incoming positron scattering o a potential will be calculated as that of a scattering electron travelling back in time (see Fig. 2.4).

e (+E,p) absorption time emission +e (E,p)


Figure 2.3: There is no dierence between the process of an absorption of a positron with p = (E, p) and the emission of an electron with p = (e, p).

e+ e
time
x

Figure 2.4: In terms of the charge current density j+( E,p) (+e) j(E,p) (e)

The advantage of this approach becomes more apparent when one considers higher order corrections to the amplitudes. Consider the scattering of an electron on a localized potential, illustrated in Fig. 2.5. To rst order the interaction of the electron with the perturbation is described by the exchange of a single photon. When the calculation is extended to second order the electron interacts twice with the eld. It is important to note that this second order contribution can occur in two time orderings as indicated in

32

LECTURE 2. WAVE EQUATIONS AND ANTI-PARTICLES

the gure. These two contributions are dierent and both of them must be included in a relativistically covariant computation. The time-ordering on the right can be viewed in two ways: The electron scatters at time t2 runs back in time and scatters at t1 . First at time t1 spontaneously an e e+ pair is created from the vacuum. Lateron, at time t2 , the produced positron annihilates with the incoming electron, while the produced electron emerges from the scattering process. The second interpretation would allow the process to be computed in terms of particles and anti-particles that travel forward in time. However, the rst interpretation is just more economic. We realize that the vacuum has become a complex environment since particle pairs can spontaneously emerge from it and dissolve into it!

e
time
x

e
t2 t1
x

t2 t1

x x

Figure 2.5: First and second order scattering.

Exercises
Exercise 2.1 (From A&H, chapter 3. see also Griths, exercise 7.1) Write down the inverse of the Lorentz transformation along the x1 axis dened in Eq. (2.26) ( i.e. express (x0 , x1 ) in (x0 , x1 )). Then use the chain rule of partial dierentiation to show that, under the Lorentz transformation, the operators (/x0 , /x1 ) transform in the same way as (x0 , x1 ).

2.5. INTERPRETATION OF NEGATIVE ENERGY SOLUTIONS

33

Exercise 2.2 (KG conserved current) Verify that the conserved current Eq. (2.38) satises the continuity equation: compute j explicitly and make use of the KG equation to show that the result is zero. Hint: note that that for any fourvectors A and B we have A B = A B . Exercise 2.3 (The Dirac -Function)
infinite

Consider a function dened by the following prescription (x) = lim 1/ for |x| < /2 0 otherwise
0
surface = 1

The integral of this function is normalized

(x) dx = 1

(2.45)

and for any (reasonable) function f (x) we have

f (x) (x) dx = f (0).

(2.46)

These last two properties dene the Dirac -function. The prescription above gives an approximation of the -function. We shall encounter more of those prescriptions which all have in common that they are the limit of a sequence of functions whose properties converge to those given here. (a) Starting from the dening properties of the -function, prove that (kx) = (b) Prove that
n

1 (x) . |k |

(2.47)

(g (x)) =
i=1

1 (x xi ) , |g (xi )|

(2.48)

where the sum i runs over the 0-points of g (x), i.e.:g (xi ) = 0. Hint: make a Taylor expansion of g around the 0-points. Exercise 2.4 (Some exercises with the -function) (a) Calculate (b) Calculate (c) Calculate (d) Simplify
3 0 3 0 3 0

ln(1 + x) ( x) dx (2x2 + 7x + 4) (x 1) dx ln(x3 ) (x/e 1) dx (5x 1) x 1

34

LECTURE 2. WAVE EQUATIONS AND ANTI-PARTICLES

(e) Simplify (sin x) and draw the function (Note: just writing a number is not enough!) Exercise 2.5 (Wave packets) In the coordinate representation a plane wave with momentum p = k is innitely dislocalised in space. This does not quite correspond to our picture of a particle, which is why we usually visualize particles as wave packets, superpositions of plane waves that have a nite spread both in momentum and coordinate space. As we shall see in the following, the irony is that such wave packets are dispersive in QM: their size increases as a function of time. So, even wave packets can hardly be thought of as representing particles. (a) Consider a one-dimensional Gaussian wave packet that at time t = 0 is given by (x, 0) = Aeax
+
2 +ik 0x

with a real and positive. Compute the normalization constant A such that | (x, 0)|2 dx = 1.

Hint:

ey dy =

(b) Take the Fourier transform to derive the wave function in momentum space at t = 0, 1/4 1 2 e(kk0 ) /4a (k ) = 2a Hint: You can write exp ax2 + bx = exp a(x b/2a)2 + b2 /4a and then move the integration boundaries by b/2a. (Dont mind that b is complex.) (c) Use this result and Eq. (2.22) to show that the solution to the Schr odinger equation 2 2 (with E (p) = p /2m or (k ) = k /2m) is given by (x, t) = with 2 a/m. (d) Compute | (x, t)|2 . Qualitatively, what happens to 2 as time goes on? (e) Now compute the same for a solution to the massless Klein-Gordon equation ( = ck ). Note that the wave packet maintains its size as a function of time. 2a
1/4

(1 + it)1/2 exp

2 ax2 + ik0 x itk0 /4a 1 + it

Lecture 3 The Electromagnetic Field


3.1 The Maxwell Equations

In classical electrodynamics the movement of a point particle with charge q in an electric eld E and magnetic eld B follows from the equation of motion dp = q (E + v B ). dt (3.1)

The Maxwell equations tell us how electric and magnetic eld are induced by static charges and currents. In vacuum they can be written as: Gauss law No magnetic charges Faradays law of induction Modied Amp eres law E =
0

(3.2) (3.3) (3.4) (3.5)

B =0 B E+ =0 t E j c2 B = t 0

where 0 is the vacuum permittivity. From the rst and the fourth equation we can derive the continuity equation for electric charges, j = . Historically, it was t the continuity equation that lead Maxwell to add the time dependent term to Amp` eres law. The constant c in the Maxwell equations is, of course, the velocity of light. When Maxwell formulated his laws, he did not anticipate this. He did realize that c is the velocity of a propagating electromagnetic wave. The value of c2 can be computed from measurements of 0 (e.g. with the force between static charges) and measurements of c2 0 (e.g. from measurements of the force between static currents). From the fact that the result was close to the known speed of light Maxwell concluded that electromagnetic waves and light were closely related. He had, in fact, made one of the great unications 35

36

LECTURE 3. THE ELECTROMAGNETIC FIELD

of physics! For a very readable account, including an explanation of how electromagnetic waves travel, see the Feynman lectures, Vol.2, section 18. From now on we choose units of charge such that we can set 0 = 1 and velocities such that c = 1. (That is, we use so-called Heaviside-Lorentz rationalised units. See section 1.6.) For what follows it is convenient to write the Maxwell equations in a covariant way (i.e. in a manifestly Lorentz invariant way). As shown below we can formulate them in terms of a single 4 component vector eld, which we denote by A = (V /c, A). As suggested by our notation, the components of this eld transform as a Lorentz vector. Remember that the following identities are valid for any vector eld A and scalar eld V: divergence of rotation is 0: rotation of gradient is 0: ( A) = 0 ( V ) = 0 (3.6) (3.7)

From electrostatics you may remember that, because the rotation of E is zero (which is the same as saying that E is a conservative vector eld), all physics can be derived by considering a scalar potential eld V . The electric eld becomes the gradient of the potential, E = V . The potential V is not unique: we can add an arbitrary constant and the physics will not change. Likewise, you may have seen in your electrodynamics course that, because the divergence of the B eld is zero, we can always nd a vector eld A such that B is the rotation of A. So, lets choose a vector eld A such that B = A and a scalar eld V such that E= (3.8)

A V (3.9) t Then, by virtue of the vector identities above, the Maxwell equations 3.3 and 3.4 are automatically satised. What remains is to write the other two equations, those that involve the charge density and the charge current density, in components of A and A0 = V /c as well. You will show in exercise 3.1 that these can be written very eciently as A A = j . (3.10)

We have argued in the previous lecture that a conserved current j transforms as a Lorentz vector. (It is easy to work this out for yourself. See also Feynman, Vol.2, section 13.6.) The derivative also transforms as a Lorentz vector. Therefore, it follows that if the equation above is Lorentz covariant, then A must transform as a Lorentz vector as well. Showing that the electromagnetic eld indeed transform this way is outside the scope of these lecture, but you may know that the transformation

3.2. GAUGE TRANSFORMATIONS

37

properties of the elds were an important clue when Einstein formulated his theory of special relativity. The expressions can be made even more compact by introducing the tensor F A A . such that F = j . (3.12) Just as the potential V in electrostatics was not unique, neither is the eld A . Imposing additional constrains on A is called choosing a gauge. In the next section we shall discuss this freedom in more detail. Written out in terms of the components E and B the (4 4) matrix for the electromagnetic eld tensor F is given by 0 Ex Ey Ez Ex 0 Bz By . = Ey Bz 0 Bx Ez By Bx 0 (3.11)

(3.13)

Note that F is uniquely specied in terms of E and B . In other words, it does not depend on the choice of the gauge.

3.2

Gauge transformations

As stated above the choice of the eld A is not unique. Transformations of the eld A that leave the electric and magnetic elds invariant are called gauge transformations. In exercise 3.2 you will show that for any scalar eld (t, x), the transformations t A = A . V = V + or in terms of four-vectors A A = A + do not change E and B . If the laws of electrodynamics only involve the electric and magnetic elds, then, when expressed in terms of the eld A, the laws must be gauge invariant: physical observables should not depend on . Sometimes we choose a particular gauge in order to make the expressions in calculations simpler. In other cases, we require gauge invariance to impose constraints on a solution, as with the photon below. (3.15)

(3.14)

38

LECTURE 3. THE ELECTROMAGNETIC FIELD

A gauge choice that is often made is called the Lorentz gauge 1 . In exercise 3.3 you will show that it is always possible to choose the gauge eld such that A satises the condition Lorentz condition: A = 0. (3.16) Note that this implies that A becomes a conserved current. In the Lorentz gauge the Maxwell equations simplify further: Maxwell in Lorentz gauge: A = j (3.17)

However, as you will see in the exercise, A still has some freedom since the Lorentz condition requires us to x only ( ) and not itself. In other words a gauge transformation of the form A A = A + with 2 = = 0 (3.18)

is still allowed within the Lorentz gauge A = 0. Consequently, we can in addition impose the Coulomb condition : Coulomb condition: A0 = 0 (3.19)

(Note that, given the Lorentz condition, also A = 0 with this choice of gauge.) Note that this choice of gauge is not Lorentz invariant. This is allowed since the choice of the gauge is irrelevant for the physics observables, but it is sometimes considered not elegant.

3.3

The photon

Let us now turn to electromagnetic waves and consider Maxwells equations in vacuum in the Lorentz gauge, vacuum: j = 0 = 2A = 0. (3.20)

We recognize in each component the Klein-Gordon equation of a particle with mass m = 0. (See Eq. (2.32) in the previous lecture.) This particle is the photon. It represents an electromagnetic wave, a bundle of electric and magnetic eld that travels freely through space, no longer connected to the source. Using results below you can show that the E and B elds of such a wave are perpendicular to the wave front and perpendicular to each other. Furthermore, the magnitudes are related by the speed of light, |E | = c|B |.
1 It is actually called the Lorenz condition, named after Ludvig Lorenz (without the letter t). It is a Lorentz invariant condition, and is frequently called the Lorentz condition because of confusion with Hendrik Lorentz, after whom Lorentz covariance is named. Since almost every reference has this wrong, we will use Lorentz as well.

3.3. THE PHOTON

39

We have seen before that the following complex plane waves are solutions of the KleinGorden equation, (x) eip x and (x) eip x (3.21) For a given momentum vector p any solution in the complex plane is a linear combination of these two plane waves. However, you may have noticed that, in contrast to the Schr odinger equation, the Klein-Gorden equation is actually real. Since the E and B elds are real, we restrict ourselves to solutions with a real eld A . We could write down the solution to 2A = 0 considering only the real axis, but it is customary (and usually more ecient) to form the real solutions by combining the two complex solutions, A (x) = a (p)eipx + a (p) eipx =2 a (p)eipx (3.22)

(Note that the second term is the complex conjugate of the rst.) The four-vector a (p) depends only on the momentum vector. It has four components but due to the gauge transformation not all of those are physically meaningful. The Lorentz condition gives 0 = A = ip a eipx + ip a eipx , which leads to p a = 0. (3.24) The Lorentz condition therefore reduces the number of independent complex components to three. However, as explained above, we have not yet exhausted all the gauge freedom: we are still free to make an additional shift A A , provided that itself satises the Klein-Gordon equation. If we choose it to be = ieipx i e+ipx with a complex constant, then its derivative is = p eipx + p eipx . (3.26) (3.25) (3.23)

With a bit of algebra we see that the result of the gauge transformation corresponds to a = a + p (3.27)

Note that a still satises the Lorentz condition only because p2 = 0 for a massless photon. As we have already seen, this additional freedom allows us to apply the Coulomb condition and choose A0 = 0, or equivalently a0 (p) = 0. In combination with the Lorentz condition this leads to ap=0 (3.28) or p A = 0.

40

LECTURE 3. THE ELECTROMAGNETIC FIELD

At this point it is customary to uniquely factorize a(p) as follows a(p) N (p) (p) (3.29)

such that the vector has unit length and N (p) is real. The normalization N (p) depends only on the magnitude of the momentum and is essentially just the energy density of the wave. The vector depends only on the direction of p and is called the polarization vector. Choosing the z axis along the direction of the momentum vector and imposing the gauge conditions, the latter can be parameterized as = (c1 ei1 , c2 ei2 , 0) . (3.30)

2 where ci and i are all real and c2 1 + c2 = 1. Note that we can remove one phase by moving the origin. (Just look at how a shift of the origin aects the factors eipx .) Therefore, only two parameters of the polarization vector are physically meaningful: these are the two polarization degrees of freedom of the photon.

Any polarization vector can be written as a (complex) linear combination of the two transverse polarization vectors 1 = (1, 0, 0) 2 = (0, 1, 0) . (3.31)

If the phases of the two components are identical, the light is said to be linearly polarized. If the two components have equal size (c1 = c2 = 2) but a phase dierence of /2, the light wave is circularly polarized. The corresponding circular polarization vectors are +1 i2 1 i2 = (3.32) + = 2 2 You will show in exercise 3.4 that the circular polarization vectors + and transform under a rotation with angle around the z -axis (the momentum direction) as + + = ei + = ei (3.33)

We now show that this means that these polarization states correspond to the two helicity eigenstates of the photon. You may remember from your QM course that the z component of the angular momentum operator, Jz is the generator of rotations around the z -axis. That means that for a wavefunction (x) the eect of an innitesimal rotation around the z axis is given by (x) U ( ) (x) (1 i J3 ) (x). (3.34)

A arbitrarily large rotation may be built up from innitesimal rotations by dividing it in innitesimal steps U () = lim (U (/n))n
n n

ninf ty

lim

1i

Jz )

= eiJz

(3.35)

3.4. ELECTRODYNAMICS IN QUANTUM MECHANICS

41

Consequently, if is an eigen vector of Jz with eigenvalue m, then for any rotation around the z -axis we have (x) U () (x) = eim (x) (3.36)

Comparing this to the eect of rotations on the polarization states above we now identify + with a m = +1 state and with a m = 1 state. Apparently, the polarization states belong to a representation of the rotation group: they are spin states. Since we nd 1 for the Jz quantum number the photon must be a spin-1 representation: it couldnt be spin zero, because than you would have only have a state with m = 0. And it couldnt have more, because there are no degrees of freedom in the photon eld that could be identied with higher values of m. Since the photon is spin-1 one could have expected to nd 3 spin states, namely for mz = 1, 0, +1. You may wonder what happened to the mz = 0 component. This component was removed when we applied the Coulomb gauge condition, exploiting p2 =, leading to A p = 0. For massive vector elds (or virtual photons!), there is no corresponding gauge freedom and a component parallel to the momentum (a longitudinal polarization) will remain. Another way to look at this is to say that to dene spin properly one needs to boost to the rest frame of the particle. For the massless photon this is not possible. Therefore, we can talk only about helicity (spin projection on the momentum) and not about spin. The equivalent of the mz = 0 state does not exist for the photon. Finally, lets compute the electric and magnetic elds. Substituting the generic expression for A in the denitions of E and B and exploiting the coulomb condition A0 V = 0, we nd E = i a p0 eipx + c.c. B = i (p a) eipx + c.c. (3.37)

Indeed, for the electromagnetic waves, the E and B elds are perpendicular to each other and to the momentum, while the ratio of their amplitudes is 1 (or rather, c).

3.4

Electrodynamics in quantum mechanics

In classical mechanics an elegant way to introduce electrodynamics is via a method called minimal substitution. The method states that the equation of motion of a charged particle under the inuence of a vector eld A can be obtained by making the substitution p p qA . (3.38)

in the equations of motion of the free particle. Written out in terms of the potential V and vector potential A, the free Hamiltonian is then replaced by 1 (p q A)2 + qV (3.39) H= 2m

42

LECTURE 3. THE ELECTROMAGNETIC FIELD

It can be shown (see e.g. Jackson 12.1, page 575) that this indeeds leads to the Lorentz force law, Eq. (3.1). Performing the operator substitution, the Schr odinger equation for the Hamiltonian above becomes 1 (i q A)2 + qV 2m (x, t) = i (x, t) t (3.40)

Comparing this to the Schr odinger equation for the free particle, we note that we have essentially made the substitution + iq A /t /t + iqV In four-vector notation this can be written as D + iqA (3.42) (3.41)

The derivative D is called a covariant derivative, but that is not so important now. We may now wonder what the eect of the gauge transformation in Eq. (3.15) is on the wave function . We have just established that the classical E and B elds do not depend on gauge transformations. However, that does not mean that the wave function is invariant as well. In fact, as one can show (see e.g. Aichison and Hey, section 2.4) the combined transformation, required to make the Schr odinger equation invariant, is given by A A = A + = exp (iq/ )

(3.43)

Note that the gauge transformation leads to a change of the phase of the wave function. If is not constant, then the change in phase is dierent at dierent points in space. That is why we also call the gauge transformation a local phase transformation. (Also note that has the dimension of action (e.g. of ) over charge. The vector potential A has the dimension of momentum over charge.) This result is at the heart of the application of gauge symmetries in quantum eld theory. Because, as we will see in more detail in Lecture 10, one can turn this argument around: Since the phase of the wave function is not an observable, the equations that describe the dynamics (a Schr odinger equation, or a Lagrangian) must be invariant to such arbitrary phase transformations. If we impose this requirement, then we are forced to introduce an A eld in the Hamiltonian via the substitution above and with transformation properties dened above. In other words, the requirement of local phase invariance imposes the form of the interaction!

3.5. THE AHARANOV-BOHM EFFECT

43

3.5

The Aharanov-Bohm Eect

In the classical equations of motion only the E and B elds appear. So, you may wonder, is the A eld real, or merely a mathematical construct that simplies expressions? Or phrased dierently, does it contribute anything to our description of moving charges that the E and B elds do not? The answer to that question is beautifully illustrated by what is called the Aharanov-Bohm eect. In quantum mechanics we do not have forces: it is the amplitude of a wave function that tells us where we are likely to nd the particle in space and time. In Feynmans path integral picture, the quantum mechanical particle follows all possible trajectories to get from point x1 to point x2 . As these trajectories have dierent length, the phases along the trajectories are not identical. It is only around the classical trajectory that these phases interfere constructively. The size of deviations along the classical trajectory is determined by . As we have seen above the vector potential appears in the Schr odinger equation and aects the wave function. In the presence of a magnetic eld, the phase of the wave function is changed along a trajectory according to (A) = q
x2

A(r, t) dr
x1

(3.44)

where the integral runs along the trajectory. (We dont prove this here. See also Feynman Vol 2, section 15-5.) Although we do not need it here, for completeness we also mention that the change in phase due an electron eld is given by the integral of the potential over the time: (V ) = q
t2

V (r, t)dt
t1

(3.45)

This last equation you could easily derive from the SE for a constant electric eld. Youll realize that when combined these two equations lead to a Lorentz covariant formulation if the integral is performed over space and time. Let us now go back to the famous two-slit experiment of Feynman in which he considers the interference between two possible electron trajectories. From quantum mechanics we know that the intensity at a detection plate positioned behind the two slits shows an interference pattern depending on the relative phases of the wave functions 1 and 2 that travel dierent paths. For a beautiful description of this see chapter 1 of the Feynman Lectures on Physics volume 3 (2-slit experiment) and pages 15-8 to 15-14 in volume 2 (Bohm-Aharanov). The idea is schematically depicted in Fig. 3.1. In case a eld A is present the phases of the wave functions are aected, such that the wave function on the detector is: = 1 ei1 (r,t) + 2 ei2 (r,t) = 1 ei(1 2 ) + 2 ei2 (3.46)

44

LECTURE 3. THE ELECTROMAGNETIC FIELD

detector

slits 2 source
1

Intensity

coil

Figure 3.1: The schematical setup of an experiment that investigates the eect of the presence of an A eld on the phase factor of the electron wave functions.

We note that the interference between the two amplitudes depends on the relative phase: 1 2 = = q
r1

dr1 A1
r2

dr2 A2 q

dr A(r , t) q (3.47)

q
S

A(r , t) dS =

B dS =
S

where we have used Stokes theorem to relate the integral around a closed loop to the magnetic ux through the surface. In this way the presence of a magnetic eld can aect, (i.e. shift) the interference pattern on the screen. In exercise 3.5 you will show that for a homogenous magnetic eld this leads to the same deection as the classical force law. Let us now consider the case that a very long and thin solenoid is positioned in the setup of the two-slit experiment. Inside the solenoid the B-eld is homogeneous and outside it is B = 0 (or suciently small), see Fig. 3.2. However, from electrodynamics we recall the A eld is not zero outside the coil. There is a lot of A circulation around the thin coil. The electrons in the experiment pass through this A eld which quantum mechanically aects the phase of their wave function and therefore also the interference pattern on the detector. On the other hand, there is no B eld in the region, so classically there is no eect. Experimentally it has been veried (in a technically dicult experiment) that the interference pattern will indeed shift. Discussion: We have introduced the vector potential as a mathematical tool to write Maxwells equations in a Lorentz covariant form. In this formulation we noticed that the A-eld has some arbitrariness due to gauge invariance. However, in QM we observe that the vector eld A is not just an alternative formulation, but the only correct way to implement the Maxwell equation in the quantum world. The freedom due to gauge transformations

3.5. THE AHARANOV-BOHM EFFECT

45

Figure 3.2: Magnetic eld and vector potential of a long solenoid.

may seem an undesirable feature now, but will turn out to be a fundamental concept in our description of interactions.

Exercises
Exercise 3.1 (Maxwell equations) Using the vector identity ( A) = 2 A + ( A) (which one can prove using ijk klm = il jm im jl ) show that (with c = 1 and Maxwells equations can be written as: A A = j Hint: Derive the expressions for and j explicitly. Exercise 3.2 (Gauge transformation) Verify that the transformation in Eq. (3.14) does not change the E and B elds. Exercise 3.3 (Lorentz gauge) Show that it is always possible to choose a gauge such that the A eld satises the Lorentz condition, Eq. (3.16). To do this assume that for a given A eld one has A = g (x). Then derive the gauge eld (x) such that A = 0. Exercise 3.4 (Helicity) Show that the circular polarization vectors + and transform under a rotation of angle around the z -axis as = ei (3.50) (3.48)
0

= 1)

(3.49)

46

LECTURE 3. THE ELECTROMAGNETIC FIELD

Figure 3.3: From The Feynman Lectures in Physics, volume II.

Exercise 3.5 (Deection in magnetic eld. Feynman, Vol II, sec. 15-5.) We have stated above that with the minimal substitution recipe the Schr odinger equation leads to the Lorentz force law. We have also stated (not proven) how the change of the phase of a wavefunction due to a vector eld A can be obtained by integrating the vector eld along the trajectory. Lets take these things for given and see if we can reproduce the deection of a particle in a magnetic eld. Feynman beautifully illustrates that by looking at the famous two-split experiment. Consider the setup in Fig. 3.3. Particles with charge q , mass m and momentum p travel from a source, via two slits, to a photographic plate. The interference of the two paths leads to a diraction pattern. The distance between the slits and the plate is L. Directly behind the slits is a thin strip of magnetic eld. The thickness of the strip is w and w L. The B eld is homogenous, coming out of the plane of the gure. We label the coordinate along the photographic plate by x. (a.) For very small deections, compute the deection of the particles of the particles using the Lorentz force law. Translate this into the displacement x at the photographic plate. Hint: Assume that the plate is thin enough that direction of the force is along the x-axis. The force lasts for a time w/v . (b.) Consider two classical (shortest distance) trajectories through the two slits (indicated by 1 and 2 ). For small deections, compute the phase shift between the two trajectories as a function of x, in the absence of a magnetic eld. Compute the distance between two maxima in the diraction pattern. Hint: The reduced wavelength of the particles is /2 = /p. (c.) Assuming again small deections use equation (3.47) to compute the increase in phase shift between the two trajectories as a result of the B eld. Translate the phase shift in a shift x of the diraction pattern.

Lecture 4 Perturbation Theory and Fermis Golden Rule


In this chapter we discuss Fermis golden rule, which allows us to compute cross-sections and decay rates. A very readable account of this is given in Griths, chapter 6.

4.1

Decay and scattering observables

Except for a few lucky ones, most species of particles do not live long. This holds for all baryons except the proton (even the neutron decays, when it is not inside a nucleus), but also for the muon and the tau. As particles do not age, the probability to decay is independent of time. Given a large number of particles N0 , the number of surviving particles is hence given by the exponential law N (t) = N0 et/ (4.1)

where is the mean lifetime. For particles that decay via the weak interaction, the mean lifetime is typically 1012 109 seconds. A notable exception is the neutron which lives for about 15 minutes. The mean decay time is inversely proportional to what is called the decay width = (4.2)

which has units of energy. If the particle can decay through dierent decay channels (e.g. a charged pion can decay to and to e ), then the decay width can be written as the sum of the decay widths to the individual channels =
i

i .

(4.3)

47

48

LECTURE 4. PERTURBATION THEORY AND FERMIS GOLDEN RULE

The ratio i / is called the branching fraction. The particle data book is full of branching fractions of species in the particle zoo. If the decay is to more than two particles, the distribution of angles and energies of particles in the nal state becomes an observable as well. That is why we often consider partial or dierential decay widths, d dp1 dpN where p1 , . . . , . . . , pN are the momenta of the N particles in the nal state. Besides decay widths we also measure scattering cross-sections. (In fact, in our computations, decays and scattering are quite similar, so we deal with both at once.) In scattering experiments we collide beams of particles and study the collision rate. Consider an experiment in which we scatter a beam of particles A and a target of particles B . If nA is the particle number density in the beam, and vA is the particle velocity, the number of collisions per second per unit volume of B is dN = vA nA nB tot dt (4.5) (4.4)

The quantity tot is called the total scattering cross-section. It has units of surface. In most cases we do not study the total collision rate, but rather the rate of particular nal states. The total cross-section is a sum of cross-sections for all possible nal states, such that tot = i (4.6)
i

Since the energy and direction of nal state particles can be measured as well, we usually consider dierential scattering cross-sections, d (A + B f1 + + fN ) dp1 dpN (4.7)

The expression for the calculation of a (dierential) cross section can be written schematically as W d = d (4.8) ux The ingredients to this expression are: 1. the transition rate W . You can think of this as the probability per unit time and unit volume to go from an initial state i to a nal state f ; 2. a ux factor that accounts for the density of the incoming states; 3. the Lorentz invariant phase space factor d, sometimes referred to as dLIPS. It accounts for the density of the outgoing states. (It takes care of the fact that experiments cannot observe individual states but integrate over a number of states with near equal momenta.)

4.2. NON-RELATIVISTIC SCATTERING

49

Note that the physics (the dynamics of the interaction) is contained in the transition rate W . The ux and the phase space factors are just bookkeeping, required to compare the result with the measurements. The rigorous computation of the transition rate requires quantum eld theory, which is outside the scope of this course. However, to illustrate the concepts we will have a discussion of non-relativistic scattering of a single particle in a time-dependent potential and formulate the result in a Lorentz covariant way. In the next chapter we will derive the lowest order amplitude for the scattering of A + B A + B , which can still be done without eld theory. We can link that result to the Feynman rules derived in eld theory. This is roughly the approach followed in Halzen and Martin. Griths takes an alternative approach: it just starts with the full expressions and presents the Feynman rules without deriving them.

4.2

Non-relativistic scattering
t=T/2 t=0 t=T/2

H0 f

H0 i V (x,t)

Figure 4.1: Scattering of a single particle in a potential.

Consider the scattering of a particle in a potential as depicted in Fig. 4.1 Assume that both long before and long after the interaction takes place, the system is described by the free Schr odinger equation, i = H0 (4.9) t where H0 is the unperturbed, time-independent Hamiltonian for a free particle. Let m (x) be a normalized eigenstate of H0 with eigenvalue Em , H0 m (x) = Em m (x). The states m form an orthonormal basis,
3 m (x) n (x) d x = mn .

(4.10)

(4.11)

Note that we use the Kronecker delta, as if the spectrum of eigenstates is discrete. In the chapter 2 we considered a continuous spectrum of eigenstates for the free Hamiltonian,

50

LECTURE 4. PERTURBATION THEORY AND FERMIS GOLDEN RULE

numbered by the wave number k . Eventually, we could do that here, too, replacing the kronecker delta by a Dirac delta-function. However, it is trivial to change between the too and the notation is a bit easier when we work with a discrete set of states. The time-dependent wave function m (x, t) = m (x) eiEm t/ . (4.12)

is a solution to the Schr odinger equation. Since these states form a complete set, any other wave function can be written as a superposition of the wave functions m . Now consider a Hamiltonian that includes a time-dependent perturbation, i = (H0 + V (x, t)) . t (4.13)

Any solution can be written as

=
n=0

an (t) n (x) eiEn t .

(4.14)

We require to be normalized, which implies that nd in state n at time t is just |an (t)|2 .

|an (t)|2 = 1. The probability to

To determine the co ecients an (t) we substitute (4.14) in (4.13) and nd

i
n=0

dan (t) n (x) eiEn t = dt

V (x, t) an (t) n (x) eiEn t ,


n=0

(4.15)

where we have used that the m are solutions of the free Schr odinger equation. Multiply iEf t and integrate over x to obtain the resulting equation from the left with f = f (x) e

i
n=0

dan (t) dt

i(En Ef )t = d3 x f (x) n (x) e f n

an (t)
n=0

i(En Ef )t/ d3 x f (x) V (x, t) n (x) e

(4.16)

Using the orthonormality relation for m we then arrive at the following coupled linear dierential equation for ak (t), dak (t) i = dt where we have dened kn = (Ek En )/ (4.18)

an (t) Vkn eikn t ,


n=0

(4.17)

4.2. NON-RELATIVISTIC SCATTERING and what is sometimes called the matrix element Vkn (t) = d3 x k (x) V (x, t) n (x) .

51

(4.19)

In some cases the set of equations (4.17) can be solved explicitly. A general solution is obtained in perturbation theory, by expanding in Vkn . The approximation of order p + 1 can be obtained by inserting the p-th order result on the right hand side of Eq. (4.17), i dak
(p+1)

(t)

dt

p) ikn t a( n (t)Vkn (t)e

(4.20)

Without loss in generality we now assume that the incoming wave is prepared in eigenstate i of the free Hamiltonian, i.e. ak () = ki . The zeroeth order approximation (0) then is ak (t) = ki (no interaction occurs) and the rst order result becomes dak (t) i = Vki (t)eiki t dt
(1)

(4.21)
(1)

Using that af () = 0 and integrating this equation we obtain for the coecient ak (t) at time t, ak (t) =
(1) t

1 daf (t ) dt = dt i

Vki (t )eiki t dt

for k = i

(4.22)

Higher order approximations can be obtained by inserting the lowest order solution in the right side of Eq. (4.20). (See textbooks.) A graphical illustration of the rst and second order perturbation is given in Fig. 4.2. Note that the lowest order approximation makes one quantum step from the initial state i to the nal state f , while the second order approximation includes all amplitudes i n f .

1st order f
time

2nd order f

Vfn
space

Vfi
i i

V ni

Figure 4.2: First and second order approximation in scattering.

In the following we only consider the rst order approximation (Born approximation). We dene the transition amplitude T as the amplitude to go from a state i to a nal state f at large times, T af (t ) = 1 i

dt

d3 x f (x, t) V (x, t) i (x, t)

(4.23)

52

LECTURE 4. PERTURBATION THEORY AND FERMIS GOLDEN RULE

where we substituted the denitions of Vkn and kn . We can write the result more compactly as 1 T = (x) V (x) i (x) (4.24) d4 x f i Note that the expression for T has a Lorentz covariant form: since it is a scalar, and any dependence on x and t is integrated out, it is independent of the Lorentz frame. Indeed, the expression does not change when we consider relativistic scattering. We now make a simplication and consider a potential that is time-independent. The expression for the transition amplitude then becomes Tf i = Vf i i

eif i t dt = 2 i Vf i (Ef Ei )

(4.25)

where we have used that the integral is an important representation of the Dirac function + 1 (x) = eikx dk (4.26) 2 and substituted our denition of f i . The function expresses conservation of energy. Note that Tf i is dimensionless. Can we interprete |Tf i |2 as a probability? Well, there is one conceptual problem and one pragmatic problem. The conceptual problem is that if the potential is time-independent, then this probability will just grow with time. The pragmatic problem is that there is the function. These issues can be solved by considering a potential that is turned on for a nite time T and dening the mean transition rate in the limit for large T , Wf i lim |Tf i |2 T T (4.27)

Assume now that the interaction is turned on at time T /2 and turned o at time T /2. The equation above can then be integrated to give for the transition amplitude at T /2, af (T /2) = Vf i i
T /2

eif i t dt =
T /2

2Vf i sin(f i T ) i f i

(4.28)

Inserting this in the denition for the transition rate gives Wf i lim 4|Vf i |2 sin2 (f i T /2) 2 2 T f iT (4.29)

The function on the right is strongly peaked near f i = (Ef Ei )/ = 0, again enforcing energy conservation. In fact, for T it is yet another representation of the Dirac function, 1 sin2 x (x) = lim (4.30) x2

4.2. NON-RELATIVISTIC SCATTERING Substituting this in the equation, we obtain Wf i = 2


2

53

|Vf i |2 (f i ) =

|Vf i |2 (Ef Ei ) .

(4.31)

You can verify that Wf i is indeed a rate: Vf i is an energy, one of the factors of energy is canceled by the function and the other one is divided by to turn it into reciprocal time. As indicated before we can never actually probe nal states with denite energy in a measurement with nite duration. In general, there will be a number of states with energy close to Ei that can be reached. The number of nal states with energy between Ef and Ef + dEf is given by dn = (Ef )dEf (4.32) where (Ef ) is the density of states per unit energy near Ef . Integrating the expression for the transition rate over all nal state energies, we obtain Fermis (Second) Golden Rule, W fi = 2 W (Ef ) dEf (4.33) |Vf i | (Ei ) .
2

Above, we encountered a function in the transition amplitude. To deal with the square of that function we considered a nite time interval and went back to the expression (0) for ak (T /2) for nite times T , taking the limit T only after taking the square. To make the nal step you need to know that in the limit sin2 x/x2 is a representation of the function. For future applications it is useful to know that you can do it also slightly dierently, namely by taking the limit T one integral at a time: 1 |W | = lim T T = |Vf i |2
2

Vf i i

T /2

e
T /2

if i t

dt
T /2

Vf i i dt

T /2

e
T /2

if i t

dt

1 2 (f i ) lim T T

T /2 T

The results are of course identical. We will encounter this trick at various places when going from a transition amplitude to a transition rate. You may wonder why we need to consider a nite time interval T . The reason is that when we assume that the initial state is an eigenstate of the free Hamiltonian with xed momentum (or energy), then we have lost track of where a particle is in both space and time. A moving wave packet would see the static potential during a nite time, but the plane waves do not. Just like we will need to normalize the wave functions on a nite volume, we will need to normalize the potential to a nite time. A proper treatment is rather lengthy and relies on the use of wave packets. (See e.g. the book by K.Gottfried,

54

LECTURE 4. PERTURBATION THEORY AND FERMIS GOLDEN RULE

Quantum Mechanics (1966), Volume 1, sections 12, 56.) In the end, we can write transition probabilities in terms of plane waves, provided that we normalize to T and V . We discuss the normalization in more detail below.

4.3

Relativistic scattering

Fermis golden rule allows us to compute the scattering rate of non-relativistic particles on a static potential. In scattering experiments at high energies we need to deal with two scattering particles, rather than single particles scattering on a source. As an example, consider two spin-less electrons scatter in their mutual electromagnetic eld A , as depicted in Fig. 4.3.

C A e i D e f

e f

B e i

Figure 4.3: Scattering of two electrons in a electromagnetic potential.

As explained in the rst lecture, such scattering processes can be described by the exchange of virtual particles, Yukawas force carriers. Even without understanding the details of the interaction, we can readily identify one place where it should dier from the discussion above: the result must somehow encode four-momentum conservation and not just energy conservation. Our master formula for the dierential cross-section, Eq. (4.8) is essentially a generalization to problems with more than one particle in the initial or nal state. We cannot derive the expressions for a scattering cross section at high energies without going through the machinery of quantum eld theory. Instead, we will sketch the main results, then work through the electrodynamics of spin-less particles as an example in the next lecture. In quantum electrodynamics with scalar particles the transition amplitude T for the process A + B C + D still takes the form in Eq. (4.24). Performing the integral using incoming and outgoing plane waves = N eipx the result can be written as T = i NA NB NC ND (2 )4 4 (pA + pB pC pD ) M . (4.34)

4.3. RELATIVISTIC SCATTERING

55

where Ni are the plane wave normalization factors, which we will discuss shortly. The -function takes care of energy and momentum conservation in the process. (Note that the momentum vectors are four-vectors). The quantity M is called the invariant amplitude. It is computed using Feynman diagrams. For topologies with n particles (counting both incident and nal state), the dimension of M is p4n . The invariant amplitude does not depend on arbitrary time intervals T or normalization volumes V . To nd the transition probability we square the expression for T , |T |2 = |NA NB NC ND |2 |M|2 d4 x ei(pA +pB pC pD )x
pD )x d4 x ei(pA +pB pC (4.35)

= |NA NB NC ND |2 |M|2 (2 )4 4 (pA + pB pC pD ) lim

T,V T,V

d4 x
TV

(4.36) (4.37)

= |NA NB NC ND |2 |M|2 (2 )4 4 (pA + pB pC pD ) lim T V

Since we now have a -function over 4 dimensions (the four-momentum rather than just the energy), the integral becomes proportional to both T and V . To get rid of them we consider a transition probability per unit time and per unit volume: W |T |2 T,V T V = |NA NB NC ND |2 |M|2 (2 )4 (pA + pB pC pD ) lim

(4.38)

To use this result in our master formula, we now need to discuss a few remaining ingredients, namely the normalization of the wave functions, the ux factor and the phase space factor.

4.3.1

Normalisation of the Wave Function

Above we dened the eigenstates of the free Hamiltonian to have unit normalization. As we have seen in lecture 2 the eigenstates for free particles (for both the Schr odinger equation and the Klein-Gordon equation) are plane waves (x, t) = N ei(Etxp) . (4.39)

In contrast to wave packets the plane waves cannot be normalized over full space x (which further on leads to problems when computing the square of -functions as above). The solution is to apply so-called box normalization : we choose a nite volume V and normalize all wave functions such that (x, t) (x, t)d3 x = 1 .
V

(4.40)

56

LECTURE 4. PERTURBATION THEORY AND FERMIS GOLDEN RULE

For the plane waves this gives N = 1/ V . Like the time interval T , the volume V is arbitrary and must drop out once we compute an observable cross-section or decay rate. It should be noted that although we called W a transition probability per unit time and per unit volume, it is not actually a rate. Through the normalization factors Ni it still depends on the arbitrary volume V . These factors V will be canceled by those in the ux factor and phase space factors later on. For the classical wave function we had a density = | |2 so that the normalization gives one particle per unit volume. For the plane wave solutions of the Klein-Gordon equation, we had = 2|N |2 E , which with the box normalization becomes = 2E/V. (4.41)

In other words, in the relativistic case we have 2E particles per unit volume. Remember that the density is proportional to E in order to compensate for the Lorentz contraction of the volume element d3 x: we need the number of particles in a nite volume, d3 x, to remain remain constant in Lorentz transformations. With this result we are ready to compute the ux factor.

4.3.2

The Flux Factor

The ux factor or the initial ux corresponds to the number of particles that pass each other per unit area and per unit time. It can be most easily computed in a frame in which one of the particles is not moving. Consider the case that a beam of particles (A) is shot on a target (B ), see Fig. 4.4.

target

beam

Figure 4.4: A beam incident on a target.

The number of beam particles that pass through unit area per unit time is given by |vA | nA . The number of target particles per unit volume is nB . For relativistic plane E waves the density of particles n is proportional to = 2V such that ux = |vA | na nb 2|pA | 2mB V V (4.42)

4.3. RELATIVISTIC SCATTERING

57

(Remember that in relativity v = p/E , modulo a factor c. For the KG waves we had indeed that the current density was j = p/E .) In exercise 4.1 you will show that the kinematic factor |pA |mB is actually Lorentz invariant and that this expression can be rewritten as ux = 4
2 2 2 2 (pA, p B ) mA mB / V

(4.43)

The volume factor is not Lorentz invariant, but it will drop out later. Note that the incident ux as dened here is not actually a certain number of particles per unit surface per unit time per unit volume: we need to account for the fact that it is proportional to the square of an energy. The factors of energy will be accounted for by the other ingredients to the cross-section formula.

4.3.3

The Phase Space Factor

In the nal step to Fermis golden rule we introduced the density of nal states (E ). The phase space factor accounts for the number of accessible nal states. It depends on the volume V and on the momentum p of each nal state particle. Consider a cross-section measurement in which we measure the 3 components (px , py , pz ) of the momenta of all nal state particles. As stated above it is customary to express the cross-section as a dierential cross-section to the nal state momenta, d = . . .
f

d3 pf

(4.44)

where the product runs over all nal state particles. To compute an actual number for our experiment, we now convolute with experimental resolutions and integrate over eventual particles or momentum components that we do not measure. (For example, we often just measure the number of particles in a solid angle element d .) For the dierential cross-section the question of the number of accessible states should then be rephrased as how many states t in the momentum-space volume V d3 p. Assume that our volume V is rectangular with sides Lx , Ly , Lz . Using periodic boundary conditions to ensure no net particle ow out of the volume we need to require that Lx px = 2 nx with nx integerer. Hence, the total number of states in the range px to px + dpx is dnx = Lx dpx /2 . Since the total number of available states is N = nx ny nz , we nd that the number of states with momentum between p and p + dp (i.e. between (px , py , pz ) and (px + dpx , py + dpy , pz + dpz ) ) is: V d3 p . dN = (2 )3 (4.45)

A mildly alternative view is given by Burcham & Jobes on page 305. The number of nal states is given by the total size of the available phase space for the nal state divided

58

LECTURE 4. PERTURBATION THEORY AND FERMIS GOLDEN RULE

L n= x

Lz Lx

2 1

Ly

Figure 4.5: Schematic calculation of the number of states in a box of volume V .

by the volume of the elementary cell, h3 (within an elementary cell states cannot be distinguished):

N=

1 h3

dx dy dz dpx dpy dpz =

V (2 )3

d px d py d pz .

(4.46)

leading to the same result. As explained above, in the relativistic case the wave functions are normalized such that the volume V contains 2E particles. Therefore, the number of states per particle is:

# states/particle =

V d3 p (2 )3 2E

(4.47)

For a process in the form A + B C + D + E + .... with N nal state particles the Lorentz invariant phase space factor then becomes

d = dLIPS =
f =1

V d 3 pf . (2 )3 2Ef

(4.48)

In exercise 4.2 you will show that (ignoring V ) the phase space factor is Lorentz invariant. We will omit the factors in what follows.

4.3. RELATIVISTIC SCATTERING

59

4.3.4

Golden rules for cross-section and decay

Putting this all together, we arrive at the formula to calculate a cross section for the process Ai + Bi Cf + Df + ...: df i = T W 1 W d ux 1 = d4 x f (x) V (x) i (x) i |T |2 = lim V,T T V
N

(4.49)

d =
f =1

V d 3 pf (2 )3 2Ef
2 2 (pA pB )2 m2 A mB / V

ux = 4

In exercise 4.3 you will show that the cross-section is indeed independent on the volume V. Inserting the expression for the transition rate per unit time and volume, Eq. (4.38), we nd for the dierential cross-section of the process A + B C + D d = (2 )4 4 (pA + pB pC pD ) 4 ( pA pB )
2 2 m2 A mB

|M|2

d 3 pC d3 pD (2 )3 2EC (2 )3 2ED

(4.50)

Note that the integrals of the ux factors are only over the spatial part of the outgoing four-momentum vectors. The energy component has been integrated out, using the fact that the outgoing particles are on the mass shell. Therefore, Ef is not an independent variable, but equal to phase space. |pf |2 + m2 f . This is important when performing integrals over

In exercise 4.4 we calculate the integrals and ux factors in the centre-of-momentum system, where pA + pB = pC + pD = 0. The result is d d =
cm

1 |pf | |M|2 2 64 s |pi |

(4.51)

where we dened pi pA = pB , pf pC = pD and s = (EA + EB )2 . The computation of a decay rate for the process A C + D follows a similar strategy. The result for the partial decay rate is (2 )4 4 (pA pC pD ) d3 pC d3 pD d = |M|2 2EA (2 )3 2EC (2 )3 2ED (4.52)

60

LECTURE 4. PERTURBATION THEORY AND FERMIS GOLDEN RULE

which after integration of one of the momenta gives (4pi s 2EA = 2mA ) d d =
cm

1 |pf | |M|2 2 2 32 mA

(4.53)

Exercises
Exercise 4.1 (Lorentz invariance of the ux) Prove that (ignoring transformations of the volume V ) the ux factor derived in the lab frame in Eq. (4.42) is indeed Lorentz-invariant by proving the identity
2 (pA pB )2 m2 A mB = |pA |mB

(4.54)

Hint: Since the left hand side is Lorentz invariant, you can compute it in any frame. Note that pA pB is an inner product of the four-vectors, not the three-vectors. Exercise 4.2 (Lorentz invariance of the phase space factor) Show that for any Lorentzinvariant function M (p) of p, we have the identity M (p) 2d4 p (p2 m2 ) (p0 ) = where (p0 ) is the Heavyside function and E = d3 p 2E is Lorentz invariant. Exercise 4.3 (Box volume is arbitrary) Show that the cross-section does not depend on the arbitrary size of the volume V : identify all places where factors V enters in the summary in Eq. (4.49) and show that they cancel. Exercise 4.4 (AB CD cross-section in the cms. See also H&M, Ex. 4.2) In this exercise we derive a simplied expression for the A + B C + D cross-section in the center-of-momentum frame. (a) Start with the expression: d = d 3 pD d 3 pC (2 ) (pA + pB pC pD ) (2 )3 2EC (2 )3 2ED
4 4

M (p)

d3 p . E

(4.55)

m2 + p2 . Argue that this means that (4.56)

(4.57)

Do the integral over d3 pD using the function and show that we can write: d = 1 p2 f dpf d (EA + EB EC ED ) (2 )2 4EC ED (4.58)

4.3. RELATIVISTIC SCATTERING

61

where we have made use of spherical coordinates (i.e.: d3 pC = |pC |2 dpC d ) and dened pf |pC |. (b) In the C.M. frame we have |pA | = |pB | = pi and |pC | = |pD | = pf . Furthermore, in this frame s |pA + pB | = EA + EB W . Show that the expression becomes (hint: calculate dW/dpf ): d = 1 pf (2 )2 4 1 EC + ED dW d (W EC ED ) (4.59)

So that we nally get: d = 1 pf d 4 2 4 s (4.60)

(c) Show that the ux factor in the C.M. frame is: F = 4pi s

(4.61)

and hence that the dierential cross section for a 2 2 process in the center-ofmomentum frame is given by d d =
cm

1 pf |M|2 2 64 s pi

(4.62)

Exercise 4.5 (Important representations of the -function) (a) The delta-function can have many forms. One of them is: (x) = lim 1 sin2 x x2 (4.63)

Make this plausible by sketching the function sin2 (x)/(x2 ) for two relevant values of . (b) Remember the Fourier transform, f ( x) = g (k ) =

1 2

g (k ) eikx dk
+

(4.64) f (x) e
ikx

dx

Use this to show that another (important!) representation of the Dirac delta function is given by + 1 (x) = eikx dk (4.65) 2

62

LECTURE 4. PERTURBATION THEORY AND FERMIS GOLDEN RULE

Lecture 5 Electromagnetic Scattering of Spinless Particles


In this lecture we discuss electromagnetic scattering of spin-0 particles. First we compute the scattering of a charged particle on a static point charge. We show that in the nonrelativistic limit the result is in agreement with the well known formula for Rutherford scattering. Subsequently, we derive the cross section for two particles that scatter in each others eld. We end the lecture with a prescription on how to treat antiparticles.

5.1

Electromagnetic current

As we discussed in section 3.4 the laws of electrodynamics can be introduced in the equations of motions of free particles by the method of minimal substitution, p p qA , which in terms of operators in coordinate space takes the form + iqA . (5.2) (5.1)

Now consider a spinless particle with mass m and charge e scattering in a vector eld A , as in gure 4.1. (It is conventional to consider a charge e as for a hypothetical spin-0 electron.) The wave equation for the free particle is the Klein-Gordon equation, + m2 = 0 Substituting + iqA with q = e, we obtain ( ieA ) ( ieA ) + m2 = 0. 63 (5.4) (5.3)

64LECTURE 5. ELECTROMAGNETIC SCATTERING OF SPINLESS PARTICLES Be aware that the operators act on all eld on their right, so both on and A . This equation can be rewritten as + m2 + V (x) = 0 with a perturbation potential V (x) given by V (x) ie ( A + A ) e2 A2 . (5.6) (5.5)

The sign of V is chosen such that compared to the kinetic energy it gets the same sign as in the Schr odinger equation, Eq. (3.39). Since e2 is small ( = e2 /4 = 1/137) and we only consider the Born level cross-section, we neglect the second order term, e2 A2 0. From the previous lecture we take the general expression for the transition amplitude in the Born approximation and insert the expression for V (x), Tf i = i = i d4 x f (x) V (x) i (x)
d4 x f (x) (ie) (A + A ) i (x).

(5.7)

Note that the second operator on the right hand side acts on both A and . However, we can use integration by parts to write
d4 x f (A i ) = f A i

4 f A i d x

(5.8)

Requiring the eld to be zero at t = , the rst term on the left vanishes, such that we can rewrite the transition amplitude as Tf i = i
4 (ie) f (x) ( i (x)) f (x) i (x) A d x .

(5.9)

In this expression the derivatives no longer act on the eld A . Remember the denition of the charge current density for the Klein-Gordon eld of the electron, Eq. (2.42) j = (ie) [ ( ) ( ) ] . In complete analogy we dene the electromagnetic transition current to go from initial state i to nal state f as
fi j (ie) f ( i ) f i .

(5.10)

You may verify that if f and i are both solutions to a Klein-Gordon equation with fi mass m, then also this current satises the continuity equation j = 0. The transition amplitude can now be written as Tf i = i
fi 4 j A dx

(5.11)

5.2. COULOMB SCATTERING

65

This is the expression for the transition amplitude for going from free particle solution i to free particle solution f in the presence of a perturbation caused by an electromagnetic eld. Restricting ourselves to plane wave solutions of the unperturbed Klein-Gordon equation, ipf x i = Ni eipi x and , (5.12) f = Nf e we nd for the transition current of spinless particles
i(pf pi )x f fi . pi = (e) Ni Nf j + p e

(5.13)

Inserting this in the transition amplitudes gives Tf i = i


i(pf pi )x 4 dx (e) Ni Nf p i + pf A e

(5.14)

5.2

Coulomb scattering

Consider the case that the external eld is a static eld of a point charge Ze located at the origin, Ze A = (V, A) = (V, 0) with V (x) = . (5.15) 4 |x| With a vector eld of this form, we have p k A = Ek V (x). Consequently, we nd for the transition amplitude Tf i = i
Ni Nf (Ei + Ef ) ei(pf pi )x

Ze2 4 dx 4 |x|

(5.16)

Since V (x) is not time-dependent, we split the integral over space and time. As we have seen before the integral over time turns into a function, expressing energy conservation, ei(Ef Ei )t dt = 2 (Ef Ei ) For the integral over x we use an important Fourier transform 1 = |q |2 Using this with q (pf pi ) we obtain
Tf i = ieNi Nf (Ei + Ef ) 2 (Ef Ei )

(5.17)

d3 x eiqx

1 4 |x|

(5.18)

Ze . |pf pi |2

(5.19)

As we have seen before for a time-independent potential, we consider a time-averaged transition rate, |Tf i |2 Wf i = lim (5.20) T T

66LECTURE 5. ELECTROMAGNETIC SCATTERING OF SPINLESS PARTICLES where the time-averaging eectively takes care o one of the functions when taking the square of the amplitude. The result is Wf i = |Ni Nf | 2 (Ef Ei )
2

Ze2 (Ei + Ef ) |pf pi |2

(5.21)

Working with normalization of the plane waves over a box with volume V , we have |Ni Nf |2 = V 2 . The ux factor for a single particle is given by Flux = |vi | while the phase space factor is dLips = V d3 pf . (2 )3 2Ef (5.23) 2Ei 2|pi | = , V V (5.22)

Inserting these expressions in our master formula for the cross-section, Eq. (4.8), we nd 2 (Ef Ei ) d = 2|pi | where we have canceled all factors V . We can still simplify this by integrating over the outgoing momentum. Choose the z -axis along pi and switch to polar coordinates for pj such that
2 d3 pf = p2 f dpf dcos d = pf dpf d

Ze2 (Ei + Ef ) |pf pi |2

d 3 pf (2 )3 2Ef

(5.24)

(5.25) (5.26)

and |pf pi |2 = 2pf pi (1 cos ) = 4pf pi sin2 /2.


2 Since Ef = m2 + p2 f , we have pf dpf = Ef dEf , and therefore,

(Ef Ei ) dpf = The dierential cross-section then becomes

Ef (Ef Ei ) dEf pf

(5.27)

d Z 2 Ei2 e4 Z 2 |Ei |2 2 = = d 64 2 |pi |4 sin4 /2 4 |pi |4 sin4 /2

(5.28)

where we dened e2 /4 . In the non-relativistic limit we have E m and p2 = 2mEkin , giving d Z 2 2 = , (5.29) 2 d 16Ekin sin4 /2 which is the well-known Rutherford scattering formula.

5.3. SPINLESS K SCATTERING

67

Above we have not explicitly shown the solution of the wave function itself. Without deriving it here, we just state that for a spherically symmetric potential, V (r ) = V (r), and an incident wave with momentum along the z -axis, the wave function for large r takes the form eikr r ikz (r ) e + f (, ) (5.30) r where the rst term is the incident unscattered wave and the second term the scattered wave. Expressed in terms of the f (, ), the dierential cross-section can be written as d = |f (, )|2 (5.31) d The interference between the scattered and the unscattered wave leads to a shadow behind the scattering potential. The ux that missing in the shadow is exactly the total scattered ux. This is expressed in the optical theorem which states that the total cross-section is proportional to value of f in the forward direction, k . 4 See also appendix H of Aichison and Hey, and references therein. Imf (0) = (5.32)

5.3

Spinless K Scattering

We now proceed with the electromagnetic scattering of two particles, A + B C + D. As an example we consider the scattering of a particle and a K particle. We ignore the fact that pions and kaons also are subject to the strong interaction, which is ne as long as the recoil momentum is small compared to the binding energy. We could equally well consider a process like e scattering, provided that we ignore the lepton spin. For the computation presented here, the essential restrictions are that the incident particles carry no spin and that they are of dierent type. B : K D : K

A :

C :

Figure 5.1: Leading order diagram for electromagnetic scattering of a charged kaon and a charged pion.

We have seen above how a particle scatters in an external eld. In this case the eld is not external as the particles scatter in each others eld. How do we deal with this?

68LECTURE 5. ELECTROMAGNETIC SCATTERING OF SPINLESS PARTICLES First consider a pion scattering in the vector eld A generated by the current of the kaon. The transition current of the kaon is given by (see Eq. (5.13))
i(pD pB )x jBD = eNB ND (p B + pD ) e

(5.33)

We now assume that the eld generated by the kaon can be computed by inserting this current in the Maxwell equations for the vector potential, i.e.
A = jBD

(5.34)

where we have adopted the Lorentz gauge. (A proof that this indeed works requires the full theory.) Since eiqx = q 2 eiqx , we can easily verify that the solution is given by A = 1 j q 2 BD , (5.35)

where we dened q = pD pB . Note that the latter corresponds to the four-momentum transfered by the photon from the kaon to the pion. Tf i = i
jAC A d4 x = i jAC

1 BD 4 j d x = i q2

jAC

g jBD d4 x 2 q

(5.36)

Four-momentum conservation (which appears as a result of the integral) makes that the momentum transfer is also equal to q = (pC pA ). Therefore, Tf i is indeed symmetric in the two currents. It does not matter whether we scatter the pion in the eld of the kaon or the kaon in the eld of the pion. Also note that the expression has a pole for q 2 = 0, the mass of a real photon: zero momentum transfer (non-scattered waves) has innite probability. The only contribution to scattering under non-zero angles comes from photons that are o the mass-shell. We call these virtual photons. Inserting the plane wave solutions Tf i = ie2
i(pC pA )x (NA NC ) (p A + pC ) e

1 i(pD pB )x 4 (NB ND ) (p d x (5.37) B + pD ) e 2 q

and performing the integral over x we obtain 1 D (NB ND ) pB (2 )4 4 (pA + pB pC pD ) + p 2 q (5.38) where the -function that takes care of four-momentum conservation appears. Usually this is written in terms of the invariant amplitude M (sometimes called matrix element) as Tf i = i NA NB NC ND (2 )4 4 (pA + pB pC pD ) M (5.39)
Tf i = ie2 (NA NC ) (p A + pC )

with the invariant amplitude given by iM = ie (pA + pC )


vertex factor propagator

ig ie (pB + pD ) . 2 q
vertex factor

(5.40)

5.3. SPINLESS K SCATTERING

69

The signs and factors i are assigned such that the expressions for vertex factors and propagator are also appropriate for higher orders. These are in fact, our rst set of Feynman rules! B ie(pB + pD )
ig q2

ie(pA + pC )

Figure 5.2: Feynman rules for the t-Channel contribution to electromagnetic scattering of spinless particles.

Feynman rules allow us to specify the amplitude corresponding to a particular Feynman diagram without going through the explicit computation of the amplitude in quantum eld theory. (The rules are obtained from the full theory.) In gure 5.2 we illustrate the rules for the diagram that we are considering. The invariant amplitude contains: a vertex factor: for each vertex we introduce the factor: iep , where: e is the intrinsic coupling strength of the particle to the e.m. eld. p is the sum of the 4-momenta before and after the scattering (remember the particle/anti-particle convention). a propagator: for each internal line (photon) we introduce a factor q is the 4-momentum of the exchanged photon quantum.
ig , q2

where:

The ingoing and outgoing four-momenta, and the four-momenta of internal particles are free, but we are also required to add a function at each vertex to ensure energymomentum conservation. In the end, all internal momentum vectors are integrated over, and what remains is a single function over ingoing and outgoing momenta. By convention, the latter does not belong to M. The full set of rules also specify how this works for higher order diagrams, but we delay that for later discussion. We can now insert the invariant amplitude into the expression for the A + B C + D cross-section that we derived in the previous lecture, d = (2 )4 4 (pA + pB pC pD ) 4 (pA pB )
2 2 m2 A mB

|M|2

d3 pC d 3 pD . (2 )3 2EC (2 )3 2ED

(5.41)

In the centre-of-momentum frame (pA = pB ) this expression became d 1 1 pf = |M|2 . 2 d 64 s pi where s = (pA + pB )2 . (5.42)

70LECTURE 5. ELECTROMAGNETIC SCATTERING OF SPINLESS PARTICLES Finally, consider the limit of massless particles. Dene p pA and p pC . In the centre-of-momentum frame the four-vectors are given by p A p B p C pD = (|p|, p) = (|p|, p) = (|p |, p ) = (|p |, p )

Dene p |p| which, by four-vector conservation is also equal to |p |. Dene as the angle between pA and pC , which means that cos = pA pC /|pA ||pC | = pA pC /p2 . We then have (pA + pC ) g (pB + pD ) = (pA ) (pB ) + (pA ) (pD ) + (pC ) (pB ) + (pC ) (pD ) = 2p2 + p2 (1 + cos ) + p2 (1 + cos ) + 2p2 = p2 (6 + 2 cos ) Likewise, we get for q 2 , q 2 = (pA pC )2 2 = p2 A + pC 2(pA ) (pC ) = 2p2 (1 cos ) Consequently, we obtain for the invariant amplitude dened above M = e2 p2 (6 + 2 cos ) = e2 2p2 (1 cos ) 3 + cos 1 cos . (5.43)

Inserting this in Eq. (5.42) gives (with = e2 /4 ), d 1 1 2 = e d 64 2 s


2

3 + cos 1 cos

2 = 4s

3 + cos 1 cos

(5.44)

This is the leading order QED cross section for the scattering of massless spin-0 particles in the centre-of-momentum frame. In the exercises you will derive the formula for particles with non-zero mass.

5.4

Feynman calculus: propagators and vertex factors

Above we expressed the invariant amplitude in two vertex factors for the particlephoton-particle points in the Feynman diagram and one propagator for the photon-line connecting the two vertices. The expression of the amplitude in terms of propagators and vertex factors is part of what we refer to as the Feynman calculus. To derive the validity

5.4. FEYNMAN CALCULUS: PROPAGATORS AND VERTEX FACTORS

71

of the Feynman calculus approach required eld theory. However, the attractiveness of this approach is that once you have established the recipe, you can derive the Feynman rules (the expressions for the propagators and vertices) directly from the Lagrangian density that species the dynamics of your favourite theory: if you insert a new type of particle or interaction in your Lagrangian, you do not really need eld theory anymore to compute cross-sections. For a reminder of what a Lagrangian is, see your classical mechanics course, or the short addendum at the end of this section. We discuss the Lagrangian density in more detail in Lecture 10. However, because it is a small omission in the rest of these lectures, we will now very briey sketch how such Feynman rules are extracted from the Lagrangian. To extract the propagators for a certain type of particle, one takes the part of the Lagrangian that species the free particle, derives its wave equation using the EulerLagrange equations and then takes the inverse of the wave equation in momentum space, multiplied by i. For instance, for a complex scalar (spin-0) eld, the Lagrangian density is L = ( )( ) m2 (5.45) The corresponding Euler-Lagrange equations (obtained by requiring the action to be stationary, see Lecture 10) lead to the Klein-Gordon equation that we encountered in Lecture 2, + m2 (x) = 0 (5.46) In momentum space (p i ) this reads p p m2 (p) = 0 The resulting propagator for the eld with momentum p is then propagator = i p p m2 (5.48) (5.47)

You cannot directly use this result for a spin-1 particle like the photon, but you will notice that the result for the photon propagator (ig /p2 ) resembles this result closely in the limit m = 0. (The propagator for spin-1 particles with non-zero mass is called the Proca propagator and we will see it in Lecture 11.) Interactions in the Lagrangian density take the form of terms that have elds of dierent types. You cannot just write down arbitrary terms: the requirement that they be Lorentz invariant restricts their form. For instance, the interaction term for the photon with the spin-0 eld that we have seen above, takes the form Lint = gj A (5.49)

where g is the coupling constant (e for the EM interaction) and j is the current for the complex scalar eld j i [ ( ) ( ) ] . (5.50)

72LECTURE 5. ELECTROMAGNETIC SCATTERING OF SPINLESS PARTICLES It is important that and are dierent elds here: in our discussion above one would be the incoming eld and the other the outgoing eld. The interaction term then has three elds and leads to a vertex with three lines. Labeling by A and by C , we now express the current in momentum space,
jAC = i [ C (pA A ) + (pC C ) A ]

(5.51)

(Note how the plus sign appears: it is because p i .) The rule to obtain the vertex factor is now to omit all the elds from the interaction term and write down what is left, multiplied by i, vertex factor = ig (p (5.52) A + pC ) This summarized the recipe to get the vertex factors and propagators. It directly leads to the Feynman rules for a particular Lagrangian. Using the rules we can now compute the amplitude M for any Feynman diagram.

5.5

Form factors

In the previous sections we studied point charges, objects with their charge located in an innitely small region. If the charge distribution has a nite size, the dierential crosssection is dierent from that of a point source. Consequently, the measured dierential cross-section can tell us important information over the substructure of particles. For example, most information about the structure of the proton has been obtained in electron-proton scattering experiments, most notably at the Hera collider in Hamburg. Consider a static source with a charge distribution Ze(x), normalized so that (x)d3 x = 1 (5.53)

By following the same procedure as above for the static source, one can show that the dierential cross-section can be written as d d = |F (pi pf )|2 d d point (5.54)

where F (q ) is called the form factor and just given by the Fourier transform of the charge distribution F (q ) = (x)eiqx d3 x (5.55)

In real electron-proton scattering we also need to account for the spins and the magnetic moment of the proton. The form factor will then become more complicated. You will learn more about this in the Particle Physics II course.

5.6. PARTICLES AND ANTI-PARTICLES

73

5.6

Particles and Anti-Particles

We have seen that the negative energy state of a particle can be interpreted as the positive energy state of its anti-particle. How does this eect energy conservation that we encounter in the -functions? We have seen that the invariant amplitude has the form of: M Let us examine four cases. Scattering of an electron and a photon: M pi pf k Energy and momentum conservation are guaranteed by the -function. Scattering of a positron and a photon: Replace the anti-particles always by particles by reversing (E, p E, p) such that now: incoming state = pf , outgoing state = pi : pf k M = ei(pi )x

f (x) V (x) i (x) dx

eipf x

eikx eipi x dx

ei(pi +kpf )x dx

= (2 )4 (Ei + Ef ) 3 (pi + k pf )

pi

eikx ei(pf )x dx

ei(pi pf +k)x dx

= (2 )4 (Ei + Ef ) 3 (pi + k pf ) Electron positron pair production: -p+ k p

M =

eip x

ei(p+ +k)x dx

ei(kp+ p )x dx

= (2 )4 (k p p+ )

74LECTURE 5. ELECTROMAGNETIC SCATTERING OF SPINLESS PARTICLES Electron positron annihilation: p k -p+

M =

ei(kp+ )x

ei(p )x dx

ei(p +p+ k)x

= (2 )4 (p + p+ k )

Addendum: The principle of least action


In classical mechanics the equations of motion can be derived using the variational principle of Hamilton which states that the action integral S should be stationary under arbitrary variations of the so-called generalized coordinates qi , q i . For a pedagogical discussion of the principle of least action read the Feynman lectures, Vol.2, chapter 19. Generalized coordinates are coordinates that correspond to the actual degrees of freedom of a system. For example, take a swinging pendulum in two dimensions. We could describe the movement of the weight of the pendulum in terms of both its horizontal coordinate x and its vertical coordinate y . However, only one of those is independent since the length of the pendulum is xed. Therefore, we would say that the movement of the pendulum can be described by one generic coordinate. We could choose x, but also the angle of the pendulum with the vertical axis (usually called the amplitude). We denote generalized coordinates with the symbol q and call the evolution of q with time a trajectory or path. Dening the Lagrangian of the system as the kinetic energy minus the potential energy, L = T (q ) V (q ) (5.56)

(where the potential energy only depends on q and the kinetic energy only on q ), we denote the action (or action integral) of a path that starts at t1 and ends at t2 with
t1

S (q ) =
t0

L(q, q )dt.

(5.57)

Hamiltons principle now states that the actual trajectory q (t) followed by the system is the trajectory that minimizes the action. You will derive later in this course that for each of the coordinates qi , this leads to the so-called Euler-Lagrange equation of motion L d L = . dt q i qi (5.58)

5.6. PARTICLES AND ANTI-PARTICLES This may also be written more symmetrically as Hamiltons equations, p i = L qi with pi = L , q i

75

(5.59)

where pi is called the generalized momentum, or sometimes the momentum canonical to qi . In terms of these coordinates, the Hamiltonian takes the form H =
i

pi q i L.

(5.60)

Finally, the classical system can be quantized by imposing the fundamental postulate of quantum mechanics, [qi , pj ] = i ij . (5.61) The formulation of physics laws in terms of Euler-Lagrange equations derived from a Lagrangian (or Lagrangian density, for elds) plays a fundamental role in particle physics. As we shall see in chapter 10, any symmetry of the Lagrangian leads to a conserved quantity. (This is Noethers theorem. As an example, U (1) symmetry leads to the conserved current for the Klein-Gordon and Dirac elds.) Furthermore, all interactions in the standard model can be constructed by requiring so-called local gauge-symmetries of the Lagrangian.

Exercises
Exercise 5.1 (Scattering o a static source as a limit) Above we derived the expression for A + B C + D scattering in the special case that A, B , C and D are massless. We will now show that in the limit in which mB mA and mB p the cross-section for the process A + B A + B reduces to the formula that we derived for scattering of a static source. Call mA the mass of A and mB the mass of B . We consider scattering in the CMS. Choose a coordinate system such that the initial momenta are along the z -axis, with A going in the positive z direction and B in the negative z direction. Label the outgoing particles with a prime. Call the polar angle of pA . Momentum conservation makes that all momenta have the same size, which we label by p. (a) Express (pA + pA ) (pB + pB ) in mA , mB , p and cos . (You may of course also use 2 2 m2 EA = m2 A + p and EB = B + p .) Hint: one method is to rst just write down all fourvectors in these symbols. Take the x, y -coordinates together, because we do not care about the azimuthal angle ().Now you can either rst add and them take the inner product, or vice versa. (b) Do the same for q 2 = (pA pA )2 and for s = (pA + pB )2 . (c) Write down the dierential cross-section d/d using Eq. (5.42). Note that this result more general than our massless particle result in Eq. (5.44).

76LECTURE 5. ELECTROMAGNETIC SCATTERING OF SPINLESS PARTICLES (d) Take the limit mB p and mB static source, Eq. (5.28). mA . Compare to the result for scattering of a

Exercise 5.2 (Propagator for a massive photon) Suppose the photon had mass. If youd follow a similar approach as the one that led us to the propagator for a massless photon in section 5.3, what would be the propagator for a massive photon? Hint: Look at how we arrived at the photon propagator in section 5.3. Add the mass to the equation that species A and derive how the propagator is modied. If you do it this way, the result is not actually quite right, but good enough for your homework. We will see the real propagator for spin-1 particles with non-zero mass later in the course. Exercise 5.3 (Equal particles) When computing the scattering of two particles A and B above, we explicitly restricted the computation to the case where the particles were dierent. (a) What changes if B is an anti-A? (b) What changes if B is an A? Hint: Look at gure 5.2. Can you imagine additional leading order diagrams? If so, draw them! Exercise 5.4 (Decay rate of 0 (see also Griths, ex.6.6)) (a) Write down the expression for the total decay width for the decay: A C + D (b) Assume that particle A is a 0 particle with a mass of 140 MeV and that particles C and D are photons. Draw the Feynman diagram for this decay (i) assuming the pion is a uu state. state. (ii) assuming the pion is a dd (c) For the invariant amplitude we have: M f e2 , where for the decay constant we insert f = m . (i) Where does the factor e2 come from? (ii) What do you think is the meaning of the factor f ? Describe it qualitatively. wave with 3 colour degrees of freedom. (d) The 0 is actually a uu + dd (i) Give the expression for the decay width. (ii) Calculate the decay width expressed in GeV. (iii) Convert the rate into a mean lifetime in seconds. (iv) How does the value compare to the Particle Data Group (PDG) value? Remark: Do not be disappointed if your prediction is completely wrong! It turns out that the 0 lifetime is quite hard to compute.

Lecture 6 The Dirac Equation


It is sometimes said that Schr odinger discovered the Klein-Gordon equation before the equation bearing his own name, but that he rejected it because it was quadratic in /t. In Lecture 2 we have seen how the Klein-Gordon equation leads to solutions with negative energy and negative probability density. In 1928 in an attempt to avoid this problem Dirac developed a relativistic wave equation that is linear in /t. Lorentz invariance requires that such a wave equation is also linear in . What Dirac found, to his own surprise, was an equation that describes particles , just what was needed for electrons. We now know that all fundamental with spin- 1 2 fermions are described by this wave equation. Dirac also predicted the existence of antiparticles, an idea that was not taken seriously until 1932, when Anderson discovered the positron.

6.1

Spin, spinors and the gyromagnetic ratio

Before we proceed with the derivation of the Dirac equation, we briey discuss spin and the Pauli-Schr odinger equation, since this allows us to introduce some important concepts. A lump of charge rotating around an axis through its centre-of-gravity is a magnetic dipole. It has a magnetic moment = S (6.1) where S is the classical spin, the integral of r p over the mass density distribution. The factor is called the gyromagnetic ratio. If the charge distribution follows the mass distribution, this ratio is given by classic = 77 q . 2m (6.2)

78

LECTURE 6. THE DIRAC EQUATION

When placed in a magnetic eld B , the particle experiences a torque, B . The potential energy associated with the torque is H = B = SB (6.3)

So, far this is just classical electrodynamics. The classical spin S is nothing but the total angular momentum of all bits and pieces that the particle is made up from. However, as you remember from your quantum mechanics course, elementary particles also carry intrinsic spin. Though we sometimes imagine it as a result of a charged particle spinning around an axis, this interpretation actually falls short. In particular, the prediction of the gyromagnetic ratio that would come out of this picture is wrong. On the other hand, elementary particles do feel a torque in a magnetic eld (as demonstrated in the Stern-Gerlach experiment) and the energy-levels are proportional to the spin orientation as described above. So, in 1927 Pauli tried to address the question of how to describe their magnetic moment in quantum mechanics. system. As you know, such a system has two values for the Pauli considered a spin- 1 2 eigenvalue of spin, namely 1 . An arbitrary spin wave function is a superposition of 2 the two eigenstates. Pauli represented it as a two-component vector, called a spinor, = where the basis vectors + 1 0 and 0 1 (6.5) a b = a+ + b (6.4)

represent the spin-up and spin-down state respectively. The hermitian operator that measures spin, the spin operator S , satises the same algebra as for orbital angular momentum in quantum mechanics, namely [Si , Sj ] = i ijk Sk . In the basis above, S is represented by 2x2 matrices. Choosing the z -axis as the quantization axis, S is given by S = where the Pauli spin matrices are 1 = 0 1 1 0 2 = 0 i i 0 3 = 1 0 0 1 . (6.7) 2 (6.6)

Note that the i all have zero trace, are hermitian, and satisfy i j = ij + i
ijk k

(6.8)

which implies as well that they anti-commute ({i , j } = 2ij ).

6.1. SPIN, SPINORS AND THE GYROMAGNETIC RATIO

79

In a modern interpretation, Pauli proposed to represent the momentum operator in spinor space by p p (6.9) Take a careful look at what is written on the right-hand-side: it is an inner product of a vector of 2x2 matrices with the momentum operator. The result is again a 2x2 matrix, which is more apparent when written out, p= pz px ipy px + ipy pz . (6.10)

You will show in an exercise that ( p)2 = p2 1 12 , where 1 12 is the 2x2 identity matrix. Therefore, the Schr odinger equation for free spinors is just the ordinary Schr odinger equation, p2 d 1 a a a 2 = i = ( p) (6.11) b b dt 2m 2m b and the two spin states are degenerate in energy. This is no longer the case if we introduce the vector eld. Using again minimal substitution, the Hamiltonian (a matrix in spinor space) for a particle in a vector eld (V, A) becomes 1 H= [ (p q A)]2 + qV (6.12) 2m It is a not entirely trivial exercise to show1 that this equation can be rewritten as H= q 1 (p q A)2 + qV B 2m 2m (6.13)

The Schr odinger equation with this Hamiltonian is called the Pauli-Schr odinger equation, or simply Pauli equation. Comparing this Hamiltonian to the Hamiltonian of the classical spin, we nd that the gyromagnetic ratio for a spin- 1 particle is 2 spin-1/2 = q m (6.14)

exactly a factor 2 larger than for the classical picture of a spinning charge distribution. The ratio of the magnetic moment relative to that of the classical case is called the g -factor. So for spin- 1 particles the Schr odinger-Pauli equation predicts g = 2. In 2 QED the magnetic moment is modied by higher order corrections. The predictions and measurements of the magnetic moment of the electron and muon are so precise that they make QED the most precisely tested theory in physics. Pauli introduced the spin matrices in the Hamiltonian on purely phenomenological grounds. As we shall see in the rest of this Lecture and the next, Dirac found a theoretical motivation: His construction of a wave equation that is linear in space and time
1 Take into account that p = i and A do not commute, then use the chain rule to see that in coordinate space only a term B = A survives.

80

LECTURE 6. THE DIRAC EQUATION

derivatives, leads (in its simplest form) to the description of spin- 1 particles and anti2 particles. As you will prove in an exercise in the next Lecture, the Pauli-Schr odinger equation can be obtained as the low relativistic limit of the equation of motion of Dirac particles in a vector eld.

6.2

Dirac equation

In order to appreciate what Dirac discovered we follow (a modern interpretation of) his approach leading to a linear wave equation. (For a dierent approach, which may be closer to what Dirac actually did, see Griths, 7.1.) Consider the usual form of the Schr odinger equation, (6.15) i = H . t The classical Hamiltonian is quadratic in the momentum. Dirac searched for a Hamiltonian that is linear in the momentum. We start from the following ansatz 2 : H = ( p + m) (6.16)

with coecients 1 , 2 , 3 , . In order to satisfy the relativistic relation between energy and momentum, we must have for any eigenvector (p) of H so that H 2 = p2 + m2 (6.17)

where p2 + m2 is the eigenvalue. What should H look like such that these eigenvectors exist? Squaring Diracs ansatz for the Hamiltonian gives H2 =
i

i pi + m
j

j pj + m i pi m +
i i

=
i,j

i j pi pj +
2 2 i pi + i i>j

i pi m + 2 m2 (i + i )pi m + 2 m2
i

(6.18)

(i j + j i )pi pj +

where we on purpose did not impose that the coecients (i , ) commute. In fact, comparing to equation (6.17) we nd that the coecients must satisfy the following requirements:
2 2 2 1 = 2 = 3 = 2 = 1

1 , 2 , 3 , anti-commute with each other.


2

We take p = x px + y py + z pz .

6.3. COVARIANT FORM OF THE DIRAC EQUATION With the following notation of the anti-commutator {A, B } = AB + BA. we can also write these requirements as {i , j } = 2ij {i , } = 0 2 = 1 .

81

(6.19)

(6.20)

Clearly, the i and cannot be ordinary numbers. At this point Dirac had a brilliant idea, possibly motivated by Paulis picture of fermion wave functions as spinors: what if the i and are matrices that act on a wave function that is a column vector? As we require the Hamiltonian to be hermitian (such that its eigenvalues are real), the matrices i and must be hermitian as well,
i = i

and

= .

(6.21)

Furthermore, we can show using just the anti-commutation relations and normalization above that they all have eigenvalues 1 and zero trace. It then also follows that they must have even dimension. It can be shown that the lowest dimensional matrices that have the desired behaviour are 4x4 matrices. (See exercise 6.1 below and also Aitchison (1972) 8.1). The choice of the matrices i and is not unique. Here we choose the Dirac-Pauli representation, = 0 0 and =
1 1

0 0 1 1

(6.22)

Of course, we may expect that the nal expressions for the amplitudes are independent of the representation: all the physics is in the anti-commutation relations themselves. Another frequently used choice is the Weyl representation, = 0 0 and = 0 1 1 1 0 1 . (6.23)

6.3

Covariant form of the Dirac equation

With Diracs Hamiltonian and the substitution p = i we arrive at the following relativistic Schr odinger-like wave equation, i = (i + m) . t (6.24)

Multiplying on the left by and using 2 = we can write this equation as i + i m = 0 . t (6.25)

82 We now dene the four Dirac -matrices by

LECTURE 6. THE DIRAC EQUATION

(, ) The wave equation then takes the simple form (i m) = 0

(6.26)

(6.27)

This equation is called the Dirac equation. Note that is a four-element vector. We call it a bi-spinor or Dirac spinor. We shall see in the next lecture that the solutions of the Dirac equation have four degrees of freedom, corresponding to spin-up and spin-down for a particle and its anti-particle. The Dirac equation is actually a set of 4 coupled dierential equations, for each j=1,2,3,4
4 3

:
k=1 =0

i ( )jk mjk . . . . . . . .

(k ) = 0

or

: i

. . . .

. . . .

1 0 0 0

0 1 0 0

0 0 1 0

0 0 m 0 1

1 0 2 0 3 = 0 4 0

or even more explicit, in the Dirac-Pauli representation,


1 1

0 1 1

i + t

0 1 1 0

i + x

0 2 2 0

i + y

0 3 3 0

i z

1 1

0 1 1

1 0 2 0 m 3 = 0 4 0

Take note of the use of the Dirac (or spinor) indices (j, k = 1, 2, 3, 4) simultaneously with the Lorentz indices ( = 0, 1, 2, 3). As far as it concerns us, it is a coincidence that both types of indices assume four dierent values. To simplify notation even further we dene the slash operator of a four-vector a as a = a . particles can then be written very concisely as The wave equation for spin- 1 2 (i m) = 0 . (6.29) (6.28)

Note that, although we write like a contra-variant four-vector, it actually is not a four-vector. It is a set of four constant matrices that are identical in each Lorentz frame. For a four-vector a , a is a (4 4) matrix, but it is not a Lorentz invariant and still depends on the frame. The behaviour of Dirac spinors under Lorentz transformations is not entirely trivial. See also Griths 7.3 and Halzen and Martin 5.6.

6.4. DIRAC ALGEBRA

83

6.4

Dirac algebra

From the denitions of and we can derive the following relation for the anticommutator of two -matrices { , } + = 2 g 1 14 (6.30)

where the identity matrix on the right-hand side is the 4 4 identity in bi-spinor space. Text books usually leave such identity matrices away. However, it is important to realize that the equation above is a matrix equation for every value of and . In particular, g is not a matrix in spinor space. (In the equation, it is just a number!) Using this result we nd 0
2

=1 14

= 2

= 3

= 1 14

(6.31)

Also we have the Hermitian conjugates: 0

= 0 =
i

(6.32) = = i = i
i

(6.33)

Using the anticommutator relation once more then gives = 0 0 (6.34)

In words this means that we can undo a hermitian conjugate 0 by moving a 0 through it, 0 = 0 . Finally, we dene 5 = i 0 1 2 3 which has the characteristics 5 = 5

(6.35)

14 =1

5, = 0

(6.36)

In the Dirac-Pauli representation 5 is 5 = 0


1 12

12 1
0

(6.37)

6.5

Adjoint spinors and current density

Similarly to the case of the Schr odinger and the Klein-Gordon equations we can dene a current density j that satises a continuity equation. First, we write the Dirac equation as i 0 +i k k m = 0 (6.38) t x k=1,2,3

84

LECTURE 6. THE DIRAC EQUATION

As we now work with matrices, we use hermitian conjugates rather than complex conjugates and nd for the conjugate equation i 0 i k m = 0 k t x k=1,2,3 (6.39)

However, we now see a potential problem: the additional minus sign in k disturbs the Lorentz invariant form of the equation. We can restore Lorentz covariance by multiplying the equation from the right by 0 . Therefore, we then dene the adjoint Dirac spinor = 0 . (6.40) Note that the adjoint spinor is a row-vector: 1 2 Dirac spinor : Adjoint Dirac spinor: 3 4 The adjoint Dirac spinor satises the equation i k 0 i m = 0 k t x k=1,2,3 (6.41)

1, 2, 3, 4

which can be written in short-hand as i + m = 0. (6.42)

Now we multiply the Dirac equation from the left by and we multiply the adjoint Dirac equation from the right by : i + m = 0 (i m ) ( ) + = 0 + = 0

Consequently, we realize that if we dene a current as j = (6.43)

then this current satises a continuity equation, j = 0. The rst component of this current is simply
4

j = = =
i=1

|i |2

(6.44)

6.6. BILINEAR COVARIANTS

85

which is always positive. This property was the original motivation of Diracs work. Note that the current density can also be written as j = . (6.45)

The form Eq. (6.43) suggests that the Dirac probability current density transforms as a contravariant four-vector. In contrast to the Klein-Gordon case, this is not so easy to show since the Dirac spinors transform non-trivially. We will leave the details to the textbooks.

6.6

Bilinear covariants

The Dirac probability current in Eq. (6.43) is an example of a so-called bilinear covariant : a quantity that is a product of components of and and obeys the standard transformation properties of Lorentz scalars, vectors or tensors. The bilinear covariants represent the most general form of currents consistent with Lorentz covariance. Given that and each have four components, we have 16 independent combinations. Requiring the currents to be covariant, then leads to the following types of currents: scalar vector tensor axial vector pseudo scalar # of components 1 4 6 5 4 5 1

(6.46)

where the (anti-symmetric) tensor is dened as i ( ) 2 (6.47)

The names axial and pseudo refer to the behaviour of these objects under the parity transformation, x x. The scalar is invariant under parity, while the pseudo scalar changes sign. The space components of the vector change sign under parity, while those of the axial vector do not. We shall discuss the bilinear covariants and their transformation properties in more detail in Lecture 9.

6.7

Charge current and anti-particles

Once we consider interactions of fermions in QED, we are interested in charge density rather than probability density. Following the Pauli-Weiskopf prescription for the

86

LECTURE 6. THE DIRAC EQUATION

complex scalar eld, we multiply the current of (negatively charged) particles by e,


jem = e .

(6.48)

Using the ansatz = u(p)eipx (motivated in the next lecture) the interaction current density 4-vector takes the form
jf i = e (

uf

ui ei(pf pi )x

(6.49)

In the next lecture we will see that although the probability density of the Dirac elds is now positive, the negative energy solutions just remain. Following the FeynmanSt uckelberg interpretation the solution with negative energy is again seen as the antiparticle solution with positive energy. However, when it comes to the Feynman rules, there is an additional subtlety for fermions. In the case of Klein-Gordon waves the current of an antiparticle (j = 2|N |2 p ) gets a minus sign with respect to the current of the particle, due to reversal of 4-momentum. This cancels the change in the sign of the charge and that is how we came to the nice property of crossing: simply replace any anti-particle by a particle with opposite momentum. For fermions this miracle does not happen: the current does not automatically change sign when we go to anti-particles. As a result, if we want to keep the convention that allows us to replace anti-particles by particles, we need an additional ad-hoc minus antiparticle. sign in the Feynman rule for the current of the spin- 1 2 This additional minus sign between particles and antiparticles is only required for fermionic currents and not for bosonic currents. It is related to the spin-statistics connection: bosonic wavefunctions are symmetric, and fermionic wavefunctions are antisymmetric. In eld theory3 the extra minus sign is a result of the fact that bosonic eld operators follow commutation relations, while fermionic eld operators follow anticommutation relations. This was realized rst by Pauli in 1940.

Exercises
Exercise 6.1 (Representations of Dirac matrices) (a) Write a general Hermitian 2 2 matrix in the form a b where a and c b c are real. Write then b = s + it and show that the matrix can be written as: {(a + c) /2} I + s1 t2 + {(a c) /2} 3 How can we conclude that and cannot be 2 2 matrices?

See Aitchison & Hey, 3rd edition 7.2

6.7. CHARGE CURRENT AND ANTI-PARTICLES

87

(b) Show that the and matrices in both the Dirac-Pauli as well as in the Weyl representation have the required anti-commutation behaviour. Exercise 6.2 (From Dirac to Klein-Gordon) Each of the four components of the Dirac equation satises the Klein Gordon equation, ( + m2 ) i = 0. Show this explicitly by operating on the Dirac equation from the left with (i + m). ( a b + a b ) by just Hint: For any a and b we can write a b = 1 2 renaming indices. Now take the special case that b = a and the a commute. Then 1 we can write a a = 2 ( + )a a . Now use the anti-commutation relation of the -matrices. Exercise 6.3 (Dirac algebra: traces and products of matrices) Use the anticommutator relation for Dirac -matrices in Eq. (6.30) (or anything that follows from that) to show that: (a) a b + b a = 2 (a b) 1 14 (b) i) = 4 1 14 ii) a = 2 a iii) a b = 4 (a b) 1 14 iv) a b c = 2 c b a (c) i) Tr (odd number of -matrices) = 0 ii) Tr( a b ) = 4 (a b) iii) Tr( a b c d ) = 4 [ (a b)(c d) (a c)(b d) + (a d)(b c) ] (d) i) Tr 5 = Tr i 0 1 2 3 = 0 ii) Tr 5 a b = 0 iii) Optional excercise for die-hards: Tr 5 a b c d = 4 i a b c d where = +1 (1) for an even (odd) permutation of 0,1,2,3; and 0 if two indices are the same.

88

LECTURE 6. THE DIRAC EQUATION

Lecture 7 Solutions of the Dirac Equation


The notes of this Lecture follow very closely sections 5.2 - 5.5 of Halzen & Martin. The same material is covered in Griths section 7.4

7.1

Plane waves solutions with p = 0

In this lecture we look for solutions of the Dirac equation, (i m) = 0 . (7.1)

In exercise 6.2 you have shown that each of the components of the Dirac wave satises the Klein-Gordon equation. Therefore, we try the construct the solutions as plane wave solutions (x) = u(p) eipx (7.2) where u(p) is a 4-component column-vector that does not depend on x. After substitution in the Dirac equation we nd what is called the Dirac equation in the momentum representation, ( p m) u(p) = 0 (7.3) which, using the slash notation can also be written as ( p m) u(p) = 0 . (7.4)

In momentum-space the coupled dierential equations reduce to a set of coupled linear equations. In the Pauli-Dirac representation we have
1 1

0 0 1 1

0 i i 0

pi 89

1 1 0 0 1 1

uA uB

=0

(7.5)

90

LECTURE 7. SOLUTIONS OF THE DIRAC EQUATION

where uA and uB are two-component spinors. We can rewrite this as a set of two equations ( p) uB = (E m) uA ( p) uA = (E + m) uB , where (1 , 2 , 3 ). Now consider a particle with non-zero mass in its restframe, p = 0. In this case, the two equations decouple, E uA = m uA E uB = m uB . There are four independent solutions, which we write as 1 0 0 , u(2) = N 1 , u(3) = N u(1) = N 0 0 0 0 (7.7) (7.6)

0 0 u(4) = N 0 1 (7.8) The rst two have a positive energy eigenvalue E = m and the second two a negative energy E = m. We discuss the normalization constant N later.

0 0 , 1 0

7.2

Plane wave solutions for p = 0

To extend the solution to particles with non-zero momentum, we rst dene (1) 1 0 and (2) 0 1 (7.9)

Now consider two Dirac spinors for which the two upper coordinates uA (p) of u(p) are given by , i.e. (s {1, 2}) uA = (1)
(1)

and

uA = (2) .

(2)

(7.10)

Substituting this into the second equation of (7.6) gives for the lower two components uB
(1,2)

p (1,2) p (1,2) uA = . E+m E+m

(7.11)

To prove that these are indeed solutions of the equations, one can use the identity ( p) ( p) = |p|2 1 12 such that uA
(1,2)

(7.12)

and uB

(1,2)

also satisfy the rst equation in (7.6). (See also exercise 7.2.)

7.2. PLANE WAVE SOLUTIONS FOR P = 0 In the Pauli-Dirac representation the Hamiltonian is given by H = p + m = 0 p p 0 + m1 12 0 0 m1 12

91

(7.13)

With a bit of algebra we obtain for our solution Hu


(1)

m+

p2 E +m (1) E uB

uA

(1)

(7.14)

which illustrates two things: In order that u(1) be a solution we need indeed that E 2 = p2 + m2 . Furthermore, in the limit that p 0, the energy eigenvalue is +m, such that this is a positive energy solution. The calculation for u(2) is identical. Hence, two orthogonal positive-energy solutions are u(1) = N (1)
p (1) E +m

and

u(2) = N

(2)
p (2) E +m

(7.15)

where N is again a normalization constant. In an exactly analogous manner, we can start for our E < 0 solutions with the lower component given by (s) , uB = (1)
(3)

uB = (2) .

(4)

(7.16)

Using the rst of the equations in Eq. (7.6) gives for the upper coordinates uA
(3,4)

p p (3,4) uB = (1,2) Em (E ) + m

(7.17)

Note the dierence in the enumerator: it has become (E m) rather than (E + m). Evaluating the energy eigenvalue, we now nd e.g. Hu
(3)

E uA m +

(3)

p2 E m

uB

(3)

(7.18)

which again requires E 2 = p2 + m2 and in the limit p 0 gives E = m, a negative energy solution. Consequently, two negative-energy orthogonal solutions are given by u
(3)

=N

p (1) E m (1)

and

(2)

=N

p (2) E m (2)

(7.19)

To gain slightly more insight, lets write them out in momentum components. Use the denition of the Pauli matrices we have p= 0 1 1 0 px + 0 i i 0 py + 1 0 0 1 pz (7.20)

92 to nd ( p ) uA =
(2) (3) (1)

LECTURE 7. SOLUTIONS OF THE DIRAC EQUATION

pz px ipy px + ipy pz

1 0

pz px + ipy

(7.21)

and similar for uA , uB , uB . The solutions 1 0 E > 0 spinors u(1) (p) = N p z E+ m E < 0 spinors u(3) (p) = N

(4)

can then be written as , ,

px +ipy E +m pz E +m px ipy E +m

u(2) (p) = N

0 1

1 0

u(4) (p) = N

px ipy E +m pz E +m px +ipy E +m pz E +m

0 1

You can verify that the u(1) - u(4) solutions are indeed orthogonal, i.e. that u(i) u(j ) = 0 for i = j . (7.22)

7.3

Helicity

The Dirac spinors for a given momentum p have a two-fold degeneracy. This implies that there must be an additional observable that commutes with H and p and the eigenvalues of which distinguish between the degenerate states. Could the extra quantum number be spin? So, eg.: u(1) = spin up, and u(2) =spin down? Dene the spin operator as S =
1 2

, with = 0 0 . (7.23)

In exercise 26 you will show that does not commute with the Hamiltonian in Eq. (7.13). We can also realize this by looking directly at our Dirac spinor solutions: If spin is a good quantum number then those solutions should be eigenstates of the spin operator, u(i) = s u(i) ? where s is the spin eigenvalue. Now insert 1 0 0 0 pz / (E + m) (px + ipy ) / (E + m) one of the solutions, for example u(1) , 1 ? 0 = s pz / (E + m) (px + ipy ) / (E + m)

and you realize that this could never be true for arbitrary px , py , pz .

7.4. ANTIPARTICLE SPINORS The orbital angular momentum operator is dened as usual as L=rp You will also show in exercise 26 that the total angular momentum
1 J = L+ 2

93

(7.24)

(7.25)

does commute with the Hamiltonian. Now, as we can choose an arbitrary axis to get the spin quantum numbers, we can choose an axis such that the orbital angular momentum vanishes, namely along the direction of the momentum. Consequently, we dene the helicity operator as 1 p 1 0 (7.26) = p 0 p 2 2 p/|p|. We could interpret the helicity as the spin component in the direction where p of movement. One can verify that indeed commutes with the Hamiltonian in (7.13). As and H commute, they have a common set of eigenvectors. However, that does not necessarily mean that our solutions u(i) are indeed also eigenvectors of . In fact, with our choice above, they are only eigenvectors of if we choose the momentum along the z -axis. The reason is that the two-component spinors (s) are eigenvectors of 3 only. For other directions of the momentum, we would need to choose a dierent linear combination of the u(i) to form a set of states that are eigenvectors for both H and . Now, consider a momentum vector p = (0, 0, p). Applying the helicity operator on u(i) gives 1 1 1 ) uA = ( p 3 uA = uA 2 2 2 1 1 1 ) uB = ( p 3 uB = uB 2 2 2 where the plus sign holds for u(1,3) and the minus sign for u(2,4) . So you see that indeed u is an eigenvector of with eigenvalues 1/2. Positive helicity states have spin and momentum parallel, while negative helicity states have them anti-parallel.

7.4

Antiparticle spinors

As for the solutions of the K.-G. equation, we interprete u(1) and u(2) as the positive energy solutions of a particle (electron, charge e ) and u(3) , u(4) as the positive energy solutions of the corresponding antiparticle (the positron). We dene the antiparticle components of the wave function as v (1) (p) u(4) (p) v (2) (p) u(3) (p) . (7.27)

94

LECTURE 7. SOLUTIONS OF THE DIRAC EQUATION

The minus sign in u(3) is chosen such that the charge conjugation transformation (discussed in section 7.7) implies u(1) v (1) and u(2) v (2) . Using this denition we can replace the two negative energy solutions by the following anti-particle spinors with positive energy, E = + p2 + m2 , v (1) (p) = N
px ipy E +m pz E +m

v (2) (p) = N

pz E +m (px +ipy ) E +m

0 1

1 0

The spinors u(p) of matter waves are solutions of the Dirac equation in momentum space, Eq. (7.3). Replacing p with p in the Dirac equation we nd that our positive energy anti-particle spinors satisfy another Dirac equation, ( p + m) v (p) = 0 (7.28)

7.5

Normalization of the wave function

As for the Klein-Gordon case we choose a normalization such that there are 2E particles per unit volume. Remember that we had in the previous lecture for the rst component of the current of the Dirac wave (x) = (x) (x) . (7.29)

Substituting the plane wave solution = u(p) eipx , and integrating over a volume V we nd d3 x = u (p) eipx u(p) eipx d3 x = u (p) u(p) V (7.30)
V V

Consequently, to nd 2E particles per unit volume we must normalize such that u (p) u(p) = 2E Explicit calculation for the positive energy solutions (s {1, 2}) gives u(s) u(s) = N 2 (s) (s) + (s) = N2 1 + p2 (E + m)2
T T

(7.31)

( p) ( p) (s) (E + m)2 = N2 2E E+m

Consequently, in order to have 2E particles per unit volume we choose N = E+m.

(7.32)

7.6. THE COMPLETENESS RELATION

95

The computation for the positive energy antiparticle waves v (p) leads to the same normalization. We can now write the orthogonality relations as (with r, s {1, 2}) u(r) u(s) = 2E rs v (r) v (s) = 2E rs

(7.33)

7.6

The completeness relation

We now consider the Dirac equation for the adjoint spinor u, v . Taking the hermitian conjugate of Eq. (7.3) and multiplying on the right by 0 we have u 0 p u 0 m = 0 (7.34)

Using that 0 = 0 we then nd for the Dirac equation of the adjoint spinor u = u 0 , u ( p m) = 0 In the same manner we nd for the adjoint antiparticle spinors v ( p + m) = 0 (7.36) (7.35)

Using these results you will derive in exercise 25 the so-called completeness relations u(s) (p) u(s) (p) = ( p + m)
s=1,2

v (s) (p) v (s) (p) = ( p m)


s=1,2

(7.37)

These relations will be used later on in the calculation of amplitudes with Feynman diagrams. Note that the left-hand side is not an inner product. Rather, on both sides we have a (4 4) matrix, or schematically
. . (....) = . .
1 1

p +

(Note:

s=3,4

u(s) (p) u(s) (p) =

s=1,2

v (s) (p) v (s) (p) = ( p + m) )

96

LECTURE 7. SOLUTIONS OF THE DIRAC EQUATION

7.7

The charge conjugation operation

The Dirac equation for a particle in an electromagnetic eld is obtained by substituting + iqA in the free Dirac equation. For an electron (q = e) this leads to: [ (i + eA ) m] = 0 . Similarly, there must be a Dirac equation describing the positron (q = +e): [ (i eA ) m] C = 0 , (7.39) (7.38)

where the positron wave function C is obtained by a one-to-one correspondence with the electron wave function . To nd the relation between C and , lets take the complex conjugate of the electron equation, [ (i eA ) m] = 0 . (7.40) Now suppose that there is a matrix M such that = M 1 M . We can then rewrite the equation above as M 1 [ (i eA ) m] M = 0 . and we obtain the relation C = M = M 0 C
T T

(7.41)

(7.42)

(7.43)

where we have used the denition of the adjoint spinor (see Lecture 6) and dened the charge conjugation matrix C = M 0 . It can be shown (see Halzen and Martin exercise 5.6) that in the Pauli-Dirac representation a possible choice of M is 1 1 . M = C 0 = i 2 = (7.44) 1 1 Interpreting the probability current as a charge current, we dene the electron current as j (7.45) e = e The current of the charge conjugate wave function is then
C j = eC C = . . . = e e

(7.46)

7.7. THE CHARGE CONJUGATION OPERATION

97

Exercises
Exercise 7.1 (Pauli vector identity) (a) Prove the Pauli vector identity

12 + i (a b) ( a)( b) = a b 1

(7.47)

Hint: Write the inner products as sums over i and j . Use that i j = ij + i k i,jk k . Use that i,j ijk ai bj = (a b)k . (b) Prove the identity in Eq. (7.12), i.e.

12 ( p) ( p) = |p|2 1
Hint: you can use your answer to (a)! If you didnt manage to derive (a), try by using the explicit form of the Pauli matrices in Eq. (6.7).

Exercise 7.2 (Energy eigenvalue of solutions to Dirac equation) Starting from the Dirac equation in momentum space, Eq. (7.6), show . . . (a) . . . by eliminating uB that a solution to the Dirac equation satises the relativistic relation between energy and momentum. (b) . . . for a non-relativistic particle with velocity , uB is a factor 1 smaller than 2 uA . (In a non-relativistic description uA and uB are often called respectively the large and small components of the wave function.) Hint: Use the result from exercise 7.1b.

Exercise 7.3 (See also H&M p.110-111 and Griths p. 242) The spinors u, v , u and v are solutions of respectively: ( p m) u ( p + m) v u ( p m) v ( p + m) (a) Use the orthogonality relations: u(r) u(s) = 2E rs v (r) v (s) = 2E rs to show that: u (s) u(s) = 2m v (s) v (s) = 2m = = = = 0 0 0 0

98

LECTURE 7. SOLUTIONS OF THE DIRAC EQUATION Hint: evaluate the sum of u 0 ( p m)u and u( p m) 0 u and use 0 k = k 0 (k = 1, 2, 3).

(b) Derive the completeness relations: u(s) (p) u (s) (p) = p + m


s=1,2

v (s) (p) v (s) (p) = p m


s=1,2

Hint: For s = 1, 2 take the solution u(s) from Eq. (7.15) and write out the rowvector for us using the explicit form of 0 in the Dirac-Pauli representation. Then 12 . Finally, note that write out the matrix u(s) u(s) and use that s=1,2 (s) (s) = 1 p = E1 12 p p E 1 12 (7.48)

Exercise 7.4 (See also exercise 5.4 of H& M and exercise 7.8 of Griths) The purpose of this problem is to demonstrate that particles described by the Dirac equation carry intrinsic angular momentum (S ) in addition to their orbital angular momentum (L). We will see that L and S = /2 are not conserved individually but that their sum is. (a) Consider the Hamiltonian that leads to the Dirac equation, H = p + m Use the fundamental commutator [xi , pj ] = i ij (with [H, L] = i p = 1) to show that (7.49)

where L = x p. Hint: To do this eciently use the Levi-Civita tensor to write out the cross product as Li = j,k ijk xj , pk . Now evaluate the commutator [H, Li ]. (b) Show that [k , l ] = 2i
m klm m

where the operator (see also Eq. (7.23)) and in the Pauli-Dirac representation were 0 0 = and = 0 0 Hint: Use the commutation relation for the Pauli spin matrices [i , j ] = 2i (which follows from i j = i k ijk k ).
ijk k

7.7. THE CHARGE CONJUGATION OPERATION (c) Use the result in (b) to show that [H, ] = 2 i p

99

(7.50)

We see from (a) and (c) that the Hamiltonian commutes with J = L + 1 . 2 Exercise 7.5 (Pauli equation as non-relativistic limit) In this exercise you will show that in a non-relativistic approximation, the Dirac equation combined with minimal substitution leads to the Schr odinger-Pauli equation, and hence to the prediction of the gyromagnetic ratio of the electron. Consider again minimal substitution for a charge q and a eld A = (V, A): p p qA = E E qV p p qA

In the following we concentrate on the uA component because in exercise 7.2 you have shown that in the non-relativistic limit the other component is small. (a) Starting from the Dirac equation in momentum space Eq. (7.6),write down the equations for uA and uB after minimal substitution. (b) In coordinate space p and E are operators. Therefore, they do not commute with A and V . That means that unlike in the case for free particles (exercise 7.2) you cannot just eliminate uB ! However, in the non-relativistic limit and assuming |qV | m we can use the approximation E qV + m 2m. Use this to eliminate uB and obtain the Dirac equation for uA [ (p q A)] [ (p q A)] uA = 2m(Ekin qV )uA where Ekin = E m. (c) Use the Pauli vector identity, Eq. (7.47), to show that [ (p q A)] [ (p q A)] = (p q A)2 iq (p A + A p) (d) Show that in coordinate space (p A + A p) uA = i ( A) uA (7.53) (7.52) (7.51)

where on the right hand side the derivative works only on A and not on uA . Therefore, we can replace it with B = A. (e) Using these results, show that the Dirac equation for an electron with charge q = e in the non-relativistic limit in an electromagnetic eld A = (A0 , A) reduces to the Schr odinger-Pauli equation i d A = dt 1 e (p + eA)2 + B eA0 A , 2m 2m (7.54)

where we have identied Ekin as the classical operator id/dt.

100

LECTURE 7. SOLUTIONS OF THE DIRAC EQUATION

The term with eA0 in (7.54) is a constant potential energy that is of no further importance. The term with B arises due to the fact that p and A dont commute. In this term we recognise the magnetic eld: B = g e SB . 2m (7.55)

Here g is the gyromagnetic ratio, i.e. the ratio between the magnetic moment of a particle and its spin. Classically we have g = 1, but according to the Dirac equation 1 (S = 2 ) one nds g = 2. The current value of (g 2)/2 is according to the Particle Data Book (g 2)/2 = 0.001159652193 0.000000000010 (7.56) This number, and its precision, make QED the most accurate theory in physics. The deviation from g = 2 is caused by high order corrections in perturbation theory.

Lecture 8 Spin-1/2 Electrodynamics


8.1 Feynman rules for fermion scattering

With the spinor solutions of the Dirac equation we nally have the tools to calculate particles. Analogously to the case of spin-0 particles we cross section for fermions, spin- 1 2 determine the solutions of the Dirac equations in the presence of an electromagnetic eld A by starting from the free equation of motion and applying minimal substitution, p p qA . For a particle with mass m and charge q = e, the perturbed Dirac equation then becomes ( p m) + e A = 0 . (8.1) To isolate the perturbation term we write this again in terms of a Hamiltonian, (H0 + V ) = E (8.2)

One can either start from the Dirac equation in terms of and , or, work towards that form by multiplying the Dirac equation on the left by 0 . The result is E = 0 k pk + 0 m e 0 A
H0 =p+m V

(8.3)

Consequently, the perturbation potential is V (x) = e 0 A . (8.4)

In analogy to spinless scattering we now write for the transition amplitude Tf i = i


f (x) V (x) i (x) d4 x

(8.5)

Note the dierences with the case of the KG solutions in spinless scattering: The wave function has four components and the perturbation potential V (x) becomes a (4 4) 101

102

LECTURE 8. SPIN-1/2 ELECTRODYNAMICS

matrix. We take a hermitian conjugate of the wave , rather than its complex conjugate. The transition amplitude is still just a scalar. Substituting the expression for V (x) we obtain Tf i = i = i
f (x) e 0 A (x) i (x)d4 x

(8.6) f (x) (e) i (x)A (x) d x


4

In Lecture 6 we dened the charge current density of the Dirac wave as j (x) = e (x) (x) In complete analogy to the spinless particle case we dene the electromagnetic transition current between states i and f as
jf i (x) = e f (x) i (x) ,

(8.7)

such that the transition amplitude can be written as Tf i = i


fi 4 j A d x.

(8.8)

After inserting the plane wave decomposition (x) = u(p)eipx , the transition current becomes i(pf pi )x . (8.9) jf i = euf ui e Note that the current is a scalar in Dirac spinor space, or schematically,
jf i = ( uf )

ui

(8.10)

Now consider again the two-body scattering A + B C + D: uB


jBD

uD

q2
jAC

uA

uC

Just as we did for the scattering of spinless particles, we obtain the vector potential A by using the Maxwell equation with the transition current of one of the two particles (say particle AC) as a source. That is, we take
2A = jAC .

8.1. FEYNMAN RULES FOR FERMION SCATTERING and obtain for the potential A = 1 j . q 2 AC

103

where q pi pf is the momentum transfer. The transition amplitude becomes Tf i = i


BD j

1 4 j d x = i q 2 AC

jBD

g 4 j dx q 2 AC

(8.11)

which is symmetric in terms of particle BD and AC. Inserting the expressions of the plane wave currents using Eq. (8.9) we obtain Tf i = i e uC uA ei(pC pA )x g e uD uB ei(pD pB )x d4 x 2 q (8.12)

Performing the integral (and realizing that nothing depends on x except the exponentials) leads us to the expression Tf i = i (2 )4 4 (pD + pC pB pA ) M with the matrix element given by iM = ie (uC uA )
vertex propagator

(8.13)

ig q2

ie (uD uB )
vertex

(8.14)

From the matrix element we can now read of the Feynman rules. Again, as for the spinless case, the various factors are dened such that the rules can also be applied to higher order diagrams. without spin: 1 ie (pf + pi ) 1 ui with spin: ie uf

Figure 8.1: Diagrams for a spin-0 (left) and spin- 1 2 (right) particle with charge e interacting with the EM eld.

The rules for the vertex factors for spin-0 and spin- 1 particles are shown side-by-side 2 in Fig. 8.1. A spinless electron can interact with A only via its charge. The coupling is proportional to (pf + pi ) . However, an electron with spin can also interact with the magnetic eld via its magnetic moment. As you will prove in exercise 8.1, we can rewrite the Dirac current as uf ui = 1 uf (pf + pi ) + i (pf pi ) ui 2m (8.15)

104 where the tensor is dened as

LECTURE 8. SPIN-1/2 ELECTRODYNAMICS

i ( ) . 2

(8.16)

(We have seen this tensor before in lecture 6, when we discussed bilinear covariants.) This formulation of the current is called the Gordon decomposition. We observe that in addition to the contribution that appears for the spinless wave, there is a new contribution that involves the factor i (pf pi ). In the non-relativistic limit this leads indeed to a term proportional to the magnetic eld component of A , just as you would expect from a magnetic moment.

8.2

Electron-muon scattering

We will now use the Feynman rules to calculate the cross section of the process e e .The Feynman diagram is drawn in Fig. 8.2. : u B ieuD uB iM = ig q2 ieuC uA e : uA e : u C
Figure 8.2: Lowest order Feynman diagram for e scattering.

: uD

Applying the Feynman rules we nd for the lowest-order amplitude iM = e2 uC uA and for its square |M| = e
2 4

i uD uB q2

(8.17)

1 (uC uA ) 2 (uD uB ) q

1 (uC uA ) 2 (uD uB ) q

(8.18)

Note that for a given value of and the currents are just complex numbers. (The -matrices are sandwiched between the bi-spinors.) Therefore, we can reorder them and write the amplitude as |M|2 = e4 q4 [(uC uA ) (uC uA ) ] [(uD uB ) (uD uB ) ]

(8.19)

We have factorized the right hand side into two tensors, each of which only depends on one of the leptons. We call these the polarized lepton tensors.

8.2. ELECTRON-MUON SCATTERING

105

Up to now we have ignored the fact that the particle spinors come in two avours, namely one for positive and one for negative helicity. Assuming that we do not measure the helicity (or spin) of the incoming and outgoing particles, the cross-section that we need to compute is a so-called unpolarized cross-section: If the incoming beams are unpolarized, we have no knowledge of initial spins. Therefore, we average over all spin congurations of the initial state; If the spin states of the outgoing particles are not measured, we should sum over spin congurations of the nal state. Performing the summation and averaging leads to the following unpolarized matrix element 1 |M|2 |M|2 = |M|2 (8.20) (2sA + 1) (2sB + 1) spin where 2sA + 1 is the number of spin states of particle A and 2sB + 1 for particle B. So the product (2sA + 1) (2sB + 1) is the number of spin states in the initial state. Some of you may wonder why in the spin summation we add up the squares of the amplitudes, rather than square the total amplitude. The reason is that it does not make a dierence since the nal states over which we average are orthogonal: there cannot be interference between states with dierent helicity in the nal state. It turns out that the math is easier when we sum over amplitudes squared.
1 . Inserting the expression for the amplitude Both the electron and the muon have s = 2 above, we can write the spin averaged amplitude as

1 e4 |M| = Lelectron Lmuon 4 4 q


2

(8.21)

where the unpolarized lepton tensors are dened as L electron =


espin

[uC uA ] [uC uA ] [uD uB ] [uD uB ] .


spin

L muon =

(8.22)

The spin summation is in fact rather tedious. The rest of the lecture is basically just the calculation to do this! First, take a look at the complex conjugate of the transition current that appears in the tensor. Since it is just a (four-vector of) numbers, complex conjugation is the same as hermitian conjugation. Consequently, we have [uC uA ] = [uC uA ] =
0 u C uA 0 = u A uC

(8.23)

= uA 0 0 uC = [uA uC ]

106

LECTURE 8. SPIN-1/2 ELECTRODYNAMICS

In other words, by reversing the order of the spinors, we can get rid of the complex conjugation and nd (uC uA ) (uA uC ) L (8.24) e =
e spin

Next, we apply what is called Casimirs trick. Write out the matrix multiplications in the tensors explicitly in terms of the components of the matrices and the incoming spins s and outgoing spins s , L e =
s s klmn uC,n uC,k kl uA,l uA,m mn (s ) (s) (s) (s )

(8.25)

Note that all of the factors on the right are just complex numbers, so we can manipulate their order and write this as L e =
klmn s uC,n uC,k kl s (s ) (s ) uA,l uA,m mn (s) (s)

(8.26)

Now remember the completeness relation, Eq. (7.37), that we derived in the previous lecture1 , u(s) u(s) = p + m (8.27)
s

This is exactly what we need to sum over the spin states! Substituting this expression for the spin sums gives L e =
klmn ( pC + me )nk kl ( pA + me )lm mn

(8.28)

where me is the electron mass. Lets look more carefully at this expression: the right hand side contains products of components of (4 4) matrices. Call the product of these matrices A. We could obtain the components of A by summing over the indices k , l and m. The nal expression for the tensor would then be L = n Ann , which is nothing else but the trace of A. Consequently, we can write the expression for the lepton tensor also as
L e = Tr [( pC + m) ( pA + m) ]

(8.29)

You will now realize why we made you compute the traces of products of -matrices in lecture 6. We briey repeat here the properties that we need: In general, for matrices A, B and C and any complex number z T r(zA) = z T r(A) Tr (A + B ) = Tr(A) + Tr(B )
for anti-fermions this gives an overall sign in the tensor: L e Le for each particle anti-particle. 1

8.2. ELECTRON-MUON SCATTERING Tr (ABC ) = Tr (CAB ) = Tr (BCA) For -matrices (from the anti-commutator + = 2g ): Tr(odd number of -matrices = 0) Tr ( ) = 4 g Tr( ) = 4 g g g g + g g Using the rst rule we can write out the tensor as a sum of traces,
L e = Tr [( pC + m) ( pA + m) ]

107

= Tr [ pC pA ] + Tr [m m ] + Tr [ pC m ] + Tr [m pA ]
case 1 case 2 3 s0 3 s0

(8.30)

The last two terms vanish because they contain an odd number of -matrices. For the second term (case 2) we nd Tr [m m ] = m2 Tr [ ] = 4 m2 g . Finally, for the rst term (case 1) we have Tr [ pC pA ] Tr pC, pA, = T r pC, pA, = 4 g g g g + g g pC, pA, = 4 ( p C pA + pC pA g (pA pC )) , (8.32) (8.31)

where we used the trace formula for four -matrices in the third step. Adding the two contributions gives for the lepton tensor
2 L = 4 p e C pA + pC pA + me pC pA g

(8.33)

The expression for the muon tensor is obtained with the substitution (pA , pC , me ) (pB , pD , m ), 2 L = 4 p (8.34) D pB + pD pB + m pD pB g To compute the contraction of the two tensors, which appears in the amplitude, we just write everything out
2 4 pD pB + pD pB + m2 L pD pB g e L = 4 pC pA + pC pA + me pC pA g

= 16 (pC pD ) (pA pB ) + (pC pB ) (pA pD ) (pC pA ) (pD pB ) + (pC pA ) m2 + (pC pB ) (pA pD ) + (pC pD ) (pA pB ) (pC pA ) (pD pB ) + (pC pA ) m2 (pC pA ) (pD pB ) (pC pA ) (pD pB ) + 4 (pC pA ) (pD pB ) 4 (pC pA ) m2
2 2 2 2 +m2 e (pD pB ) + me (pD pB ) 4me (pD pB ) + 4me m 2 2 2 = 32 (pA pB ) (pC pD ) + (pA pD ) (pC pB ) m2 e (pD pB ) m (pA pC ) + 2me m

108

LECTURE 8. SPIN-1/2 ELECTRODYNAMICS

Combining everything we obtain for the square of the unpolarized amplitude for electronmuon scattering |M|2 = 8 e4 q4 (pC pD ) (pA pB ) +
2 2 2 (pC pB ) (pA pD ) m2 e (pD pB ) m (pA pC ) + 2me m

(8.35)

We now consider the ultra-relativistic limit and ignore the rest masses of the particles. The amplitude squared then becomes |M|2 8 e4 q4 (pC pD ) (pA pB ) + (pC pB ) (pA pD ) (8.36)

Furthermore, we dene the Mandelstam variables


2 s (pA + pB )2 = p2 A + pB + 2 (pA pB ) t (pD pB )2 q 2 u (pA pD )2

2 (pA pB ) 2 (pD pB ) 2 (pA pD )

(8.37)

where the approximation on the right again follows in the ultra-relativistic limit. From energy-momentum conservation (p A + pB = pC + pD ) we have (pA + pB )2 = (pC + pD )2 (pD pB )2 = (pC pA )2 (pA pD )2 = (pB pC )2 which gives (pA pB ) (pC pD ) = 1 1 1 s s = s2 2 2 4 1 1 (pA pD ) (pC pB ) = u u 2 2 (8.39) 1 = u2 4 (8.40) (8.41) Inserting this in the amplitude, we nd |M|2 2 e4 s2 + u 2 t2 (8.42) pA pB = pC pD pD p B = pC pA pA p D = pB pC

(8.38)

Finally, as we did for the spinless scattering in Lecture 5, consider again the scattering process in the centre-of-momentum system. The four-vectors can then be written as p A = (|pA |, pA ) pC = (|pC |, pC ) p B = (|pA |, pA ) pD = (|pC |, pC )

8.3. CROSSING: THE PROCESS E E+ +

109

Dene p |pA | which, by four-vector conservation is also equal to |pB,C,D |. Dene as the angle between pA and pC (see Fig. 8.3), such that pA pC = pB pD = p2 cos We then nd for the Mandelstam variables s = 4 p2 t = 2 p2 (1 cos ) u = 2 p2 (1 + cos ) which gives for the amplitude squared |M|
2

(8.43)

8 e4

4 + (1 + cos )2 . (1 cos )2

(8.44)

Inserting this in the expression for the dierential cross-section (which we obtained after integrating over the nal state momenta in Lecture 5) we nd d d with e2 /4 . iM = e 1 1 = |M|2 2 64 s 2 4 + (1 + cos )2 2s (1 cos )2 (8.45)

c.m.

ieuD uB q2 ieuC uA

pA
q2

pC pB

pD
e

Figure 8.3: e e scattering. Left: the Feynman diagram. Right: denition of scattering angle in C.M. frame.

8.3

Crossing: the process ee+ +

We will now illustrate the method of crossing to obtain the amplitude for e e+ + scattering from the amplitude of e e scattering. The method is illustrated in Fig. 8.4, and comes down to the following: 1. assuming that you had computed the original amplitude in terms of particles, replace p p for every anti-particle in the diagram 2. now relabel the momenta such that the ingoing and outgoing lines correspond to those in the original diagram

110

LECTURE 8. SPIN-1/2 ELECTRODYNAMICS

3. for every crossed fermion line, i.e. for every outgoing fermion that became incoming or vice-versa, multiply the amplitude squared by a factor (1). (This has to do with the sign of the current which we discussed at the end of Lecture 6.)

e
e p A

e e pC pD

e e+
e p A

+ p
C

pB
Replace: P A P B P C P D P A P D P B P C

p B

e+

p
D

An antiparticle is a particle where p is replaced by p. The diagram in terms of particles is:

e p A p
B

p
C

p D

Figure 8.4: The principle of crossing. Use the anti-particle interpretation of a particle with the 4-momentum reversed in order to related the Matrix element of the crossed reaction to the original one.

Following these rules, we have


+ + + + |M|e e+ + (p e , pe , p , p ) = |M|e e (pe , p , pe , p ) 2 2

(8.46)

+ Labeling the outgoing momenta with a prime such that pA = p e , p B = pe , pC = p , + pD = p , we have

pA = pA

pB = pD

pC = pB

pD = p C

and nd for the Mandelstam variables of the original particle diagram s (pA + pB )2 = (pA pD )2 u t (pD pB )2 = (pC + pD )2 = s u (pA pD )2 = (pA pC )2 = t (8.47)

(To express the results in primed Mandelstam variables we have used that pA + pB = pC + pD .) Using the result in Eq. (8.42) the amplitude squared for the two processes are then

8.4. SUMMARY OF QED FEYNMAN RULES

111

|M|e e = 2 e4

s2 + u2 t2 u2 +t2 s2

t-channel:

q2 = t

|M|e e+ + = 2 e4

s-channel:

q2 = s

It is customary to label these a t-channel and an s-channel process, because we have q 2 = t and q 2 = s, respectively. We can express the momenta in the centre-of-momentum frame in terms of an initial momentum p and a scattering angle , where is now the angle between the incoming e (pA ) and the outgoing (pC ). The expressions for u , s and t are identical to those in (8.43). We immediately get for the matrix element: |M|c.m. = 2 e4
2

t2 +u2 = e4 1 + cos2 s2

(8.48)

The dierential cross-section becomes 2 d = 1 + cos2 d 4s (8.49)

Finally, to calculate the total cross section for the process we integrate over the azimuthal angle and the polar angle : e e
+ +

4 2 = 3 s

(8.50)

Note that the shape of the angular distribution does not depend on the available energy, but that the total cross-section scales as 1/s: the higher the cms energy, the smaller the cross-section. (If you look back to our original formulation of the golden rule, youll nd that 1/s dependence comes from the density of the incoming waves.) Figure 8.5 shows a comparison of the kinematic factors in the dierential cross-section of the t-channel process e e and the s-channel process e e+ + for leptons. For the t-channel process the dierence is only visible in the spin-0 and spin- 1 2 very backward region, while in the s-channel process there is a constant oset.

8.4

Summary of QED Feynman rules

In the computation of the e above we have seen only a subset of the Feynman rules for QED. As an example of things we missed, consider the annihilation process e+ e . (Draw it!) To compute the cross-section for this process we need more Feynman rules, namely those for the electron propagator and those for external photon

112

LECTURE 8. SPIN-1/2 ELECTRODYNAMICS

t-channel

s-channel

4s d 2 d

4s d 2 d 0.5 1 cos()

103
2

10

spin-1/2 spin-0

10 1 1 -1 -0.5 0 0 -1 -0.5 0 0.5 1 cos()

Figure 8.5: Leading order QED dierential cross-section d/d divided by 2 /4s as function of cos for the t-channel process e e (left) and the s-channel process e e+ + (right) in the ultra-relativistic limit (me = m = 0).

lines. We now briey summarize the rules for QED. You can nd these in more detail in the textbooks, e.g. in appendix D of Griths and on the inside of the cover of Halzen and Martin. For the external lines, we have in the matrix element:

spin-0: nothing incoming particle: outgoing particle: spin- 1 : 2 incoming anti-particle: outgoing particle: spin-1: incoming: outgoing:

u u v v

(8.51)

We have seen the photon polarization vectors in Lecture 3. Both the spin- 1 and spin-1 2 external lines carry also an index for the helicity. In calculations for cross-sections or decays in which we measure the spin, we need explicit forms of the Dirac spinors and the photon polarization vectors. However, often we sum over all incoming and outgoing spins (spin averaging) and we can use the completeness relations.

8.4. SUMMARY OF QED FEYNMAN RULES For the internal lines (the propagators) we have spin-0: spin- 1 : 2 q2 i m2

113

ig q2 spin-1: 2 masssive: i [g + q q /m ] q 2 m2 Finally, the QED vertex factors are


spin-0: ige (p in + pout )

i( q + m) q 2 m2 massless:

(8.52)

(8.53) spin-1: ige with ge the charge of the particle in the vertex. Section 7.6 of Griths contains worked out examples of several key QED processes, both with and without spin averaging.

Exercises
Exercise 8.1 (The Gordon decomposition) A spinless electron can interact with A only via its charge; the coupling is proportional to (pf + pi ) . An electron with spin, on the other hand, can also interact with the magnetic eld via its magnetic moment. This coupling involves the factor i (pf pi ). The relation between the Dirac current and the Klein-Gordon current can be studied as follows: (a) Show that the Dirac current can be written as uf ui = 1 uf (pf + pi ) + i (pf pi ) ui 2m

where the tensor is dened as = i ( ) 2

Hint: Start with the term proportional to and use: + = 2g and use the Dirac equations: pi ui = mui and uf pf = muf . (b) (optional!) Make exercise 6.2 on page 119 of H& M which shows that the Gordon decomposition in the non-relativistic limit leads to an electric and a magnetic interaction. (Compare also exercise 7.5.)

114

LECTURE 8. SPIN-1/2 ELECTRODYNAMICS

Exercise 8.2 Can you easily obtain the cross section of the process e+ e e+ e from the result of e+ e + ? If yes: give the result, if no: why not? Exercise 8.3 (The process e+ e + ) We consider scattering of spin 1/2 electrons with spin-0 pions. We assume pointparticles; i.e. we forget that the pions have a substructure consisting of quarks. Also we only consider electromagnetic interaction and we assume that the particle masses can be neglected. (a) Consider the process of electron - pion scattering: e e . Draw the Feynman diagram and write down the expression for the iM using the Feynman rules. (b) Perform the spin averaging of the electron and compute |M| . (Note the extra minus sign that appears from the 3rd crossing rule.) (c) Use the principle of crossing to nd |M| for e+ e + (d) Determine the dierential cross section d/d for e+ e + in the centre-ofmomentum of the e+ e -system.
2 2

8.4. SUMMARY OF QED FEYNMAN RULES

115

Figure 8.6: From Halzen and Martin, Quarks and Leptons

116

LECTURE 8. SPIN-1/2 ELECTRODYNAMICS

Lecture 9 The Weak Interaction


In 1896 Henri Becquerel was studying the eect of uorescence, which he thought was related to X-rays that had been discovered by Wilhelm R ontgen. To test his hypothesis he wrapped a photographic plate in black paper and placed various phosphorescent salts on it. All results were negative until he used uranium salts. These aected photographic plates even when put in the dark, such that the eects clearly had nothing to do with uorescence. Henri Becquerel had discovered natural radioactivity. We know now that the most nuclear decays of (not very) heavy nuclei in nature are the result of the transition of a neutron to an electron, a proton and an anti-neutrino,

p+ n e e
(9.1)

or in a formula, n p + e + e . A free neutron has a lifetime of about 15 minutes. However, as you know, the lifetime of various elements spans a very wide range. (The reason is that neutrons are not free particles in the nucleus!) Compare the lifetime of the following particles: particle 0 0 lifetime [sec] 4.4 1023 8.4 1017 2.6 108 2.2 106 dominant decay mode 0 + 0 e e

The listed decay modes are the dominant decay modes. Other decay modes exist, but they contribute marginally to the total decay width. As we have seen before, the lifetime 117

118

LECTURE 9. THE WEAK INTERACTION

of a particle is inversely proportional to the total decay width, = 1 . (9.2)

We have also seen that the decay width to a particular nal state is proportional to the matrix element squared. For example, for the two-body decay A B + C we had (in particle As rest frame) (A B + C ) = |M|2 pB d = |M|2 2EA 8m2 A (9.3)

scattering, the leading order contribution to the matrix For both spin-less and spin- 1 2 element is proportional to the square of the coupling constant. Consequently, the lifetime of particles tells us something about the strength of the interaction that is responsible for the decay. All fundamental fermions in the standard model feel the weak interaction. However, in processes that can also occur via the strong or electromagnetic interaction, those interactions will dominate. The reason that we still see the eects of the weak interaction is because the strong and electromagnetic interaction do not change quark and lepton avour. Consequently, if a particle has net quark or lepton avour and cannot decay to lighter states preserving avour, then it can only decay through the weak interaction. Note that in contrast to quarks and charged leptons, neutrinos feel only the weak interaction. That is the reason why they are so hard to detect! We can now understand the hierarchy of the lifetimes above as follows: The 0 particle (which is an excited meson consisting of u and d quarks and their anti-quarks) decays via the strong interaction to two pions. The 0 is the lightest neutral hadron such that it cannot decay to hadrons. It decays via the electromagnetic interaction to two photons, as we have seen in exercise 5.4. The + is the lightest charged hadron. Because it is charged, it cannot decay two photons. Instead, it decays via the weak interaction to a + and a neutrino. (It could also decay to an e+ and a neutrino, but for reasons explained later that mode is kinematically suppressed, despite the larger phase space.) The + is a lepton and therefore does not couple to the strong interaction. As it is charged, it cannot decay to photons. Its dominant decay is via the weak interaction to an electron and neutrinos. Considerations like these explain the gross features in the hierarchy of lifetimes. However, as you can also judge from the wide range in lifetimes of particles that decay weakly, kinematic eects must be important as well if we want to understand the lifetimes quantitatively.

9.1. THE 4-POINT INTERACTION

119

Besides the fact proper that the weak interaction unlike the electromagnetic and strong interaction does not honour the quantum numbers for quark and lepton avour, the weak interaction is special in at least two more ways: it violates parity symmetry P . Until 1956, when the parity violating aspects of the weak interaction were demonstrated, physicists were convinced that at least at the level of fundamental interactions our world was left-right symmetric; in the quark sector, it even violates CP symmetry. That means, because of CP T invariance, that it also violates T (time-reversal) symmetry. As we shall see, the existence of a third quark family was predicted from the observation that neutral Kaon decays exhibit CP violation.

9.1

The 4-point interaction

Lets turn back to nuclear decays. Based on the model of electromagnetic interactions Fermi proposed in 1932 the so-called 4-point interaction model, introducing the Fermi constant as the strength of the interaction: GF 1.166 105 GeV2 . n p+ The Feynman diagram of the 4-point interaction neutrino scattering on a neutron has the following matrix element: M = GF (up un ) (ue u ) e

(9.4)

This is to be compared to the electromagnetic diagram for electron proton scattering: p+ q2 e e p+ Here the matrix element was: M= 4 (up up ) (ue ue ) q2 (9.5)

1. e2 = 4 is replaced by GF 2. 1/q 2 is removed

We take note of the following properties of the weak interaction diagram: 1. The matrix element involves a hadronic current and a leptonic current. In contrast to electromagnetic scattering, these currents change the charge of the particles involved. In this particular process we have Q = 1 for the hadronic current and Q = 1 for the leptonic curent. Since there is a net charge from the hadron to the lepton current we refer to this process as a charged current interaction. We will see later that there also exists a neutral current weak interaction.

120

LECTURE 9. THE WEAK INTERACTION

2. There is no propagator; ie. a 4-point interaction. 3. There is a coupling constant GF , which plays a similar r ole as in QED. Since there is no propagator, the coupling constant is not dimensionless. 4. The currents have what is called a vector character similar as in QED. This means that the currents are of the form . The vector character of the interaction was in fact just a guess that turned out successful to describe many aspects of -decay. There was no reason for this choice apart from similarity of quantum electrodynamics. In QED the reason that the interaction has a vector behaviour is the fact that the force mediator, the foton, is a spin-1 (or vector) particle. In the most general case the matrix element of the 4-point interaction can be written as: (9.6) M = GF p (4 4) n e (4 4) where the (4 4) is a matrix. Lorentz invariance puts restrictions on the form of these matrices. In fact we have seen them already in lecture 6: Any such matrix needs to be a so-called bilinear covariant. The bilinear covariants all involve 4 4 matrices that are products of matrices: Scalar Vector Tensor Axial vector Pseudo scalar current 5 5 # components 1 4 6 4 1 # -matrices 0 1 2 3 4 spin 0 1 2 1 0

The most general 4-point interaction now takes the following form:
S,P,V,A,T

M = GF
i,j

Cij (up Oi un ) (ue Oj u )

(9.7)

where Oi , Oj are operators of the form S , V , T , A, P . It will not surprise you that the kinematics of a decay depend on the type of operator involved. For example, it can be shown (see eg. Perkins: Introduction to High Energy Physics, 3rd edition, appendix D) that for the decay n pe e S , P and T interactions imply that the helicity of the e and the e have the same sign; V and A interactions imply that they have opposite sign. In 1958 Goldhaber et. al. measured experimentally that the weak interaction is of the type: V , A, (ie. it is not S , P , T ). See Perkins ed 3, 7.5 for a full description of the experiment. The basic idea is the following.

9.2. PARITY

121

Consider the following decay of a particular Europium isotope to Samorium via a socalled electron capture reaction, 152 Eu + e 152 Sm (J = 1) + .
152Eu

+
1/2

e e e

152

Sm* +
+ +

1/2

1 1

A) B)

+ +
1/2

= +1/2
1/2

= 1/2

The excited Samorium nucleus is in a J = 1 state. To conserve angular momentum, J must be parallel to the spin of the electron, but opposite to that of the electronneutrino. Since in the rest frame of the decaying nucleus, the directions of the decay products are opposite, the polarisation (spin projection on momentum) of the nucleon and the neutrino are opposite as well. The neutrino in this decay cannot be observed. However, the spin projection of the Samorium nucleus can be probed with the photon that is emitted in its decay to the ground state, 152 Sm 152 Sm + . Measuring the photon spin is a work-of-art by itself, but assuming it can be done, this allows to distinghuish the two decay topologies above. The measurement by Goldhaber and collaborators showed that only case B actually . From this occurs. In other words: the neutrinos in this decay always have helicity 1 2 it was concluded that in the weak interaction only the V , A currents are involved and not S , P or T . Of course, that does not quite settle it. Goldhabers results established that the weak coupling was vector (like the photon) or axial vector. But which one was it? Or was it both? This question had actually already been answered a year earlier. The story is intreaging enough that we look at it in more detail, because it has everything to do with parity.

9.2

Parity

Parity, or (space) inversion, is the operation that multiplies all spatial coordinates by 1, so x x. It is closely related to reection in a mirror: the parity operation is identical to a reection in a plane through the origin, followed by a rotation under 180 degrees around an axis through the origin perpendicular to the mirror. Therefore, for systems that are rotation and translation invariant, the two are equivalent. When illustrating parity violation in pictures, we usually use an image with a reection in a mirror. Yet, when formulating the eect of parity in a physics theory, we work with space inversion (not in the least because that does not require choosing a plane for the mirror).

122

LECTURE 9. THE WEAK INTERACTION

Now, consider a process i f for some initial state i and nal state f , and the relation between i and f given by an operator that describes the time evolution, f i i f = U (9.8) can just be a continuous function (We can look at the process at any time scale. So U of time.) Denoting the parity operation by P we can also consider the mirror process, i and = P f . We dene the process to be symmetric under characterised by i = P f parity when it does not make any whether we rst tranform i to its mirror image and then look at its time evolution, i i f , or rst wait for the system to evolve and then reect it, i f phif . Or, in terms more common in quantum mechanics, the and U commute, process is symmetric under parity when P , P ] = 0 [U (9.9) (t) = eiHt/ 1 iHt/ , it follows that such P Because for small times t we have U also commutes with the Hamiltonian. This denition of a symmetry is not limited to commutes with H then mirror symmetry, but holds for any operator: if an operator Q it is called a symmetry operator. and H commute, then they have a common set of eigenvectors. If we now Now, if P consider eigenvectors with energy E that are not degenerate (that is, there is no other state with equal energy) then this immediately implies that these states have denite parity: they are eigenstates of the parity operator and there is a quantum number associated with the parity operation. If we apply the parity operator twice, then we put the system back in its original state. Consequently, if p is the eigenvalue of our state under the parity operator, then p2 = 1. (Strictly speaking, the system would be in the same state even if we had changed the wave function by an arbitrary phase. However, for simplicity we will not deal with the minor complications that this introduces.) Therefore, the eigenvalue is either +1 or 1. We call such states states of even and odd parity respectively. Until 1956 all the known laws of physics were invariant under inversion symmetry. At the scale of elementary particles our world was perfectly left-right symmetric. This symmetry was well tested for the electromagnetic and strong interaction and it was generally assumed that it held for the weak interaction as well. Since all all our leptons, mesons, and baryons are characterised by dierent masses (e.g. by dierent eigenvalues of the total Hamiltonian) they all have denite parity: they are either odd or even under the parity operation. (You will nd their quantum numbers for parity in the PDG.) These facts, denite parity for all quasi-stable particle and parity conservation in known interactions, is exactly what lead in the early fties to what was called the theta-tau puzzle. The and the were charged particles with strangeness one that decayed through the weak interaction to two and three pions respectively, + + + 0 + + + 2 0 or 2 + + (9.10)

9.3. COVARIANCE OF THE WAVE EQUATIONS UNDER PARITY

123

The pions were all known to have parity 1. Then, assuming parity to be conserved in these processes the theta had even parity and the tau odd parity. However, what was truely strange is that the theta and tau were otherwise seemingly identical particles: they had the same mass and same lifetime. After verifying that there had never been any experimental tests of parity conservation in the weak interaction, Lee and Yang hypothesized in 1956, that the tau and theta were actually the same particle, and that the weak interaction was responsible for the apparent violation of parity. They also proposed a number of experiments that could establish parity violation in weak decays directly. Within half a year two of these experiments were performed (Wu et all (1957), Garmin, Lederman and Weinrich (1957)) and the parity violating character of the weak interaction rmly established.

9.3

Covariance of the wave equations under parity

Let us now consider the parity transformation of the solutions to the wave equations in more detail. In our notation the parity operation transforms x x = x tt =t (x) (x )

(9.11)

We would now like to establish the relation between (x ) and (x). For the wave equation to be covariant (that is, the same form in every frame) this relation is to be chosen such that if (x) satises the wave equation, then (x ) satises the same wave equation. Now remember the Klein-Gordon equation for a free particle: ( + m2 )(x) = 0 (9.12)

Because this equation is quadratic in , the equation itself does not change under parity. Hence, one possible solution is simply KG (x ) = KG (x) = KG (x ) (9.13)

Note that any global change in the phase for is allowed as well: such a phase change can be considered part of our denition of parity and we can choose it to be zero. If we take to be a positive energy plane-wave solution with momentum p, then is also a positive energy solution but with momentum p. (Try!) Now consider the more complicated case of the Dirac equation. Split in its time and space part, the Dirac equation in coordinate space can be written as i (x, t) = (i + m) (x, t) t (9.14)

124

LECTURE 9. THE WEAK INTERACTION

In order for this equatin to be covariant under parity transformations we must nd the eld (x , t) such that it satises the tranformed Dirac equation, i (x , t) = (i + m) (x , t) t (9.15)

Applying our denition of space inversion, which implies = , we nd i (x, t) = (i + m) (x, t) t (9.16)

Note that (x, t) does not satisfy the Dirac equation due to the additional minus sign in front of the spatial derivative. However, we now multiply the equation on the left by and use that fact that and ant-commute. The result is i (x, t) = (i + m) (x, t) t (9.17)

Hence (x, t) does satify the Dirac equation. Consequently, one choice for the parity operation is (x , t) = (x , t) (9.18) Note, again, that we could insert any constant phase factor in the transformation. By convention, we choose that factor to be one. We now look at the solutions to the Dirac equation in the Pauli-Dirac representation. In this representation, we have for the = 0 matrix: 0 =
1 1

0 0 1 1

(9.19)

Consequently, the parity operator has opposite sign for the positive and negative energy solutions. In other words, fermions and anti-fermions have opposite parity. With our choice of the phase of the parity transformation, fermions have positive parity and antifermions have negative parity. What does this mean for the currents in the interactions? Under the parity operation we nd S: P : V : A: 5 5 0 0 0 5 0 0 0 0 5 0 = = = = 5 0 k 0 k Scalar Pseudo Scalar Vector Axial Vector.

9.4. THE V A INTERACTION

125

The experements by Wu and others made us conclude that parity is violated in the weak interactions. The experiment by Goldhaber showed that it must be vector or axial vector, or both,
V,A

M = GF
i,j

Cij (up Oi up ) (ue Oj u )

(9.20)

Note that the vector and axial vector currents have dierent behaviour under parity. This implies that if a process receives contributions from both currents, then it must be violating parity.

9.4

The V A interaction

The cumulative evidence from many experiments involving weak interactions leads to the conclusion that the weak interaction violates parity maximally. Rather than the vector form assumed by Fermi, the charge-lowering lepton current is actually J = ue
1 2

1 5 u .

(9.21)

The current for quarks looks identical, for example for a u d transition J = uu
1 2

1 5 ud .

(9.22)

We call this the V-A form. For massless particles (or in the ultrarelativistic limit), the projection operator 1 5 (9.23) PL 1 2 selects the left-handed helicity state of a particle spinor and the right-handed helicity state of an anti-particle spinor. As a result, only left handed neutrinos (L ) and righthanded anti-neutrinos ( R ) are involved in weak interactions. For decays of nuclei the structure is more complicated since the constituents are not free elementary particles. The matrix element for neutron decays can be written as GF M = up CV CA 5 un 2 ue 1 5 u . (9.24)

For neutron decay, the vector and axial vector couplings are CV = 1.000 0.003, CA = 1.260 0.002

9.5

The propagator of the weak interaction

The Fermi theory has a 4-point interaction: there is no propagator involved to transmit the interaction from the lepton current to the hadron current. However, we know now that forces are carried by bosons:

126

LECTURE 9. THE WEAK INTERACTION

the electromagnetic interaction is carried by the massless photon which gives rise to a propagator 1 q2 the weak interaction is carried by the massive W , Z bosons, for which we have the propagators: 1 1 and . 2 2 2 MW q MZ q 2
2 At low energies, i.e. when q 2 MZ,W , the q 2 dependence of the propagator vanishes and the interaction looks like a four-point interaction. The charged-current propagator 2 reduces to 1/MW , which allows us to relate Fermis coupling constant to the actual coupling constant g of the weak interaction:

W g g
g2
2 8MW

strength:

G F 2

It is an experimental fact that the strength of the coupling of the weak interaction, the coupling constant g , is identical for quarks and leptons of all avours. For leptons this is sometimes called lepton-universality.
e How weak is the weak interaction? For the electromagnetic coupling we have = 4 2 g 1/137.. It turns out that the weak coupling is equal to w = 4 1/29. We see that at low energies, the weak interaction is weak compared to the electronmagnetic interaction not because the coupling is small, but because the propagator mass is large! At high 2 the weak interaction is comparable in strength to the electromagnetic energies q 2 MW interaction.
2

9.6

Muon decay

Similar to the process e+ e + in QED, the muon decay process e e is the standard example of a weak interaction process.

(p)

e (p)
e(k) (k)
(p)
W

(k) e(p) e(k)

Figure 9.1: Muon decay: left: Labelling of the momenta, right: Feynman diagram. Note that for the spinor of the outgoing antiparticle we use: ue (k ) = ve (k ).

9.6. MUON DECAY

127

Using the Feynman rules we can write for the matrix element: g 1 1 g 1 u(p ) 1 5 M = u(k ) 1 5 u(p) 2 2 MW 2 2 2
outgoing incoming outgoing e propagator

v (k )
outgoing e

(9.25) Next we square the matrix element and sum over the spin states, exactly similar to the case of e+ e + . Then we use again the tric of Casimir as well as the completeness relations to convert the sum over spins into a trace. The result is: 1 |M| = 2
2

1 |M| = 2 Spin
2

g2 2 8MW

Tr 1 5 ( p + me ) 1 5 k Tr 1 5 k 1 5 ( p + m )
G F 2

Now we use some more trace theorems (see below) and also |M| = 64 G2 F (k p ) (k p)
2

g2 2 8MW

to nd the result: (9.26)

Intermezzo: Trace theorems used (see also Halzen & Martin p 261): Tr ( a b ) Tr ( c d ) = 32 [(a c) (b d) + (a d) (b c)] Tr a 5 b Tr c 5 d = 32 [(a c) (b d) (a d) (b c)] Tr 1 5 a 1 5 b Tr 1 5 c 1 5 d = 256 (a c) (b d)

The decay width we can nd by applying Fermis golden rule: d = where : dQ = with :E E = = = =
2 1 |M| dQ 2E d3 p d3 k d3 k (2 )4 4 (p p k k ) 3 3 3 (2 ) 2E (2 ) 2 (2 ) 2 muon energy electron energy electron neutrino energy muon neutrino energy

First we evaluate the expression for the matrix element. We have the relation (p = (m , 0, 0, 0)): p=p +k+k so : (k + p ) = (p k ) (9.27)

128

LECTURE 9. THE WEAK INTERACTION

We can also see the following relations to hold: (k + p ) (p k )


2

= =

k 2 + p 2 +2 (k p )
=0 m2 e 0

p2
2 m2 m

+ k 2 2 (p k )
=0 m

Therefore we have the relation: 2 (k p ) = m2 2m , which we use to rewrite the matrix element as:
2 2 |M| = 64 G2 F (k p ) (k p) = 32 GF m 2m 2

(9.28)

We had the expression for the dierential decay width


2 1 16G2 F (9.29) (m2 2m m dQ |M| dQ = 2E m (E is replaced by m since the decaying muon is in rest). For the total decay width we must integrate over the phase space:

d =

2 1 16G2 F |M| dQ = 2E m

(m2 2m m dQ

(9.30)

We note that the integrand only depends on the neutrino energy . So, let us rst perform the integral in dQ over the other energies and momenta: dQ =
other

1 8 (2 )5 1 = 8 (2 )5

(m E ) 3 (p + k + k ) (m E ) d3 p d3 k E

d3 p d3 k d3 k E

since the -function gives 1 for the integral over k. We also have the relation: = |k | = |p + k | = E 2 + 2 + 2E cos

(9.31)

where is the angle between the electron and the electron neutrino. We choose the z -axis along k , the direction of the electron neutrino. From the equation for we derive: 2E sin d d d = (9.32) d = E sin 2 E 2 + 2 + 2E cos

Next we integrate over d3 p = E 2 sin dE d d with d as above: dQ = 1 E 2 sin d3 k 1 ( m E ) dE d d E 8 (2 )5 1 d3 k = ( m E ) dE d 2 2 8 (2 )5

9.6. MUON DECAY


(using the relation: E sin d = d ).

129

Since we integrate over , the -function will cancel: dQ = 1 8 (2 )4 dE d3 k 2 (9.33)

such that the full expression for becomes: = 2G2 F (2 )4 m2 2m dE d3 k 2 (9.34)

Next we do the integral over k as far as possible with: d3 k = so that we get: = G2 Fm (2 )3 (m 2 ) d dE (9.36) 2 sin d d d = 4 2 d (9.35)

Before we do the integral over we have to determine the limits: maximum electron neutrino energy: =1 m 2 minimum electron neutrino energy: mE =1 2

e e e

Therefore, we obtain for the distribution of the electron energy in the muon rest frame G2 m d = F 3 dE (2 )
1 m 2 1 mE 2

(m 2 ) d =

2 G2 Fm E2 12 3

34

E m

(9.37)

which can be measured experimentally. Finally, integrating the expression over the electron energy we nd for the total decay width of the muon 1 G2 m5 = F 3 192 (9.38)

The measurement of the muon lifetime is the standard method to determine the coupling constant of the weak interaction. The muon lifetime has been measured to be = 2.19703 0.00004 s. From this we derive for the Fermi coupling constant GF = (1.16639 0.00002) 105 GeV2 .

130

LECTURE 9. THE WEAK INTERACTION

9.7

Quark mixing

For the decay of the muon we studied the weak interaction acting between leptons. We have seen in the process of neutron decay that the weak interaction also operates between quarks. All fundamental fermions are susceptible to the weak interaction. Both the leptons and quarks are usually ordered in a representation of three generations: t b (9.39) To leptons and quarks of each generation we assign a quantum number called avour. The strong and electromagnetic interaction conserve avour. That is just another way of saying that they do not couple to currents that connect quarks or leptons from dierent generation doublets. Leptons : Quarks : We could assume that the charged current weak interaction works inside the generation doublets as well, e.g. that we only have Feynman diagrams of the following types: e g W u g W d c g W e g W s t g W g W b e e u d c s

As far as we can tell experimentally, the weak interaction indeed conserves lepton avour. However, consider the decays of charged pions and kaons to a muon and muon antineutrino, 1. pion decay g4 2 M 4 GF
W

u d

g W

2. kaon decay K This decay does occur!


?? W

Since the kaon decay occurs in nature, the weak coupling must violate quark avour conservation.

9.7. QUARK MIXING

131

9.7.1

Cabibbo - GIM mechanism

We have to modify the model by the replacements: d d = d cos c + s sin c s s = d sin c + s cos c or, in matrix representation: d s = cos c sin c sin c cos c d s (9.40)

where c is the Cabibbo mixing angle. In terms of the diagrams the replacement implies:

g W

g W

d =

g cos c W

d +

g sin c W

Both the u, d coupling and the u, s coupling exist. In this case the diagrams of pion decay and kaon decay are modied: 1. Pion decay 2 G2 F cos c 2. Kaon decay K 2 K G2 F sin c

d u

gcos W

gsin W

In order to check this we can compare the decay rate of the two reactions. A proper calculation gives: (K ) tan2 c ( ) m mK
3 2 m2 K m 2 m2 m 2

(9.41)

As a result the Cabibbo mixing angle is observed to be C = 13.0 (9.42)

132

LECTURE 9. THE WEAK INTERACTION

The couplings for the rst two generations are:

g cos c W

g cos c W

g sin c W

g sin c W

Cabibbo favoured decay

Cabibbo suppressed decay

Formulated in a dierent way: The avour eigenstates u, d, s, c are the mass eigenstates. They are the solution of the total Hamiltonian describing quarks; ie. mainly strong interactions. The states u c , are the eigenstates of the weak interaction Hamiltonian, d s which aects the decay of the particles.

The relation between the mass eigenstates and the interaction eigenstates is a rotation matrix: d cos c sin c d = (9.43) s sin c cos c s with the Cabibbo angle as the mixing angle of the generations.

9.7.2

The Cabibbo - Kobayashi - Maskawa (CKM) matrix

We extend the picture of the previous section to include all three generations. This means that we now make the replacement: u d c s t b u d c s t b (9.44)

with in the most general way can be written as: d d Vud Vus Vub s = Vcd Vcs Vcb s b Vtd Vts Vtb b
CKMmatrix

(9.45)

9.7. QUARK MIXING The g couplings involved are:

133

Vud W

Vus W

Vub W

Vcd W

Vcs W

Vcb W

Vtd W

Vts W

Vtb W

It should be noted that the matrix is not uniquely dened since the phases of the quark wave functions are not xed. The standard representation of this unitary 3 3 matrix contains three mixing angles between the quark generations 12 , 13 , 23 , and one complex phase : c12 c13 s12 s13 s13 ei s23 c13 VCKM = s12 c23 c12 s23 s13 ei c12 c23 s12 s23 s13 ei (9.46) i i s12 s23 c12 c23 s13 e c12 s23 s12 c23 s13 e c23 c13 where sij = sin ij and cij = cos ij . In the Wolfenstein parametrization this matrix is: 1 2 /2 A3 ( i ) 1 2 /2 A2 VCKM 3 2 A (1 i ) A 1 It can be easily seen that it has 4 parameters: 3 real parameters : , A, 1 imaginary parameter : i This imaginary parameter is the source of CP violation in the Standard Model. It means that it denes the dierence between interactions involving matter and those that involve anti-matter. We further note that, in case neutrino particles have mass, a similar mixing matrix also exists in the lepton sector. The Pontecorvo-Maki-Nakagawa-Sakata matrix UP M N S is then dened as follows: 1 e U11 U12 U13 = U21 U22 U23 2 (9.48) U31 U32 U33 3
P M N S matrix

(9.47)

134

LECTURE 9. THE WEAK INTERACTION

In a completely similar way this matrix relates the mass eigenstates of the leptons (1 , 2 , 3 ) to the weak interaction eigenstates (e , , ). There is an interesting open question whether neutrinos are their own anti-particles (Majorana neutrinos) or not (Dirac neutrinos). In case neutrinos are of the Dirac type, the UP M N S matrix has one complex phase, similar to the quark mixing matrix. Alternatively, if neutrinos are Majorana particles, the UP M N S matrix includes three complex phases. It is currently not clear whether the explanation for a matter dominated universe lies in quark avour physics (baryogenesis) or in lepton avour physics (leptogenesis) and whether it requires physics beyond the Standard Model. It is however interesting to note that there exist 3 generations of particles!

Exercises
Exercise 9.1 (Helicity versus chirality. See also H&M exercise 5.15) (a) Write out the chirality operator 5 in the Dirac-Pauli representation. , where p is a unit vector along the (b) The helicity operator is dened as = p momentum. Show that in the ultra-relativistic limit (E m) the helicity operator and the chirality operator have the same eect on a spinor solution, i.e. 5 = 5 (s)
p (s) E +m

(s)
p (s) E +m

(c) Explain why the weak interaction is called left-handed. Exercise 9.2 (Pion Decay) Usually at this point the student is asked to calculate pion decay, which requires again quite some calculations. The ambitious student is encourage to try and do it (using some help from the literature). However, the exercise below requires little or no calculation but instead insight in the formalism. (a) Draw the Feynman diagram for the decay of a pion to a muon and an anti-neutrino: . Due to the fact that the quarks in the pion are not free particles we cannot just apply the Dirac formalism for free particle waves. However, we know that the interaction is transmitted by a W and therefore the coupling must be of the type: V or A. (Also, the matrix element must be a Lorentz scalar.) It turns out the decay amplitude has the form: GF M = (q f ) u(p) 1 5 v (k ) 2 where p and k are the 4-momenta of the muon and the neutrino respectively, and q is the 4-momentum carried by the W boson. f is called the decay constant.

9.7. QUARK MIXING

135

(b) Can the pion also decay to an electron and an electron-neutrino? Write down the Matrix element for this decay. Would you expect the decay width of the decay to electrons to be larger, smaller, or similar to the decay width to the muon and muon-neutrino? Base your argument on the available phase space in each of the two cases. The decay width to a muon and muon-neutrino is found to be: G2 2 m m2 = F f 8
2 m2 m m2 2

The measured lifetime of the pion is = 2.6 108 s which means that f m . An interesting observation is to compare the decay width to the muon and to the electron: ( e e ) = ( ) me m
2 2 m2 me 2 m2 m 2

1.2 104 !!

(c) Can you give a reason why the decay rate into an electron and an electron-neutrino is strongly suppressed in comparison to the decay to a muon and a muon-neutrino. Consider the spin of the pion, the handedness of the W coupling and the helicity of the leptons involved.

136

LECTURE 9. THE WEAK INTERACTION

Lecture 10 Local Gauge Invariance


In the last three lectures of this course we discuss the theory of the electroweak interaction, the so-called Glashow-Salam-Weinberg model. This theory can be formulated starting from the principle of local gauge invariance. As we did before, we try to focus on the concepts rather than on formal derivations. A good book on this topic is Gauge Theories of the Strong, Weak, and Electromagnetic Interactions, by Chris, Quigg, in the series of Frontiers in Physics, Benjamin Cummings. You can nd the material for Lecture 10 also in Griths chapter 10 and Halzen and Martin chapter 14.

10.1

Introduction

At the end of Lecture 5 we briey discussed the formulation of the laws of physics in terms of a Lagrangian and the principle of least action (Hamiltons principle). The reason that this approach is popular is that it is particularly suitable to understand symmetries and the conservation laws that follow from them. Symmetries play a fundamental role in particle physics. There is a theorem stating that a symmetry is always related to a quantity that is fundamentally unobservable. In general one can distinguish1 four types of symmetries: permutation symmetries: These lead to Bose-Einstein statistics for particles with integer spin (bosons) and to Fermi-Dirac statistics for particles with half integer spin (fermions). The unobservable is the absolute identity of a particle; continuous space-time symmetries: translation, rotation, acceleration, etc. The related unobservables are respectively: absolute position in space, absolute direction and the equivalence between gravity and acceleration;
1

T.D. Lee: Particle Physics and Introduction to Field Theory

137

138

LECTURE 10. LOCAL GAUGE INVARIANCE

discrete symmetries: space inversion, time inversion, charge inversion. The unobservables are absolute left/right handedness, the direction of time and an absolute denition of the sign of charge; unitary symmetries or internal symmetries, also called gauge invariance: These are the symmetries discussed in this lecture. As an example of an unobservable quantity think of the absolute phase of a complex wave function in quantum mechanics. The relation between symmetries and conservation laws is expressed in a fundamental theorem by Emmy Noether: each continuous symmetry transformation under which the Lagrangian is invariant in form leads to a conservation law. Invariances under external operations as time and space translation lead to conservation of energy and momentum, and invariance under rotation to conservation of angular momentum. Invariances under internal operations, like the rotation of the complex phase of wave functions lead to conserved currents, or more specic, conservation of charge. We discuss the application of Noethers theorem to phase transformations in section 10.3. In our current understanding elementary interactions of the quarks and leptons (electromagnetic, weak and strong) are all the result of gauge symmetries. Starting from a Lagrangian that describes free quarks and leptons, the interactions can be constructed simply by requiring the Lagrangian to be symmetric under particular transformations. The idea of local gauge invariance will be discussed in this rst lecture and will be further applied in the unied electroweak theory in the second lecture. In the third lecture we will calculate the electroweak process e+ e , Z + , using the techniques we have developed.

10.2

The Lagrangian

In Lecture 5 we briey introduced the concept of a Lagrangian and generalized coordinates. We describe the dynamics of a system by a nite set of coordinates qi with time-derivatives (or velocities) q i . For the classical Lagrangian L = T (q ) V (q ) (10.1)

(where the potential energy V only depends on q and the kinetic energy T only on q ), we dened the action (or action integral) of a trajectory (or path) that starts at t1 and ends at t2 with
t1

S (q ) =
t0

L(q, q )dt.

(10.2)

The principle of least action, or Hamiltons principle, states that the actual trajectory q (t) followed by the system is the one for which the action is stationary. That is, for a given point q, q on the trajectory, the change in the action following from a small deviation q, q is zero: S (q, q, q, q ) = 0 (10.3)

10.2. THE LAGRANGIAN

139

You will show in exercise 10.1 that from Hamiltons principle we can derive the EulerLagrange equation of motion for each of the coordinates qi : L d = qi dt L q i (10.4)

The classical theory does not treat space and time symmetrically as the Lagrangian might depend on the parameter t. This causes a problem if we want to make a Lorentz covariant theory. A solution is found by going to eld theory : Rather than a nite set of degrees of freedom we consider an innite set of degrees of freedom, represented by the values of a eld , that is a function of the space-time coordinates x . The Lagrangian is replaced by a Lagrangian density L (usually just called Lagrangian), L(q, q, t) such that the action becomes
x2

L(, , x )

(10.5)

S =
x1

d4 x L (x), (x), x

(10.6)

Following the principle of least action we can also obtain an Euler-Lagrange equation for the elds: L L = (10.7) (x) ( (x)) (If at this point you are confused about the position of Lorentz indices on the righthand-side, then remember that what is meant is L L L L L = + + + (10.8) (x) x0 (/x0 ) x1 (/x1 ) x2 (/x2 ) x3 (/x3 ) You could also use upper indices as long as you are consistent.) To create a Lorentz covariant theory, we require the Lagrangian to be a Lorentz scalar. (This also means that in the expression for L above the loose Lorentz indices must somehow be contracted with others.) This requirement imposes certain conditions on the Lorentz transformation properties of the elds. (We have not discussed these in detail. See textbooks.) Furthermore, although we consider complex elds, we always require the Lagrangian to be real. As we shall see below, this eectively leads to a global U (1) symmetry. In quantum eld theory, the coordinates become operators that obey the standard quantum mechanical commutation relation with their associated generalized momenta. The wave functions that we have considered before can be viewed as single particle excitations that occur when the eld operators act on the vacuum. For the discussions here we do not need eld theory. What is important to know is that eld theory tells us

140

LECTURE 10. LOCAL GAUGE INVARIANCE

that, given a Lagrangian, we can nd a set of Feynman rules that can be used to draw diagrams and compute amplitudes. Now consider the following Lagrangian for a complex scalar eld: L = ( )( ) m2 (10.9)

You will show in an exercise that the equation of motion corresponding to this Lagrangian is the Klein-Gordon equation. Because the eld is complex, it has two separate components. We could choose these to be the real and imaginary part of the eld, such that = 1 + i2 with 1,2 real. It is easy to see what the Lagrangian looks like and what the equations of motion become. However, rather than choosing 1 and 2 we can also choose and to represent the independent components of the eld. A similar argument can be made for the Lagrangian of the Dirac eld.

10.3

Global phase invariance and Noethers theorem

As we have seen above Lagrangians for complex elds are constructed such that the Lagrangian is real (and the Hamiltonian is hermitian). As a result, the Lagrangian is not sensitive to a global shift in the complex phase of the eld. Such a global phase change is called a U (1) symmetry. (The group U (1) is the group of unitary matrices of dimension 1, e.g. complex numbers with unit modulus.) Noethers theorem tells us that there must be a conserved quantity associated with such a phase invariance. For internal symmetries of the Lagrangian this works as follows. Consider a transformation of the eld components with a small variation , i i + i (x) The resulting change in the Lagrangian is L =
i

(10.10)

L i

L ( i )

=
i

L ( i )

(10.11)

where in the second step we have used the Euler-Lagrange equation to remove L/. Consequently, if the Lagrangian is insensitive to the transformation, then the quantity j =
i i

L ( i )

(10.12)

is a conserved current. Lets now apply this to the complex scalar eld for a (small) U (1) phase translation. The two independent eld components are and . Under the phase translation these

10.4. LOCAL PHASE INVARIANCE change as ei (1 + i) ei (1 i)

141

(10.13)

Consequently, we have = i and = i . Inserting these into the expression for the Noether current, Eq. (10.12), we nd j = i ( ) ( ) (10.14)

Since is an arbitrary constant, we omit it from the current. Note that we have obtained exactly the current that we constructed for the Klein-Gordon wave in Lecture 2.

10.4

Local phase invariance

We can also look at the U (1) symmetry from a slightly more general perspective. The expectation value of a quantum mechanical observable (such as the Hamiltonian) is typically of the form: O = O (10.15)

If we make the replacement (x) ei (x) the expectation value of the observable remains the same. We say that we cannot measure the absolute phase of the wave function. (We can only measure relative phases between wave functions in interference experiments.) But this holds for a phase that is constant in space and time. Are we allowed to choose a dierent phase convention on, say, the moon and on earth, for a wave function (x)? In other words, can we choose a phase that depends on space-time, (x) (x) = ei(x) (x)? (10.16)

In general, we cannot do this without breaking the symmetry. The problem is that the Lagrangian density L ( (x), (x)) depends on both on the elds (x) and on the derivatives (x). The derivative term yields: (x) (x) = ei(x) (x) + i (x) (x) and therefore the Lagrangian is not invariant. However, suppose that we now introduce local U (1) symmetry as a requirement. Is it possible to modify the Lagrangian such that it obeys this symmetry? The answer is yes, provided that we introduce a new eld, the so-called gauge eld. The recipe consists of two steps: First, we replace the derivative by the so-called gauge-covariant derivative : D + iqA (10.18) (10.17)

142

LECTURE 10. LOCAL GAUGE INVARIANCE

where A is a new eld and q is (for now) an arbitrary constant. Second, we require that the eld A transforms as 1 A (x) A (x) = A (x) (x) q (10.19)

By inserting the expression for A in the covariant derivative, we nd that it just rotates with the local phase (x): D (x) D (x) = ei(x) 1 (x) + i (x) (x) + iqA (x) (x) iq (x) (x) q (10.20)

= ei(x) D (x)

As a consequence, terms in the derivative that look like D are phase invariant. With the substitution D the Klein-Gordon and Dirac Lagrangians (and any other real Lagrangian that we can construct with 2nd order terms from a complex eld and its derivatives) satisfy the local phase symmetry.

10.5

Application to the Lagrangian for a Dirac eld

Now consider the eect of the replacement of the derivative D + iqA (x) in the Dirac Lagrangian, L = (i D m) = (i m) qA Lfree Lint We dened the interaction term Lint = J A with the familiar Dirac current J = q (10.24) Note that this is exactly the form of the electromagnetic interaction that we discussed in the previous lectures! We can now identify q with the charge and the gauge eld A with the electromagnetic vector potential, i.e. the photon eld. The transformation of the eld in Eq. (10.19) corresponds exactly to the gauge freedom that we identied in the electromagnetic eld in Lecture 3 (with = q). The picture is not entirely complete yet, though. We know that the photon eld satises its own free Lagrangian. This is the Lagrangian that leads to the Maxwell equations. It is given by 1 Lfree (10.25) A = F F 4 (10.23) (10.22) (10.21)

10.6. YANG-MILLS THEORY

143

with F = A A . We call this the kinetic term of the gauge eld Lagrangian and we simply add it to the total Lagrangian. The full Lagrangian for a theory that has one Dirac eld and obeys local U (1) symmetry is then given by 1 LQED = (i m) qA F F 4 This is called the QED Lagrangian. At this point you may wonder if we could also add a mass term for the photon eld. If the photon would have a mass, the corresponding term in the Lagrangian would be L = 1 2 m A A . 2 (10.27) (10.26)

However, this term violates local gauge invariance, since: A A (A ) (A ) = A A (10.28)

Therefore, the requirement of U (1) invariance, automatically implies that the photon is massless. This actually holds for other gauge symmetries as well. Later on, in the PPII course, it will be discussed how masses of vector bosons can be generated in the Higgs mechanism, by breaking the symmetry.

10.6

Yang-Mills theory

The concept of non-abelian gauge theories is introduced here in a somewhat historical context as this helps to also understand the origin of the term weak isospin and the relation to (strong-) isospin. In the 1950s Yang and Mills tried to understand the strong interaction in the protonneutron system in terms of a gauge symmetry. Ignoring the electric charge, we can write the free Lagrangian for the nucleons as L = p (i m) p + n (i m) n Written in terms of the bi-spinor doublet = the Lagrangian becomes L = i m 0 0 i m (10.31) p n , (10.30) (10.29)

Have a careful look at what is written here: The doublet is a 2-component column vector with a Dirac spinor for each component. Each of the entries in the matrix in the Lagrangian is again a 4x4 matrix.

144

LECTURE 10. LOCAL GAUGE INVARIANCE

Note that we have taken the two components to have identical mass m. Because they have identical mass and no charge the nucleons are indistinguishable. Therefore, we consider a global transformation of the eld with a complex unitary (2 2) matrix U that rotates the proton-neutron system, U and U

Since U U = 1, our Lagrangian is invariant to this transformation. Any complex unitary (2 2) matrix can be written in the form U = ei exp i 2 , (10.32)

where and are real and = (1 , 2 , 3 ) are the Pauli spin matrices 2 . We have already considered the eects of a phase transformation ei , which was the U (1) symmetry. Therefore, we concentrate on the case where = 0. Since the matrices all have zero trace, the matrices U with this property all have determinant 1. They form the group SU (2) and the matrices are the generators of this group. (In group theory language we say that SU (2) is a subgroup of U (2) and U (2) = U (1) SU (2).) Note that members of SU (2) do in general not commute. This holds in particular for the generators. We call such groups non-Abelian. In contrast, the U (1) group is Abelian since complex numbers just commute. Using the same prescription as for the U (1) symmetry, we can derive a conserved current. Consider a small SU (2) transformation in doublet space =
1 1+

i 2

(10.33)

and similar for . The Lagrangian transforms as L = L L L L + ( ) + + ( ) L L + = ( ) ( )

(10.34)

where in the second line we have used the Euler-Lagrange equation to eliminate L/ , just as we did in Eq. (10.11). Computing the derivatives of the Lagrangian, we nd that the right term vanishes, while the left term gives 1 L = 2
2

(10.35)

We had labeled these by , but for some obscure reason the textbooks also switch from to at 0 1 0 i 1 0 this point. Our default representation is: 1 = , 2 = , 3 = , 1 0 i 0 0 1

10.6. YANG-MILLS THEORY

145

Since is a constant and since the requirement of phase invariance must hold for any , we can drop and obtain three continuity equations J = 0 for the three conserved currents J = . (10.36) 2 As for the U (1) symmetry, we now try to promote the global symmetry to a local symmetry. The strategy is similar to that for U (1), but because the group is nonAbelian, the implementation is more subtle. The rst step is to make the parameters depend on space time. To simplify the notation we dene the gauge transformation as follows, (x) (x) = G(x) (x) i (x) with G(x) = exp 2

(10.37)

We have again, as in the case of QED, that the derivative transforms non-trivially (x) G ( ) + ( G) (10.38)

such that the Lagrangian is not locally phase invariant. To restore this, we introduce the 2x2 covariant derivative D = 1 1 + igB (10.39) where g is a (so far arbitrary) coupling constant and B a gauge eld. In spinor space the latter is a 2x2 unitary matrix with determinant 1 and it is customary to parametrize it in terms of three new real vector elds b1 , b2 and b3 , B = 1 1 b = 2 2 k bk =
k

1 2

b3 b1 ib2 b1 + ib2 b3

(10.40)

We call the eld bi the gauge elds of the SU (2) symmetry. We need three elds rather than one, because SU (2) has three generators. In terms of the covariant derivative the Lagrangian is now L = (i D 1 1m) (10.41)

In order to obtain a Lagrangian that is invariant, we need the gauge transformation to take the form D D = G (D ) (10.42) Inserting the expression for the covariant derivative, we nd for the term on the left side of the equality D = + igB = G ( ) + ( G) + igB G

(10.43)

146 while the right side is

LECTURE 10. LOCAL GAUGE INVARIANCE

G (D ) = G + igGB Comparing these two expressions we nd that invariance implies that igB (G ) = igG (B ) ( G)

(10.44)

(10.45)

Since this expression must hold for all values of the eld , we can omit the eld from this expression. If we subsequently multiply both sides of the equation on the right by G1 we nd for the transformation of the gauge eld B = GB G1 + i ( G) G1 . g (10.46)

Although this looks rather complicated we can again try to interpret this by comparing to the case of electromagnetism. For Gem = eiq(x) we have
1 A = Gem A G em +

i 1 ( Gem ) G em q (10.47)

= A which is exactly what we had before.

We see that for an SU (2) symmetry the transformation of the gauge eld B involves both a rotation and a gradient. The gradient term was already present in QED. The rotation term is new. It arises due to the non-commutativity of the elements of SU (2). If we write out the gauge eld transformation formula in the components of the real vector elds 1 k k l m bk (10.48) = b klm b g we can see that there is a coupling between the dierent components of the eld. We call this the self-coupling. The eect of this becomes clear if one also considers the kinetic term of the SU (2) gauge eld. Analogous to the QED case, the three new elds require their own free Lagrangian, which we write as Lfree = b 1 4 1 Fl F,l = F F . 4 (10.49)

Mass terms like m2 b b are again excluded by gauge invariance: as for the U (1) symmetry, the gauge elds must be massless. However, while for the photon the eld tensor in the kinetic term was given by F = A A , this form does not work here because it would break the symmetry. Rather, the individual components of the eld tensor are given by (10.50) Fl = b l bl + g jkl bj bk or in vector notation F = b b g b b (10.51)

10.7. HISTORICAL INTERLUDE 1: ISOSPIN, QCD AND WEAK ISOSPIN

147

As a consequence of the last term the Lagrangian contains contributions with 2, 3 and 4 factors of the b-eld. These couplings are respectively referred to as bilinear, trilinear and quadrilinear couplings. In QED there is only the bilinear photon propagator term. In the SU (2) theory there are self interactions by a 3-gauge boson vertex and a 4 gauge boson vertex. Summarizing, we started from the free Lagrangian for a doublet = elds with equal mass,
ree Lf = (i m)

p n

of two

This Lagrangian has a global SU (2) symmetry. We then hypothesized a local SU (2) phase invariance which we could implement by making the replacement D = 1 b . The full Lagrangian of the theory (which is called the + igB with B = 2 Yang-Mills theory) is then given by 1 LSU (2) = (i D m) F F 4 1 = (i m) g J b F F 4 free interaction free L + L + Lb

(10.52)

where we now absorbed the coupling constant g in the denition of the conserved current, g J = 2 Comparing this to the QED Lagrangian 1 ree LU (1) = Lf A J F F 4 (10.54) (10.53)

(with the electromagnetic current J = q ), we see that instead of one eld, we now have three new elds. Furthermore, the kinetic term is more complicated and gives rise to self-coupling vertices with three and four b-eld lines.

10.7

Historical interlude 1: isospin, QCD and weak isospin

Yang and Mills tried to extend the isospin symmetry in the proton-neutron system to a local isospin symmetry in the hope of formulating a viable theory of strong interactions. In order to do so, they needed three new gauge elds, and obviously they wondered what those were. They had to be massless vector bosons that couple to the proton and neutron. Clearly, it could not be the three pions, since those are pseudo-scalar particles rather then vector bosons.

148

LECTURE 10. LOCAL GAUGE INVARIANCE

As we know now, the SU (2) theory cannot describe the strong interaction. Rather the strong interactions follow from an SU (3) symmetry. The implementation is a carbon copy of the Yang-Mills theory for SU (2) symmetry. The mediators of the force are the eight massless gluons, corresponding to the 8 generators of the fundamental representation of SU (3), namely 0 1 0 0 i 0 1 = 1 0 0 2 = i 0 0 3 = 0 0 0 0 0 0 0 0 1 0 0 i 4 = 0 0 0 5 = 0 0 0 6 = 1 0 0 i 0 0 1 0 0 0 0 0 1 7 = 0 0 i 8 = 0 1 0 3 0 0 2 0 i 0 1 0 0 0 1 0 0 0 0 0 0 0 0 0 1 0 1 0

In this case, too, the Lagrangian contains self-coupling terms for the gauge elds. The strong interaction is discusses in the Particle Physics II course. As we have seen in Lecture 1, the isospin symmetry in the proton-neutron is a avour symmetry. Extended to the system of all other hadrons it is essentially just the symmetry between u and d quarks. We know that such a symmetry only exists if we ignore electromagnetic interactions, and the small dierence in mass between the u and the d quark (or the proton and the neutron). Since the symmetry is not exact, we call it an approximate symmetry. Although the Yang-Mills isospin theory is of no real use to the proton-neutron system, it turns out to be exactly what is needed to describe the weak interactions. For historical reasons the local SU (2) symmetry applied to the Lagrangian of Dirac fermion doublets, discussed in the next lecture, is sometimes called weak isospin. It should certainly not be confused with the u d avour symmetry. In contrast with the avour symmetry, gauge symmetries are actually exact symmetries of the Lagrangian.

10.8

Historical interlude 2: the origin of the name gauge theory

The idea of gauge invariance as a dynamical principle is due to Hermann Weyl. He called it eichinvarianz (gauge = calibration). Hermann Weyl3 was trying to nd a geometrical basis for both gravitation and electromagnetism. Although his eort was unsuccessful the terminology survived. His idea is summarized here.
3

H. Weyl, Z. Phys. 56, 330 (1929)

10.8. HISTORICAL INTERLUDE 2: THE ORIGIN OF THE NAME GAUGE THEORY149 Consider a change in a function f (x) between point x and point x + x . If the space has a uniform scale we expect simply: f (x + x) = f (x) + f (x) x (10.55)

But if in addition the scale, or the unit of measure, for f changes by a factor (1 + S x ) between x and x + x, then the value of f becomes: f (x + x) = (f (x) + f (x)x ) (1 + S x ) = f (x) + ( f (x) + f (x)S ) x + O(x)2 So, to rst order, the increment is: f = ( + S ) f x (10.57) (10.56)

In other words Weyl introduced a modied dierential operator by the replacement: + S . One can see this in analogy in electrodynamics in the replacement of the momentum by the canonical momentum parameter: p p qA in the Lagrangian, or in Quantum Mechanics: + iqA , as was discussed in the earlier lectures. In this case the scale is S = iqA . If we now require that the laws of physics are invariant under a change: (1 + S x ) (1 + iq A x ) exp (iq A x ) (10.58) then we see that the change of scale gets the form of a change of a phase. When he later on studied the invariance under phase transformations, he kept using the terminology of gauge invariance.

150

LECTURE 10. LOCAL GAUGE INVARIANCE

Exercises
Exercise 10.1 (Classical Euler-Lagrange equations) Use Hamiltons principle in Eq. (10.3) to derive the Euler-Lagrange equation of motion (Eq. (10.4)). Hint: Write down the change in the action S for an arbitrary small change in the trajectory (q, q ). Now use integration by parts to replace q by q . The boundary condition (the fact that we know where the trajectory starts and ends) makes that q (t0 ) = q (t1 ) = 0. The Euler-Lagrange equations then follow from the fact that for the trajectory that minimizes the action, the change in the action should be zero, independent of q .

Exercise 10.2 (Lagrangians versus equations of motion) (a) Show that the Euler-Lagrange equations of the Lagrangian Lfree KG = 1 1 ( ) ( ) m2 2 2 2 (10.59)

of a real scalar eld leads to the Klein-Gordon equation. (b) Show the same for a complex scaler eld starting from the Lagrangian
2 Lfree KG = ( ) ( ) m

(10.60)

taking and as the (two) independent elds. (Alternatively, you can take the real and imaginary part of . Note that you obtain two equations of motion, one for and one for .) (c) Show that the Euler-Lagrange equations for the Lagrangian
Lfree Dirac = i m

(10.61)

leads to the Dirac equations for and for . Note again that you need to consider and as independent elds. (d) Show that the Lagrangian 1 1 L = LEM = ( A A ) ( A A ) j A = F F j A 4 4 (10.62) leads to the Maxwell equations: ( A A ) = j Hence the current is conserved ( j = 0), since F is antisymmetric. (10.63)

10.8. HISTORICAL INTERLUDE 2: THE ORIGIN OF THE NAME GAUGE THEORY151 Exercise 10.3 (Global phase invariance) (a) Show that the Dirac Lagrangian remains invariant under the global phase transformation (x) eiq (x) ; (x) eiq (x) (10.64)

(b) Noethers Theorem: consider an innitesimal transformation: = ei (1 + i) . Show that the requirement of invariance of the Dirac Lagrangian ( L(, , , ) = 0) leads to the conserved current j = i 2 L L ( ) = (10.65)

Consequently, the charge carried by the current is conserved.

Exercise 10.4 (Local phase invariance) (a) (i) Start with the Lagrange density for a complex Klein-Gordon eld L = ( ) ( ) m2 and show that a local eld transformation: (x) eiq(x) (x) ; (x) eiq(x) (x) (10.67) (10.66)

does not leave the Lagrangian invariant. (ii) Replace now in the Lagrangian D = + iqA and show that the Lagrangian now does remain invariant, provided that the additional eld transforms as A (x) A (x) = A (x) (x) . (10.68) (b) (i) Start with the Lagrange density for a Dirac eld L = i m and show that a local eld transformation: (x) eiq(x) (x) ; (x) eiq(x) (x) (10.70) (10.69)

also does not leave the Lagrangian invariant. (ii) Again make the replacement D = + iqA and show that the Lagrangian remains invariant provided that A transforms are above.

152

LECTURE 10. LOCAL GAUGE INVARIANCE

Exercise 10.5 (Extra exercise, not obligatory!) Consider an innitesimal gauge transformation: G=1+ i 2 || 1 (10.71)

Use the general transformation rule for B and use B = 1 b to demonstrate that 2 the elds transform as: 1 (10.72) b = b b g (Hint: use the Pauli vector identity ( a)( b) = a b + i (a b).)

Lecture 11 Electroweak Theory


In the previous lecture we have seen how imposing a local gauge symmetry requires a modication of the free Lagrangian such that a theory with interactions is obtained. We studied two symmetries, namely local U (1) gauge invariance: (i D m) = (i m) q A
J

(11.1)

local SU (2) gauge invariance for Dirac spinor douplet : g (i D m) = (i m) b 2


J

(11.2)

For the U (1) symmetry we can identify the A eld as the photon. The Feynman rules for QED, as we discussed them in previous lectures, follow automatically. For the SU (2) case we hoped that we could describe the strong nuclear interactions, but this failed. We will now show that the model with an SU (2) local gauge symmetry is still useful, but then to explain the weak interaction. (The strong interaction is derived from SU (3) gauge invariance.) Before we do so, we slightly reformulate the interaction terms above. First consider the U (1) symmetry. Every fermion eld has its own charge. Within the standard model we cannot explain why the charge of an up quark is two-thirds of the charge of an electron. This is why the symbol q appears in the interaction term above: it is a parameter that signies the strength of the interaction and it can be dierent for dierent elds. However, at this point it is customary to introduce a charge operator Q which acts as the generator of the U (1) symmetry group for electromagnetic interactions. It appears in the eld transformation rule as = ei(x)Q 153 (11.3)

154 We dene the conserved current as

LECTURE 11. ELECTROWEAK THEORY

JEM = Q

(11.4)

and write the interaction term in the Hamiltonian as


Lint = e JEM A

(11.5)

where we now use the same coupling e for all elds. The elds are eigenstates of the charge operator Q with an eigenvalue equal to the charge in units of the positron charge e. (Why we do this will become clear later.) A similar thing is done for the isospin symmetry. Rather than as the generator we consider an operator T which for unit isospin charge is given by T = /2. It enters into the douplet transformation rule as = ei(x)T and nds its way into the conserved current as
JT = T

(11.6)

(11.7)

while the interaction terms is given by


Lint = g JT b

(11.8)

When, in the following sections, we consider SU (2) symmetry to generate the weak interaction, the coupling constant g is taken to be the same for all douplets, but each douplet has its own isospin charge under the symmetry operation.

11.1

SU(2) symmetry for left-handed douplets


1 (1 5 ) 2 1 (1 + 5 ) 2

We dene for any Dirac eld the left and right handed projections L R (11.9)

These are called the chiral projections. As we have seen in chapter 9, for particles with E m these correspond to the negative and positive helicity states, respectively. Note that, unless the particle is massless, the chiral projections are not solutions of the Dirac equation even if is a solution. However, using the fact that (see exercise 11.2) = L L + R R where the chiral projections of the adjoint spinors are given by L = 1 (1 + 5 ) 2 and R = 1 (1 5 ) 2 (11.11) (11.10)

11.1. SU(2) SYMMETRY FOR LEFT-HANDED DOUPLETS we can rewrite the Dirac Lagrangian for as L = R (i m) R + L (i m) L .

155

(11.12)

Let us now introduce the following doublets for the left-handed chirality states of the leptons and quarks in the rst family: L = L eL and L = uL dL (11.13)

We call these weak isospin doublets. Again, note that is not a Dirac spinor, but a doublet of Dirac spinors. As an example, consider the Lagrangian for the electron and neutrino and verify that it can be written as L = eR (i me )eR + R (i m ) R + + ( L eL ) i m 0 0 i me L eL (11.14)

Now it comes: We impose a SU (2) gauge symmetry on the left-handed doublets only. That is, we require that the Lagrangian be invariant for local rotations of the doublet. To do this we need to ignore that the two components of a doublet have dierent charge or mass, problems that we will clearly need to deal with later. In fact, the mass problem is big enough, that we need to choose all fermions massless. The fact that we only impose the gauge symmetry on left-handed states leads to a weak interaction that is completely left-right asymmetric. This is why it is referred to as maximal violation of parity. As we shall soon see, the three vector elds (b1 , b2 , b3 from the previous lecture) can be associated with the carriers of the weak interaction, the W + , W , Z bosons. However, these bosons are now massless as well. An explicit mass term (LM = Kb b ) would break the gauge invariance of the theory. Their masses and the masses of all fermions can be generated in a mechanism that is called spontaneous symmetry breaking and involves a new scalar eld, the Higgs eld. The main idea of the Higgs mechanism is that the Lagrangian retains the full gauge symmetry, but that the ground state (or vacuum), i.e. the state from which we start perturbation theory, is not symmetric. The Particle Physics II lectures will discuss this aspect in more detail. To construct the weak SU (2)L theory1 we start again with the free Dirac Lagrangian and we impose SU (2) symmetry on the weak isospin doublets: Lf ree = L i L After introducing the covariant derivative D = + igB with B = T b (11.16) (11.15)

1 The subscript L is used to indicate that we only consider SU (2) transformations of the left-handed doublet.

156

LECTURE 11. ELECTROWEAK THEORY

the Dirac equation obtains an interaction term,


Lfree Lfree g b Jweak

(11.17) (11.18)

where the weak current is


Jweak = L T L

This is just a carbon copy of the Yang-Mills theory for strong isospin in the previous lecture.2 The gauge elds (b1 , b2 , b3 ) couple to the left-handed doublets dened above. However, the particles in our real world do not appear as doublets: we scatter electrons, not electron-neutrino doublets. We now show how these gauge elds can be recast into the physical elds of the 3 (massive) bosons W + , W , Z 0 in order to have them interact with currents of the physical electrons and neutrinos.

11.2

The Charged Current

We choose for the representation of the SU (2) generators the Pauli spin matrices, 1 = 0 1 1 0 2 = 0 i i 0 3 = 1 0 0 1 . (11.19)

Note that 1 and 2 mix the components of a doublet, while 3 does not. We now dene the elds W as follows, 1 2 (11.20) W b1 i b 2 The index on the W refers to the electric charge. However, at this point we have not yet shown that these elds are indeed electrically charged: That would require us to look at the coupling of the W elds to the photon, we will not do as part of these lectures. As an alternative, we now show that these W elds couple to charge-lowering and charge-raising currents. Charge conservation at each Feynman diagram vertex then implies the charge of the gauge boson. We dene the charged current term of the interaction Lagrangian as
1 2 LCC = g b1 g b2 J J

(11.21) 2 L 2 (11.22)

with J 1 = L

1 L 2

J 2 = L

As you will show in exercise 11.1 we can rewrite the charged current Lagrangian as
+ + LCC = g W J g W J

(11.23)

2 Note that in terms of physics strong and weak isospin have nothing to do with one another. It is just that we use the same math!

11.2. THE CHARGED CURRENT with

157

1 J , = L L 2
1 ( 2 1

(11.24)

and =

i2 ), or in our representation + = 0 1 0 0 and = 0 0 1 0 . (11.25)

The leptonic currents can then be written as g J + = L e L 2 and g J = eL L 2 (11.26)

or written out with the left-handed projection operators: 1 1 g J + = 1 + 5 1 5 e 2 2 2 and similar for J . Verify for yourself that 1 + 5 1 5 = 2 1 5 (11.28) (11.27)

such that we can rewrite the leptonic charge raising current (W + ) g J + = 1 5 e 2 2 and for the leptonic charge lowering current (W ) g J = e 1 5 2 2 . (11.30) (11.29)

Remembering that a vector interaction has an operator in the current and an axial vector interaction a term 5 , we recognize in the charged weak interaction the famous V-A interaction. The story for the quark doublet is identical. Drawn as diagrams, the charged currents then look as follows: e Charge raising: W+ e e Charge lowering: W e W d W+ d u u

158

LECTURE 11. ELECTROWEAK THEORY

11.3

The Neutral Current

The third component of the weak isospin gauge eld leads to a neutral current interaction, (11.31) Lint = g b3 J3 with b3 the third gauge boson (another real vector eld) and the conserved current given by 3 J3 = L L (11.32) 2 It is now tempting to identify this third component as the Z 0 boson and simply add the electromagnetic interaction term that we had previously constructed with a U (1) symmetry with the electromagnetic charge operator Q as generator. However, this is not a valid way to extend the symmetry of the Lagrangian: the lefthanded doublets that we have constructed are not eigenfunctions of Q since they mix elds with dierent charge. Therefore, our SU (2)L invariant Lagrangian cannot be symmetric under a transformation with Q as generator, nor even globally. The solution is to start from another U (1) gauge symmetry, called weak hypercharge. We denote its generator with the symbol Y and require that it commutes with the SUL (2) generators. The dierent members of the isospin multiplet then by construction obtain the same value of hypercharge. We denote the combined symmetry by SU (2)L U (1)Y . Under this symmetry a left handed doublet transform as L L = exp [i(x)T + i (x)Y ] L (11.33)

where for T = /2 are the SU (2) generators and Y is the generator for U (1)Y . At the same time, the right-handed components of the elds in the doublet transform only under hypercharge, R R = eiY R (11.34) The conserved current corresponding to the U (1)Y symmetry is
JY = Y

(11.35)

The electroweak Lagrangian following from local SU (2)L U (1)Y symmetry now takes the form (see H&M3 , Chapter 13):
LEW = Lf ree g JT b

g J a 2 Y

(11.36)

where a is the gauge eld corresponding to U (1)Y and g /2 is its coupling strength (the factor 2 following from a convention.)
3

Halzen and Martin, Quarks & Leptons: An Introductory Course in Modern Particle Physics

11.3. THE NEUTRAL CURRENT

159

As an example, consider the neutrino-electron doublet. A left-handed neutrino state has T3 = 1/2 while the left-handed electron has T3 = 1/2. (If you dont understand this, consider the eigenvalues of the eigenvectors L, = 0 and L,e = 0 e

for the 3 /2 generator.) The right-handed electron is a singlet under SU (2)L and has T3 = 0. Given a coupling constant e, the observed electromagnetic charge of the electron is 1. Therefore, the hypercharge of the right-handed electron is 2 while the hypercharge of the left-handed electron and neutrino are both 1. (The latter two must in fact be equal, since the SU (2)L doublet is a singlet under U (1)Y .) The transformations corresponding to T3 and Y both lead to neutral current interactions. As a result the gauge boson elds can actually mix. Neither of them couples specically to the electromagnetic charge. Therefore, an important question is whether we can recast these elds such that one becomes a physical photon eld A that couples to the fermion elds via the charge operator Q and the other one becomes the Z 0 boson. In turns out that the answer to this has everything to do with the Higgs mechanism. The gauge elds b and a in the formalism above are all massless. We have stated before that in the standard model the gauge elds and fermions can be given mass via the Higgs mechanism. We cannot discuss this in more detail in these lectures, but we will use some predictions (or conjectures). The Higgs mechanism leads to a solution in which three out of four vector elds (b and a ) acquire a mass. Two of the massive elds can be written as charge raising and charge lowering elds W as we did before, 1 2 W = b1 i b . 2 (11.37)

The physical neutral elds are linear combinations of the T3 and Y gauge elds, written as A = a cos w + b3 sin w Z = a sin w + b3 cos w (11.38)

where w is called the weak mixing angle. The Higgs mechanism now predicts that in order that the massless eld becomes the photon, its charge operator (or the generator of the original U (1)em symmetry) is related to the SU (2)L and U (1)Y generators as Q = T3 + Y 2 (11.39)

This relation is also called the Gellmann-Nishima relation. Interpreted in terms of quantum numbers, Q is the electromagnetic charge, Y is the hypercharge and T3 is the charge associated to the third generator of SU (2) (i.e. 3 /2). The quantum number for

160

LECTURE 11. ELECTROWEAK THEORY

the hypercharge of the various fermion elds is not predicted by the theory, just as the electromagnetic charges are not predicted.
Expressing the interactions terms of the b 3 and a elds in the Lagrangian above in terms of the physical elds, we nd g JY JY a = g sin w J3 + g cos w 2 2 JY g cos w J3 g sin w 2 eJEM A gZ JN C Z

3 gJ3 b

A Z (11.40)

where in the last line we dened the currents and coupling constants associated to the physical elds. A direct consequence of Eq. (11.39) is that also the currents are related, namely by 1 JEM = J3 + JY 2 . (11.41)

Comparing this to Eq. (11.40) we nd that this implies that e = g sin w = g cos w (11.42)

This relation implies that there are only two independent parameters. In particular, the weak mixing angle is related to the SU (2)L and U (1)Y coupling constants by g /g = tan w Analogously, we nd for the Z 0 interaction term,
g cos w J3

(11.43)

g sin w 2 (JEM J3 ) 2

= ... =

e cos w sin w

Z J3 sin2 w JEM

Consequently, expressed in J3 and JEM the neutral current is


2 JN C = J3 sin w JEM

(11.44)

and its coupling constant becomes gZ = e g = . cos w sin w cos w (11.45)

11.4. COUPLINGS FOR Z F F

161

11.4

Couplings for Z f f

Let us now take a closer look at the neutral current in Eq. (11.44). The J3 current only involves the left-handed doublets, while the electromagnetic current couples both to left and right handed elds. Therefore, we conclude that the neutral current cannot be purely left-handed. We dened the left-handed current in terms of a left-handed neutrino-electron (or updown quark) douplet L as J3 = L T3 L (11.46) In analogy with the procedure that we applied for the charged current, we can now rewrite this in terms of the two fermion elds in the douplet using the explicit representation of T3 as e J3 = L T3 L + eL T3 eL (11.47)
e = 1/2. = 1/2 and T3 where the neutrino and electron weak isospin charges are T3 Since we scatter particles rather douplets, we consider the currents for the neutrino and electron separately. We can then generalize this and write for any fermion eld f f J3 ,f = L T3 L 1 f = (1 5 )T3 2

(11.48)

Adding the contribution from the electromagnetic current, the neutral current for fermion f then becomes
JNC ,f = 1 (1 2

5 )T3,f sin2 w Qf

(11.49)

It is customary to write the term on the right in terms of a vector and an axial vertex coupling such that f f 5 1 JNC CV CA (11.50) ,f = 2 which implies that
f f CV = T3 2Qf sin2 w f f = T3 CA

(11.51)

Alternatively, we can write the current in terms of left- and right-handed elds as
JN C,f =

1 2

f f f f CL L L + CR R R

(11.52)

with the left- and right-handed couplings given by


f f f f CL CV + CA = T3 Qf sin2 w f f f CR CV CA = Qf sin2 w

(11.53)

162

LECTURE 11. ELECTROWEAK THEORY

As stated before the size of the charges for the dierent fermion elds is not predicted. Table 11.1 lists the quantum numbers and resulting couplings for all fermions in the standard model. The model can be experimentally scrutinized by measuring all these couplings. fermion e , , e, , u, c, t d, s, b T3 Y left-handed +1 1 2 1 2 1 1 1 +2 +3 1 1 2 +3 T3 Y right-handed 0 0 0 2 0 +4 3 0 2 3 Q 0 1 2 +3 1 3
f CA f CV

+1 2 1 2 +1 2 1 2

+1 2 1 + 2 sin2 w 2 4 3 sin2 w +1 2 2 1 2 + 3 sin2 w

Table 11.1: Gauge interaction quantum numbers and corresponding vector and axial vector couplings for the fermions in the Standard Model.

Finally, expressed in terms of the left- and right-handed couplings, the Feynman rule corresponding to Z vertex becomes

f Z
0

i f

g 1 f f 5 CA CV cos w 2

11.5

The mass of the W and Z bosons

In Lecture 9 we expressed the charged current coupling for processes with momentum transfer q MW as a four-point interaction. Comparing the expressions to those in this lecture, we can show that the Fermi coupling constant is related to the gauge eld couplings as g2 G = (11.54) 2 8MW 2 For neutral current processes we can also compute the coupling-constant of the fourpoint interaction. It is given by G g2 = 2 8MZ cos2 w 2 (11.55)

The parameter species the relative strength between the charged and neutral current weak interactions. Comparing the two expressions, we have =
2 MW 2 cos2 Mz w

(11.56)

11.5. THE MASS OF THE W AND Z BOSONS

163

Experimentally, this number is 1 within small uncertainties. In fact, this is exactly the prediction of the Higgs mechanism. In the Higgs mechanism the mass generated for the W and Z are respectively MW = 1 vg 2 and MZ = 1 v 2 g2 + g 2 , (11.57)

where v is the so-called vacuum expectation value of the Higgs eld. With g /g = tan w we nd that = 1. We can also turn the argument around, from a measurement of the Fermi coupling constant in charged current and neutral current processes, we can predict the masses of the W and the Z , 2 e = 81 GeV (11.58) MW = 8GF sin w MZ = MW (gz /g ) = MW /cos = 91 GeV (11.59)

Summary
We have introduced a local gauge symmetry SU (2)L U (1)Y to obtain a Lagrangian for electroweak interactions,
g JL b +

g J a 2 Y

(11.60)

The coupling constants g and g are free parameters. We can also take e and sin2 w . The electromagnetic and neutral weak currents are then given by: 1 JEM = J3 + JY 2
2 2 2 JN C = J3 sin w JEM = cos w J3 sin w

JY 2

and the interaction term in the Lagrangian becomes:


eJEM A +

e Z JN cos w sin w C

(11.61)

in terms of the physical elds A and Z . Note that we still have two independent coupling constants (be it e and w or g and g ). Therefore, it is sometimes said that we have not really unied the electromagnetic and the weak interaction, but rather just put them under the same umbrella. Still, there are clear predictions in this model, such as all the relations between the couplings to the dierent fermion eld and the W to Z mass ratio. Up to now the GlashowSalam-Weinberg theory, which predicted the W and Z more than a decade before their experimental conrmation, has gloriously passed all experimental tests.

164

LECTURE 11. ELECTROWEAK THEORY

Exercises
Exercise 11.1 (Charged current interaction) Show how we get from Eq. (11.21) to Eq. (11.23).

Exercise 11.2 (Lagrangian for left and right-handed chirality) Show explicitly that: = L L + R R making use of = L + R and the projection operators
1 2

(11.62)
1 2

(1 5 ) and

(1 + 5 )

Exercise 11.3 (Symmetries) What do you think is the dierence between an exact and a broken symmetry? Which symmetry is involved in the gauge theories below? Which of these gauge symmetries are exact? Why/Why not? (a) U1(Q) symmetry (b) SU2(u-d-avour) symmetry (c) SU3(u-d-s-avour) symmetry (d) SU6(u-d-s-c-b-t) symmetry (e) SU3(colour) symmetry (f) SU2(weak-isospin) symmetry (g) SU5(Grand unied) symmetry (h) SUSY

Lecture 12 The Process ee+ +


12.1 Helicity conservation

In lecture 8 we performed computations of the cross-section of several scattering processes in QED. In lecture 9 we also looked at the decay of the and the charged via the weak interaction. The results of these computations must of course obey to the conservation of total angular momentum. In the processes that we look at, orbital angular momentum plays no role such that we really talk about conservation of spin. (See also section 7.3.) The conservation rule is built into the computations, but sometimes it is still useful to understand how it comes about. In the previous lecture we have dened the chiral projections, L = 1 (1 5 ) 2
1 L = 2 (1 + 5 ) 1 R = 2 (1 + 5 )

R = 1 (1 5 ) 2

(12.1)

The states L and L are respectively the in- and outgoing left-handed particle chiral states, while R and R are the in- and outgoing right-handed particle chiral states. We have argued before that in the ultra-relativistic limit there is a correspondence between helicity states and chiral states. You can formalize this by dening the helicity projection operators as ) P = 1 (1 14 + p 2 ) P = 1 (1 14 p 2 (12.2)

and subsequently show that for any positive energy spinor with momentum pz along the z -axis uR = x u + (1 x) u uL = x u + (1 x) u where we dened x (1 + )2 2(1 + 2 ) with 165 |p| . E+m (12.3)

(12.4)

166

LECTURE 12. THE PROCESS E E + +

In the ultra-relativistic limit, we have x 1 and the correspondence between the chiral states and helicity states is obtained. As a consequence, for any process that would violate helicity conservation in the ultrarelativistic limit, such as the + e+ e decay via the weak interaction, a helicity suppressing factor (1 x) appears in the amplitude. Simply said, this is because the interaction can only couple to a fraction (1 x) of the lepton wave function. You have shown in the previous lecture that for vector coupling, we can decompose the current in the vertex factor as follows = R R + L L (12.5)

This means that a right-handed state only couples to a right-handed state, and a lefthanded state only to a left-handed state. This results holds equally well for an axial vector coupling ( 5 ). It is graphically illustrated in Fig. 12.1. Note that crossing a particle ips its chirality. R or L R L

Figure 12.1: Helicity conservation in vector and axial-vector couplings. left: A right-handed incoming electron scatters into a right-handed outgoing electron and vice versa in a vector or axial vector interaction . right: In the crossed reaction the energy and momentum of one electron is reversed: i.e. in the e+ e pair production a right-handed electron and a left-handed positron (or vice versa) are produced. This is the consequence of a spin=1 force carrier. (In all diagrams time increases from left to right.)

For scalar couplings the situation is exactly opposite, as its decomposition would read = R L + L R . (12.6)

As we shall see in the remainder of this chapter, conservation of helicity has interesting consequences for the e e+ + as well.. For example, it allows us to understand the angular dependence of a the polarized cross-sections without going in detail through the kinematics.

12.2

The cross section of ee+ +

Equipped with the Feynman rules of the electroweak theory we proceed to calculate the cross-section of the electroweak process e e+ , Z + . We study the process in the centre-of-momentum frame,

12.2. THE CROSS SECTION OF E E + +


+

167

e+

with pi the momentum of an incoming electron, pf the momentum of an outgoing muon and cos the angle between the e+ and the + . e M : e
+

e MZ : e
+

Z +

Figure 12.2: Leading order Feynman diagrams contributing to e e+ + .

In lecture 6 we considered this process in QED. At leading order there was only one contribution to the amplitude, namely via an intermediate photon. In the electroweak theory also the amplitude with an intermediate Z 0 boson. The corresponding Feynman diagrams are shown in Fig. 12.2. Once we have computed the relevant amplitudes, the dierential cross-section follows as usual from the golden rule,
2 1 1 pf d (e e+ + ) = |M| 2 d 64 s pi

(12.7)

where the matrix element is the sum of the photon and Z 0 contributions. In lecture 8 we computed the spin-averaged amplitude via a rather lengthy procedure, involving Casimirs track and the trace theorems. Because it is actually a nice illustration of the concept of helicity conservation, we will here follow a slightly dierent approach.

12.2.1

Photon contribution

Consider rst only the matrix element of the photon contribution (evaluated using the Feynman rules, see e.g. appendix A), g (12.8) M = e2 m m 2 e e q We now decompose the spinors in left- and right-handed chirality states, as we did in lecture 11, m m e e = = Lm Lm + Rm Rm Le Le + Re Re .

168 The total amplitude then becomes M e2 = s

LECTURE 12. THE PROCESS E E + +

Lm Lm + Rm rm Le Le + Re Re

(12.9)

where we have also substituted q 2 = s. The matrix element thus consists of four contributions with denite chirality for the four particles in the process. For high energies the chiral projections are helicity states and the four contributions correspond exactly to four polarized amplitudes. Since we can choose a basis with helicity eigenstates, the four contributions do not interfere, and the total cross-section becomes the sum of the amplitudes squared: d unpolarized 1 d + d + + + = { eL eR e e L R + R L d 4 d d L R d + d + + + eR eL e e L R + R L } d d R L where we average over the incoming spins and sum over the nal state spins. Note that the e+ R Le etc. Lets now look in more detail at the helicity dependence (H&M 6.6):

Initial state:

zaxis e+ e zaxis

In the initial state the e and e+ have opposite helicity (as they produce a spin 1 ).

Final state: +

The same is true for the nal state and + .

So, in the center of mass frame, scattering proceeds from an initial state with JZ = +1 or 1 along axis z into a nal state with JZ = +1 or 1 along axis z . Since the interaction proceeds via a photon with spin J = 1 the amplitude for scattering over an angle is then given by the rotation matrices1
iJy dj |jm m m ( ) jm |e
1

(12.11)
(12.10)

See H&M2.2: eiJ2 |j m =


m

dj m m ( ) |j m

and also appendix H in Burcham & Jobes

12.2. THE CROSS SECTION OF E E + +

169

where Jy is the y component of the angular momentum operator ( which is also the generator for rotations around the y -axis). The coecients dj m,m are sometimes called Wigner d-matrices. Computing them is not entirely trivial, so we take the elements that we need from a table (see e.g. H&M exercise 2.6), 1 1 d1 1 1 ( ) = d11 ( ) = (1 + cos ) 2 1 1 d1 11 ( ) = d1 1 ( ) = (1 cos ) 2 For the kinematic factors in the four amplitudes we then get Lm Lm Rm Rm Lm Lm Rm Rm

(12.12)

Le Le = d11 () = Re Re = d+1+1 () = Re Re = d+11 () = Le Le = d+1+1 () =

1 (1 2 1 (1 2 1 (1 2 1 (1 2

+ cos ) + cos ) cos ) cos ) (12.13)

Note that, as a consequence of the helicity conservation, scattering is suppressed in the direction in which the spin the is not aligned with the spin of the e . Using this result the polarized cross-sections become d + d + + + eL eR e e L R = R L d d R L d + d + + + eL eR e e R L = L R d d R L 2 (1 + cos )2 4s 2 = (1 cos )2 4s =

(12.14)

The unpolarised cross section is obtained as the spin-averaged sum, 2 d unpol 1 2 = 2 (1 + cos )2 + (1 cos )2 = 1 + cos2 d 4 4s 4s in agreement with our computation in Lecture 8. . (12.15)

12.2.2

Z 0 contribution

Having written the total cross-section as a sum of polarized amplitudes we are ready to include the contribution from the Z 0 boson amplitude. Using the Feynman rules (see e.g. appendix A) we nd for the matrix
2 g2 g q q /MZ m m 5 e e 5 C C CA e e CV m V A 2 4 cos2 w m q 2 MZ (12.16) We can simplify the Z propagator if we ignore the lepton masses (m s). In that case the Dirac equation becomes:

MZ =

e (i m) = 0

e ( p,e ) = 0

(12.17)

170 Since pe = 1 q we also have: 2

LECTURE 12. THE PROCESS E E + +

1 e ( q ) = 0 2

2 q q /Mz =0

(12.18)

As a result the q q term in the propagator vanishes and we obtain for the matrix element MZ = g 2 1 m m 5 m CV CA m 2 2 2 4 cos w q MZ
e e 5 e CV CA e

. (12.19)

e e 5 We have shown in the previous lecture how the (CV CA ) terms can be written in terms of left- and right-handed couplings. Dening

CR CV CA one nds

and

CL CV + C A

(12.20) (12.21)

CV CA 5 = CR R + CL L . Consequently, the Z 0 amplitude can be written as MZ = g2 1 2 2 4 cos w s MZ


m m CL Lm Lm + CR Rm rm e CL

(12.22)

Le Le +

e CR

Re Re

Comparing this the expression to Eq. (12.9) we realize that we can obtain the polarized cross-sections directly from the results obtained for the QED process. For two of the four contributions we then obtain d 2 + + m e 2 e (1 + cos )2 |1 + r CL CL | e = L R d ,Z L R 4s d 2 + + m e 2 e (1 cos )2 |1 + r CR CL | e = R L d ,Z L R 4s with the relative contribution of the Z and parameterized as r= g2 1 s . 2 2 2 e 4 cos w s Mz (12.24)

(12.23)

The other two helicity conguration follow directly using the relation in Eq. (12.14). Using the connection between the coupling constants GF g2 g2 = = . 2 2 8MW 8MZ cos2 w 2 we can also write r in terms of GF as r=
2 s 2GF MZ . 2 2 e s MZ

(12.25)

(12.26)

12.2. THE CROSS SECTION OF E E + +

171

12.2.3

Correcting for the nite width of the Z 0

Note that the propagator for the massive vectors bosons has a pole at the boson mass: it becomes innitely large for an on-shell (p2 = m2 ) boson. As you can readily see from the expression above, this would lead to an innite cross-section when we tune the beam energies to s = MZ , which is clearly non-physical. The problem is that the propagator does not take into account the nite decay width of the Z 0 . The Z 0 boson is not a stable particle and hence the on-shell Z 0 is actually something with a rather broad mass distribution. We can account for the width by replacing the mass in the propagator with i MZ MZ Z 2 where is the total decay width of real on-shell Z -bosons. A heuristic explanation (Halzen and Martin, 2.10) comes from the following argument. The decay of an unstable particle follows the exponential law | (t)|2 = | (0)|2 et (12.28) (12.27)

where | (0)| is the probability (density) at t = 0 and 1/ is the lifetime. Therefore, the time-dependence of the wave function, which already involves the rest mass, must also include a factor et/2 , or (t) = (0)eimt et/2 (12.29)

Consequently, with the substitution above we can correct the propagator mass for the nite decay width. The lineshape that results from such a propagator is usually called a (spin-1) Breit-Wigner. With the replacement MZ MZ iZ /2 in the propagator, the expression for r becomes 2 2GF MZ s r= 2 2 Z e s MZ i 2 (12.30) 2 2GF MZ s = 2 e2 2 Z + iM s Mz Z Z 4 Note that r is now complex and that the phase of r depends on s.

12.2.4

Total unpolarized cross-section

As a nal step we add up the unpolarized cross-sections and nd d 2 = A0 1 + cos2 + A1 (cos ) d 4s (12.31)

172 with A0 and A1 given by

LECTURE 12. THE PROCESS E E + +

e2 e2 e f + CA A0 = 1 + 2Re(r) CV CV + |r|2 CV e f e f e f CV CA CA CA + 8|r|2 CV A1 = 4Re(r) CA

f f CV + CA

(12.32)

In this expression we replaced CR and CL by CV and CA using the denitions given earlier. Since we have not actually used that the nal state fermions are muons, our result is , provided that we insert the correct weak valid for any process e e+ , Z f f couplings.
e e Assuming lepton universality, we have CV = CV and CA = CA . The expressions for A0 and A1 then become 2 2 2 A0 = 1 + 2 Re(r) CV + |r|2 CV + CA 2 2 2 A1 = 4 Re(r) CA + 8|r|2 CV CA 1 In the Standard Model the coecients for leptons are CA = 2 and CV = 1 + 2 sin2 w 2 . 2

Finally, we consider the total cross-section. In the integrated cross-section the term proportional to cos vanishes and for the other term we use (1 + cos2 ) d = Consequently, the total cross-section becomes ( s) = 42 A0 (s) 3s Z e
+

16 3

(12.33)

(12.34)

To summarize, on the amplitude level there are two diagrams that contribute: e M : e
+

e MZ :

(12.35) +

Using the following notation: d [Z, Z ] = d d [Z ] = d d [, ] = d Z Z Z |r|2 Re (r) 1

12.2. THE CROSS SECTION OF E E + + the expression for the dierential cross-section becomes d d = d d d with d d d d d

173

d d [Z, Z ] + [, Z ] d d 2 [, ] = 1 + cos2 4s 2 2 f2 f2 e f e f e2 e2 CV CA CA cos CV + CA 1 + cos2 + 8CV + CA CV [Z, Z ] = |r| 4s 2 e f e f [, Z ] = Re|r| CV CV 1 + cos2 + 2CA CA cos 4s [, ] +

12.2.5

Near the resonance

Let us take a closer look at the cross-section for beam energies close to the Z 0 mass. One can see from the expression for r, Eq. (12.30) that |r| is maximal for
2 s0 = MZ

2 Z 4

(12.36)

Since the denominator in Eq. (12.30) is purely imaginary for s = s0 , the interference term, which is proportional to Re(r), vanishes at the peak. Dening the kinematic factor in r s (12.37) (s) = 2 Z 2 s Mz 4 + iMZ Z you will show in exercise 12.1(b) that Re() = 1 s0 ||2 s with ||2 = s2 2 2 (s s0 )2 + MZ Z (12.38)

You will also show that at the resonance |r|2 1 such that we can ignore the photon contribution entirely. With neither the interference nor the photon contribution, we have A0 (s)
2 2GF MZ e2 2

s2 e2 e2 + CA CV 2 2 (s s0 )2 + Mz Z

f f CV + CA

(12.39)

: Exactly at the resonance, this gives for the total cross-section to the nal state f f ( s0 ) =
2 G2 F s0 MZ 6 2 Z e2 e2 CV + CA f f CV + CA 2 2

(12.40)

174

LECTURE 12. THE PROCESS E E + +


had [nb]

40
ALEPH DELPHI L3 OPAL

30

20
measurements, error bars increased by factor 10

10

from fit QED unfolded

MZ

86

88

90

92

94

Ecm [GeV]
Figure 12.3: left: The Z -lineshape: the cross-section for e+ e hadrons as a function of s. right: Same but now near the resonance. The Lineshape parameters for the lowest order calculations and including higher order corrections.

12.3

The Z 0 decay widths


f (12.41) f

We can also calculate the decay width: Z ff which is according Fermis golden rule: Z ff = 1 1 2 M 16 MZ Mz g2 f2 f2 CV + CA = 2 48 cos w 3 GF MZ f2 f2 CV + CA = 6 2

(12.42)

Let us now nally consider the case when f = q (a quark). Due to the fact that quarks can be produced in 3 colour states the decay width is:
3 GF MZ (Z qq ) = 6 2 f f CV + CA 2 2

NC

(12.43)

with the colour factor NC = 3. The ratio between the hadronic and leptonic width: Rl = had /lep can be dened. This ratio can be used to test the consistency of the standard model by comparing the calculated value with the observed one.

12.4. FORWARD-BACKWARD ASYMMETRY

175

12.4

Forward-backward asymmetry

A direct consequence of the photon-Z interference is that the angular distribution is not symmetric. Figure 12.4 shows the cos distribution observed at the Jade experiment, operating at the PETRA collider in Hamburg. The beam energy in this experiment was not yet sucient to directly produce Z bosons. Still, the eect of the interference was clearly visible long before the direct discovery of the Z resonance.

Figure 12.4: Angular distribution for e+ e + for s > 25 GeV at the JADE experiment. is the angle between the outgoing + and the incoming e+ . The curves show ts to the data p(1 + cos2 ) + q cos (full curve) and p(1 + cos2 ) (dashed curve). (Source: JADE collaboration, PLB, Vol108B, p108, 1981.)

The forward-backward asymmetry can be dened using the polar angle distribution. At the peak and ignoring the pure photon exchange (because it is negligibly small) 8 d 1 + cos2 + AF B cos d cos 3 where (12.44)

f f 2CV 3 CA ,f A A with A = A0 = . (12.45) e f f FB 2 2 4 CV + CA The precise measurements of the forward-backward asymmetry can be used to determine the couplings CV and CA .

12.5

The number of light neutrinos

Since the total decay width of the Z must be equal to the sum of all partial widths the following relation holds: Z = ee + + + 3uu + 3dd + 3ss + 3cc + 3bb + N (12.46)

176

LECTURE 12. THE PROCESS E E + +

0.022
68% CL

0.018
0,l

Afb

mt s

0.014
ll + ee + +
+

mH

0.01 20.6

20.7

20.8

20.9

Rl

Figure 12.5: left: Test of lepton-universality. The leptonic Af b vs. Rl . The contours show the measurements while the arrows show the dependency on Standard Model parameters. right: Determination of the vector and axial vector couplings.

combined in plots with SLD results

Using all available data to extract information on the couplings we can now determine the decay widths to all nal states within the standard model, ee = 84 MeV = 167 MeV uu cc = 276 MeV dd ss bb = 360 MeV CV 0 CV = 1 2 CA = 1 2 1 CA = 2 CA = CA = 1 2 1 2

CV 0.19 CV 0.35

(Of course tt = 0 since the top quark is heavier than the Z .) A measurement of the lineshape (the cross-section as function of s) gives for the total decay width of the Z , Z 2490 MeV So, even though we cannot see the neutrino contribution, we can estimate the number of neutrinos from the total width of the Z . The result is (see gure 12.6), N = Z 3l had = 2.984 0.008 . (12.47)

This results put strong constraints on extra generations: if there is a fourth generation, then either it has a very heavy neutrino, or its neutrino does not couple weakly. In either case, this generation would be very dierent from the known generations of quarks and leptons.

12.5. THE NUMBER OF LIGHT NEUTRINOS

177

Figure 12.6: The Z -lineshape for resp. N = 2, 3, 4.

Figure 12.7: Standard Model t of the predicted value of the Higgs boson.

178

LECTURE 12. THE PROCESS E E + +

Exercises
Exercise 12.1 (Z production and decay) (a) Derive the expression for the unpolarized dierential cross-section in Eq. (12.31) (with A0 and A1 dened in Eq. (12.32)) starting from the polarized cross-sections in (12.23). (b) Derive the expression for Re() in Eq. (12.38). (c) Calculate the relative contribution of the Z -exchange and the exchange to the cross section at the Z peak. Use sin2 W = 0.23, Mz = 91 GeV and Z = 2.5 GeV . (d) Show also that at the peak peak (e e+ + ) 12 e 2 2 Mz Z (12.48)

(e) Calculate the value of Rl = had /lep at the resonance s = s0 . You may ignore the top quark as it is too heavy. For all other fermions you may ignore their mass, as we did in the lecture. You may also ignore the contribution of the photon, as it is very small at the resonance. (f ) The actual line shape of the Z -boson is not a pure Breit Wigner: at the high s side of the peak the cross section is higher then expected from the formula derived in the lectures. Can you think of a reason why this would be the case? (g) The number of light neutrino generations is determined from the invisible width of the Z -boson as follows: N = Z 3l had

Can you think of another way to determine the decay rate of Z directly? Do you think this method is more precise or less precise?

Appendix A Summary of electroweak theory


Take the Lagrangian of free fermions (leptons and quarks) L =
f

f (i m)f

(A.1)

Arrange the left-handed projections of the lepton and quark elds in doublets L = L eL or L = uL dL (A.2)

Ignore their masses (or choose them equal within the doublet). Now consider that the Lagrangian remains invariant under U (1)Y : = eiY (x) SU (2)L : L L = exp [iY (x) ] L (A.4) (A.3)

To keep the Lagrangian invariant compensating gauge elds must be introduced. These transform simultaneously with the Dirac spinors in the doublet: U (1)Y : hypercharge eld a D = + ig Y a 2 (A.5)

2 3 SU (2)L : weak isospin elds b1 , b , b (only couple to left-handed doublet):

D = + ig b

(A.6)

Ignoring the kinetic and self-coupling terms of the gauge elds, the Lagrangian becomes g a ig JL b L = Lfree i JY 2 179 (A.7)

180

APPENDIX A. SUMMARY OF ELECTROWEAK THEORY

For the generators of SU (2) we choose the Pauli spin matrices. The rst eld in a left-handed doublet has T3 = +1/2 and the second eld T3 = 1/2. By construction the right-handed projections are singlets under SU (3)L and therefore have T3 = 0. The physical gauge elds (connecting the particle elds) become charged currents
W 2 b1 ib = 2

(A.8)

neutral currents Z = a sin w + b3 cos w A = a cos w + b3 sin w (A.9)

The Higgs mechanism takes care that 3 out of 4 gauge bosons get mass. For the eld A (the photon) to be massless, we need tan w = g g (A.10)

The coupling of the massless eld becomes proportional to a charge Q = T3 + 1 Y 2 Furthermore, the W and Z masses obey the relation MW = MZ cos The interaction Lagrangian for the doublet can now be written as g + Lint = u 1 1 5 d W 2 2 g 1 5 u W d 1 2 2 e
f =u,d

(A.11)

(A.12)

Qf f f

(A.13)

gz
f =u,d

f f 5 f 1 CV CA f 2

with e = g sin w and


f f CV = T3 2Qf sin2 w f f CA = T3

gz = g/ cos w

tan w = g /g

(A.14) (A.15)

The relevant quantum numbers for our elds are

181 f u, c, t d, s, b e , , e , , Q 2 +3 1 3 0 1
3 TL +1 2 1 2 3 TR 0

0 0

+1 2
1 2

Till now we have ignored that the weak interaction mixes the quark elds. Inserting the CKM matrix we get for the charged currents, g + 1 5 d W LC.C. = i Vud u 1 2 2 g i Vud d 1 1 5 u W 2 2

(A.16)

182

APPENDIX A. SUMMARY OF ELECTROWEAK THEORY

The Feynman rules for the vertex factors are then as follows f f

i e Qf

Z0

f 5 f i cosgw 1 (Cv CA ) 2

f d

W+

g i 1 (1 5 ) 2 2

W+

g i V 1 (1 5 ) 2 2 ud

while those for the propagators are f W, Z

i ig i(g p p /m2 ) 2 p m p p2 m 2 The photon propagator is not unique: the form above holds in the Lorentz gauge.

Appendix B Some properties of Dirac matrices i and


This appendix lists some properties of the operators i and in the Dirac Hamiltonian: E = i = i + m t

1. i and are hermitian. They have real eigenvalues because the operators E and p are hermitian. (Think of a plane wave equation: = N eip x .) 2. T r(i ) = T r( ) = 0. Since i = i , we have also: i 2 = i . Since 2 = 1, this implies: i = i and therefore T r(i ) = T r(i ) = T r(i 2 ) = T r(i ), where we used that T r(A B ) = T r(B A). 3. The eigenvalues of i and are 1. To nd the eigenvalues bring i , to diagonal form and since (i )2 = 1, the square of the diagonal elements are 1. Therefore the eigenvalues are 1. The same is true for . 4. The dimension of i and matrices is even. The T r(i ) = 0. Make i diagonal with a unitary rotation: U i U 1 . Then, using again T r(AB ) = T r(BA), we nd: T r(U i U 1 ) = T r(i U 1 U ) = T r(i ). Since U i U 1 has only +1 and 1 on the diagonal (see 3.) we have: T r(U i U 1 ) = j (+1) + (n j )(1) = 0. Therefore j = n j or n = 2j . In other words: n is even.

183

Você também pode gostar